Board review anatomy questions

Pataasin ang iyong marka sa homework at exams ngayon gamit ang Quizwiz!

30. A patient with diverticulosis of the colon presents for follow-up to his primary care physician with ongoing complaints of left lower quadrant pain and occasionally bloody stools. His physician begins workup with appropriating test by recalling that the sigmoid colon: (A) Is drained by systemic veins (B) Is a retroperitoneal organ (C) Receives parasympathetic fibers from the vagus nerve (D) Receives its blood from the superior mesenteric artery (E) Has teniae coli and epiploic appendages

30. The answer is E. The sigmoid colon has teniae coli and epiploic appendages. The sigmoid colon receives blood from the inferior mesenteric artery, drains its venous blood through the portal tributaries, has its own mesentery (sigmoid mesocolon, therefore, is not a retroperitoneal organ), and receives parasympathetic preganglionic fibers from the pelvic splanchnic nerve

30. An elderly woman fell at home and fractured the greater trochanter of her femur. Which of the following muscles would continue to function normally? (A) Piriformis (B) Obturator internus (C) Gluteus medius (D) Gluteus maximus (E) Gluteus minimus

30. The Answer is D. The gluteus maximus is inserted into the gluteal tuberosity of the femur and the iliotibial tract. All of the other muscles insert on the greater trochanter of the femur, and their functions are impaired.

30. The drummer of a local band presents to your clinic with hearing loss. Otoscopic examination reveals loss of contraction of the tensor tympani and the stapedius, which prevents damage to the eardrum and middle ear ossicles. These muscles are most likely controlled by which of the following nerves? (A) Chorda tympani and tympanic nerve (B) Trigeminal and facial nerves (C) Auditory and vagus nerves (D) Facial and auditory nerves (E) Trigeminal and accessory nerves

30. The answer is B. The tensor tympani is innervated by the trigeminal nerve, and the stapedius is innervated by the facial nerve. The other nerves are not involved.

30. A radiologist examines posterior-anterior chest radiographs of a 27-year-old victim of a car accident. Which of the following structures forms the right border of the cardiovascular silhouette? (A) Arch of the aorta (B) Pulmonary trunk (C) SVC (D) Ascending aorta (E) Left ventricle

30. The answer is C. A cardiovascular silhouette or cardiac shadow is the contour of the heart and great vessels seen on posterior-anterior chest radiographs. Its right border is formed by the SVC, right atrium, and inferior vena cava; its left border is formed by the aortic arch (aortic knob), pulmonary trunk, left auricle, and left ventricle. The ascending aorta becomes the arch of the aorta and is found in the middle of the heart.

30. A man is unable to hold typing paper between his index and middle fingers. Which of the following nerves was likely injured? (A) Radial nerve (B) Median nerve (C) Ulnar nerve (D) Musculocutaneous nerve (E) Axillary nerve

30. The answer is C. To hold a typing paper, the index finger is adducted by the palmar interosseous muscle, and the middle finger is abducted by the dorsal interosseous muscle. Both muscles are innervated by the ulnar nerve.

30. A 53-year-old bank teller is admitted to a local hospital for surgical removal of a benign pelvic tumor confined within the broad ligament. There is a risk of injuring which of the following structures that lies in this ligament? (A) Ovary (B) Proximal part of the pelvic ureter (C) Terminal part of the round ligament of the uterus (D) Uterine tube (E) Suspensory ligament of the ovary

30. The answer is D. The uterine tubes lie in the broad ligament. The anterior surface of the ovary is attached to the posterior surface of the broad ligament of the uterus. The ureter descends retroperitoneally on the lateral pelvic wall but is crossed by the uterine artery in the base (in the inferomedial part) of the broad ligament. The terminal part of the round ligament of the uterus becomes lost in the subcutaneous tissue of the labium majus. The suspensory ligament of the ovary is a band of peritoneum that extends superiorly from the ovary to the pelvic wall. .

1. A 27-year-old patient exhibits a loss of skin sensation and paralysis of muscles on the plantar aspect of the medial side of the foot. Which of the following nerves is most likely damaged? (A) Common peroneal (B) Tibial (C) Superficial peroneal (D) Deep peroneal (E) Sural

1. The Answer is B. The common peroneal nerve divides into the deep peroneal nerve, which innervates the anterior muscles of the leg and supplies the adjacent skin of the first and second toes, and the superficial peroneal nerve, which innervates the lateral muscles of the leg and supplies the skin on the side of the lower leg and the dorsum of the ankle and foot. The sural nerve supplies the lateral aspect of the foot and the little toe.

1. A 27-year-old man came to his physician with drooping of the upper eyelid (ptosis), a dilated pupil, and a difficulty in focusing on close objects. Furthermore, he has an internal strabismus (medial deviation of the eye) and inability to look inferiorly when the eye is adducted. Which of the following is the most likely cause? (A) Lesion in the medulla (B) Tumor in the optic canal (C) Thrombosis in the cavernous sinus (D) Lesion of the olfactory nerve (E) Fracture of the foramen spinosum

1. The Answer is C. Thrombosis in the cavernous sinus might damage all three CNs (III, IV, VI): lesion of CN III causes ptosis, a dilated pupil, and loss of accommodation; lesion of CN IV causes inability to look inferiorly when adducted; and lesion of CN VI causes the eyeball deviates medially (internal strabismus). Lesion in the medulla may damage CNs IX, X, and XII. Tumor in the optic canal injures the optic nerve and ophthalmic artery. Lesion of the olfactory nerve causes anosmia (loss of smell). Fracture of the foramen spinosum damages the middle meningeal artery.

1. During an outbreak of meningitis at a local college, a 20-year-old student presents to a hospital emergency department complaining of headache, fever, chills, and stiff neck. On examination, it appears that he may have meningitis and needs a lumbar puncture or a spinal tap. Cerebrospinal fluid (CSF) is normally withdrawn from which of the following spaces? (A) Epidural space (B) Subdural space (C) Space between the spinal cord and the pia mater (D) Subarachnoid space (E) Space between the arachnoid and dura mater

1. The Answer is D. Cerebrospinal fluid (CSF) is found in the subarachnoid space, which is a wide interval between the arachnoid layer and the pia mater. The epidural space contains the internal vertebral venous plexus and epidural fat. The subdural space between the arachnoid and the dura contains a little fluid to moisten the meningeal surface. The pia mater closely covers the spinal cord and enmeshes blood vessels on the surfaces of the spinal cord. Thus, the space between the spinal cord and the pia is a potential space.

1. A 22-year-old man presented to his family physician with a laceration of the fibrous sheets or bands that cover his body under the skin and invest the muscles. Which of the following structures would most likely be injured? (A) Tendon (B) Fascia (C) Synovial tendon sheath (D) Aponeurosis (E) Ligament

1. The answer is B. The fascia is a fibrous sheet or band that covers the body under the skin and invests the muscles. Although fasciae are fibrous, tendons connect muscles to bones or cartilage, aponeuroses serve as the means of origin or insertion of a flat muscle, and ligaments connect bones to bones or cartilage. Synovial tendon sheaths are tubular sacs filled with synovial fluid that wrap around the tendons.

1. A 21-year-old patient has a lesion of the upper trunk of the brachial plexus (Erb-Duchenne paralysis). Which of the following is the most likely diagnosis? (A) Paralysis of the rhomboid major (B) Inability to elevate the arm above the horizontal (C) Arm tending to lie in medial rotation (D) Loss of sensation on the medial side of the arm (E) Inability to adduct the thumb

1. The answer is C. A lesion of the upper trunk of the brachial plexus results in a condition called "waiter's tip hand," in which the arm tends to lie in medial rotation because of paralysis of lateral rotators and abductors of the arm. The long thoracic nerve, which arises from the root (C5-C7) of the brachial plexus, innervates the serratus anterior muscle that can elevate the arm above the horizontal. The dorsal scapular nerve, which arises from the root (C5), innervates the rhomboid major. The medial side of the arm receives cutaneous innervation from the medial brachial cutaneous nerve of the medial cord. The adductor pollicis is innervated by the ulnar nerve.

1. A 32-year-old patient who weighs 275 lb comes to the doctor's office. On the surface of the chest, the physician is able to locate the apex of the heart: (A) At the level of the sternal angle (B) In the left fourth intercostal space (C) In the left fifth intercostal space (D) In the right fifth intercostal space (E) At the level of the xiphoid process of the sternum

1. The answer is C. On the surface of the chest, the apex of the heart can be located in the left fifth intercostal space slightly medial to the midclavicular (or nipple) line. The sternal angle is located at the level where the second ribs articulate with the sternum. The xiphoid process lies at the level of T10 vertebra.

1. A 63-year-old man comes to the emergency department with back pain, weakness, and shortness of breath. On examination, he has an aneurysm of the abdominal aorta at the aortic hiatus of the diaphragm. Which of the following pairs of structures would most likely be compressed? (A) Vagus nerve and azygos vein (B) Esophagus and vagus nerve (C) Azygos vein and thoracic duct (D) Thoracic duct and vagus nerve (E) Inferior vena cava (IVC) and phrenic nerve

1. The answer is C. The aortic hiatus of the diaphragm transmits the azygos vein and thoracic duct. The vagus nerve passes through the esophageal hiatus, and the right phrenic nerve may run through the vena caval hiatus.

1. A 38-year-old man has had thyroid surgery to remove his papillary carcinoma. The external laryngeal nerve that accompanies the superior thyroid artery is damaged during the surgery. This injury could result in a severe impairment of function of which of the following? (A) Relaxing the vocal cords (B) Rotating the arytenoid cartilages (C) Tensing the vocal cords (D) Widening the rima glottidis (E) Abducting the vocal cords

1. The answer is C. The external laryngeal nerve innervates the cricothyroid muscle (major tensor), which tenses the vocal cord. The anterior part of the vocalis muscle can tense the vocal cord, and its posterior part can relax the vocal cord. The lateral cricoarytenoid muscle rotates the vocal process of the arytenoids cartilage medially, closing the rima glottides. The rima glottidis is opened (widened) by rotating the vocal process of the arytenoid cartilage laterally by the posterior cricoarytenoid muscle. Other laryngeal muscles adduct the vocal cords.

1. A 68-year-old woman with uterine carcinoma undergoes surgical resection. This cancer can spread directly to the labia majora in lymphatics that follow which of the following structures? (A) Pubic arcuate ligament (B) Suspensory ligament of the ovary (C) Cardinal (transverse cervical) ligament (D) Suspensory ligament of the clitoris (E) Round ligament of the uterus

1. The answer is E. The round ligament of the uterus runs laterally from the uterus through the deep inguinal ring, inguinal canal, and superficial inguinal ring and becomes lost in the subcutaneous tissues of the labium majus. Thus, carcinoma of the uterus can spread directly to the labium majus by traveling in lymphatics that follow the ligament.

10. During a domestic dispute, a 16-year-old boy receives a deep stab wound around the superior angle of the scapula near the medial border, which injures both the dorsal scapular and spinal accessory nerves. Such an injury could result in paralysis or weakness of which of the following muscles? (A) Trapezius and serratus posterior superior (B) Rhomboid major and trapezius (C) Rhomboid minor and latissimus dorsi (D) Splenius cervicis and sternocleidomastoid (E) Levator scapulae and erector spinae

10. The Answer is B. The dorsal scapular nerve innervates the levator scapulae and rhomboid muscles, whereas the accessory nerve innervates the trapezius and sternocleidomastoid muscles. The serratus posterior superior is innervated by ventral primary rami of the spinal nerves, whereas the splenius cervicis and erector spinae are innervated by dorsal primary rami of the spinal nerves.

10. A 20-year-old patient cannot flex and medially rotate the thigh while running and climbing. Which of the following muscles is most likely damaged? (A) Semimembranosus (B) Sartorius (C) Rectus femoris (D) Vastus intermedius (E) Tensor fasciae latae

10. The Answer is E. The tensor fasciae latae can flex and medially rotate the thigh, so this is the muscle most likely damaged. The hamstring muscles (semitendinosus, semimembranosus, and biceps femoris) can extend the thigh and flex the leg. The sartorius can flex the thigh and leg. The rectus femoris can flex the thigh and extend the leg. The vastus intermedius can extend the leg.

10. A thoracic surgeon removed the right middle lobar (secondary) bronchus along with lung tissue from a 57-year-old heavy smoker with lung cancer. Which of the following bronchopulmonary segments must contain cancerous tissues? (A) Medial and lateral (B) Anterior and posterior (C) Anterior basal and medial basal (D) Anterior basal and posterior basal (E) Lateral basal and posterior basal

10. The answer is A. The right middle lobar (secondary) bronchus leads to the medial and lateral bronchopulmonary segments. The right superior lobar bronchus divides into the superior, posterior, and anterior segmental (tertiary) bronchi. The right inferior lobar bronchus has the anterior, lateral, posterior, and anterior segmental bronchi

10. A 33-year-old woman develops Bell palsy. She must be cautious because this can result in corneal inflammation and subsequent ulceration. This symptom results from which of the following conditions? (A) Sensory loss of the cornea and conjunctiva (B) Lack of secretion of the parotid gland (C) Absence of the corneal blink reflex (D) Absence of sweating on the face (E) Inability to constrict the pupil

10. The answer is C. Bell palsy (facial paralysis) can involve inflammation of the cornea, leading to corneal ulceration, which probably is attributable to an absence of the corneal blink reflex. This is due to paralysis of the orbicularis oculi, which closes the eyelid. Sensory loss of the cornea and conjunctiva is due to injury of the ophthalmic nerve. Lack of secretion of the parotid salivary gland is due to injury of the glossopharyngeal, tympanic, or lesser petrosal nerve. Absence of sweating is due to damage of the sympathetic nerve. Inability to constrict the pupil is due to paralysis of the sphincter pupillae or damage of parasympathetic nerve fibers to the sphincter.

10. A patient can move his eyeballs normally and see distant objects clearly but cannot focus on near objects. This condition may indicate damage to which of the following structures? (A) Ciliary ganglion and oculomotor nerve (B) Oculomotor nerve and long ciliary nerve (C) Short ciliary nerves and ciliary ganglion (D) Superior cervical ganglion and long ciliary nerve (E) Oculomotor, trochlear, and abducens nerves

10. The answer is C. Damage to the parasympathetic ciliary ganglion and parasympathetic fibers in the short ciliary nerve impairs the ability to focus on close objects (accommodation). Because the patient can move his eyeballs normally, the oculomotor nerve is not damaged even if this nerve contains preganglionic parasympathetic fibers. The patient is able to see distant objects clearly because the long ciliary nerve also carries sympathetic fibers to the dilator pupillae. The ability to move the eyeball normally indicates that the oculomotor, trochlear, and abducens nerves are intact.

10. A 7-year-old girl comes to the emergency department with severe diarrhea. Tests show that the diarrhea is due to decreased capacity of normal absorption in one of her organs. Which of the following organs is involved? (A) Stomach (B) Gallbladder (C) Large intestine (D) Liver (E) Pancreas

10. The answer is C. The large intestine absorbs water, salts, and electrolytes. Hence, the patient's diarrhea stems from an absorption problem. The stomach mixes food with mucus and gastric juice, which contains hydrochloric acid and enzymes, and forms chyme. The gallbladder receives bile, concentrates it, and stores it. The liver produces bile, whereas the pancreas secretes pancreatic juice, which contains digestive enzymes and which releases hormones, such as insulin and glucagon.

10. A 35-year-old man walks in with a stab wound to the most medial side of the proximal portion of the cubital fossa. Which of the following structures would most likely be damaged? (A) Biceps brachii tendon (B) Radial nerve (C) Brachial artery (D) Radial recurrent artery (E) Median nerve

10. The answer is E. The contents of the cubital fossa from medial to lateral side are the median nerve, the brachial artery, the biceps brachii tendon, and the radial nerve. Thus, the median nerve is damaged. The radial recurrent artery ascends medial to the radial nerve

10. A 27-year-old woman has suffered a gunshot wound to her midabdomen. After examining the patient's angiogram, a trauma surgeon locates the source of bleeding from pairs of veins that typically terminate in the same vein. Which of the following veins are damaged? (A) Left and right ovarian veins (B) Left and right gastroepiploic veins (C) Left and right colic veins (D) Left and right suprarenal veins (E) Left and right hepatic veins

10. The answer is E. The right and left hepatic veins drain into the inferior vena cava (IVC). The right gastroepiploic vein drains into the superior mesenteric vein, but the left one drains into the splenic vein. The right gonadal and suprarenal veins drain into the IVC, whereas the left ones drain into the left renal vein. The right colic vein ends in the superior mesenteric vein, but the left one terminates in the inferior mesenteric vein.

10. A 46-year-old woman has a history of infection in her perineal region. A comprehensive examination reveals a tear of the superior boundary of the superficial perineal space. Which of the following structures would most likely be injured? (A) Pelvic diaphragm (B) Colles fascia (C) Superficial perineal fascia (D) Deep perineal fascia (E) Perineal membrane

10. The answer is E. The superior (deep) boundary of the superficial perineal space is the perineal membrane (inferior fascia of the urogenital diaphragm). Colles fascia is the deep membranous layer of the superficial perineal fascia. The deep perineal fascia essentially divides the superficial perineal space into a superficial and deep compartment. The pelvic diaphragm consists of the levator ani and coccygeus muscles.

11. A 21-year-old man was involved in a motorcycle accident, resulting in destruction of the groove in the lower surface of the cuboid bone. Which of the following muscle tendons is most likely damaged? (A) Flexor hallucis longus (B) Peroneus brevis (C) Peroneus longus (D) Tibialis anterior (E) Tibialis posterior

11. The Answer is C. The groove in the lower surface of the cuboid bone is occupied by the tendon of the peroneus longus muscle. The flexor hallucis longus tendon occupies a groove on the posterior surface of the body of the talus and a groove on the inferior surface of the calcaneus during its course. The tibialis posterior muscle tendon occupies the medial malleolar groove of the tibia. Other muscle tendons are not in the groove of the tarsal bones.

11. An elderly man at a nursing home is known to have degenerative brain disease. When cerebrospinal fluid (CSF) is withdrawn by lumbar puncture for further examination, which of the following structures is most likely penetrated by the needle? (A) Pia mater (B) Filum terminale externum (C) Posterior longitudinal ligament (D) Ligamentum flavum (E) Annulus fibrosus

11. The Answer is D. The cerebrospinal fluid (CSF) is located in the subarachnoid space, between the arachnoid layer and the pia mater. In a lumbar puncture, the needle penetrates the skin, fascia, ligamentum flavum, epidural space, dura mater, subdural space, and arachnoid mater. The pia mater forms the internal boundary of the subarachnoid space; thus, it cannot be penetrated by needle. The posterior longitudinal ligament lies anterior to the spinal cord; thus, it is not penetrated by the needle. The filum terminale externum is the downward prolongation of the spinal dura mater from the second sacral vertebra to the dorsum of the coccyx. The annulus fibrosus consists of concentric layers of fibrous tissue and fibrocartilage surrounding and retaining the nucleus pulposus of the intervertebral disk, which lies anterior to the spinal cord.

11. A 58-year-old man is diagnosed as having a slowly growing tumor in the deep perineal space. Which of the following structures would most likely be injured? (A) Bulbourethral glands (B) Crus of penis (C) Bulb of vestibule (D) Spongy urethra (E) Great vestibular gland

11. The answer is A. The deep perineal space contains the bulbourethral (Cowper) glands. The crus of the penis, bulb of the vestibule, spongy urethra, and great vestibular gland are found in the superficial perineal space. .

11. The bronchogram of a 45-year-old female smoker shows the presence of a tumor in the eparterial bronchus. Which airway is most likely blocked? (A) Left superior bronchus (B) Left inferior bronchus (C) Right superior bronchus (D) Right middle bronchus (E) Right inferior bronchus

11. The answer is C. The eparterial bronchus is the right superior lobar (secondary) bronchus; all of the other bronchi are hyparterial bronchi.

11. A 43-year-old man complains of abdominal pain just above his umbilicus. On examination, a tumor is found anterior to the IVC. Which of the following structures would most likely be compressed by this tumor? (A) Right sympathetic trunk (B) Left third lumbar artery (C) Third part of the duodenum (D) Left renal artery (E) Cisterna chyli

11. The answer is C. The third part of the duodenum (transverse portion) crosses anterior to the IVC. The other structures do not cross the IVC anteriorly.

11. A 32-year-old woman has hoarseness in her voice, and her uvula is deviated to the left on phonation. Which of the following nerve is most likely damaged? (A) Right trigeminal nerve (B) Left trigeminal nerve (C) Right vagus nerve (D) Left vagus nerve (E) Left glosopharyngeal nerve

11. The answer is C. The vagus nerve innervates the musculus uvulae. A lesion of the vagus nerve causes deviation of the uvula toward the opposite side of the injury. Because her uvula deviates to the left on phonation, the right vagus nerve is damaged. Hoarseness is caused by a paralysis of the laryngeal muscles resulting from damage to skeletal motor fibers in the recurrent laryngeal branch of the vagus nerve.

11. The police bring in a murder suspect who has been in a gunfight with a police officer. The suspect was struck by a bullet in the arm; his median nerve has been damaged. Which of the following symptoms is likely produced by this nerve damage? (A) Waiter's tip hand (B) Claw hand (C) Wrist drop (D) Ape hand (E) Flattening of the hypothenar eminence

11. The answer is D. Injury to the median nerve produces the ape hand (a hand with the thumb permanently extended). Injury to the radial nerve results in loss of wrist extension, leading to wrist drop. Damage to the upper trunk of the brachial plexus produces waiter's tip hand. A claw hand and flattening of the hypothenar eminence or atrophy of the hypothenar muscles result from damage to the ulnar nerve.

11. A 39-year-old woman presents to your clinic with complaints of headache and dizziness. She has an infection of a cranial dural sinus. The sinus that lies in the margin of the tentorium cerebelli and runs from the posterior end of the cavernous sinus to the transverse sinus is infected. Which of the following sinuses is affected by inflammation? (A) Straight sinus (B) Inferior sagittal sinus (C) Sphenoparietal sinus (D) Superior petrosal sinus (E) Cavernous sinus

11. The answer is D. The superior petrosal sinus runs from the cavernous sinus to the transverse sinus along the attached margin of the tentorium cerebelli. This patient has meningitis (inflammation of the meninges), which causes headache and dizziness. The straight sinus runs along the line of attachment of the falx cerebri to the tentorium cerebelli; the inferior sagittal sinus lies in the free edge of the falx cerebri; the sphenoparietal sinus lies along the posterior edge of the lesser wing of the sphenoid bone; the cavernous sinus lies on each side of the sella turcica and the body of the sphenoid bone.

11. A 16-year-old girl with urinary diseases comes to a local hospital. Her urologist's examination and laboratory test results reveal that she has difficulty in removing wastes from the blood and in producing urine. Which of the following organs may have abnormal functions? (A) Ureter (B) Spleen (C) Urethra (D) Bladder (E) Kidney

11. The answer is E. The urinary system includes the kidneys, which remove wastes from the blood and produce the urine; the ureters, which carry urine; the urinary bladder, which stores urine; and the urethra, which conveys urine from the bladder to the exterior of the body. The spleen filters blood to remove particulate matter and cellular residue, stores red blood cells, and produces lymphocytes. Because the patient is not producing urine properly, the malfunctioning organs are the kidneys.

12. An automobile body shop worker has his middle finger crushed while working on a transmission. Which of the following muscles is most likely to retain function? (A) Extensor digitorum (B) Flexor digitorum profundus (C) Palmar interosseous (D) Dorsal interosseous (E) Lumbrical

12. The answer is C. The extensor digitorum, flexor digitorum profundus, dorsal interosseous, and lumbrical muscles are attached to the middle digit, but no palmar interosseous muscle is attached to the middle digit.

12. A 27-year-old stuntman is thrown out of his vehicle prematurely when the car used for a particular scene speeds out of control. His spinal cord is crushed at the level of the fourth lumbar spinal segment. Which of the following structures would most likely be spared from destruction? (A) Dorsal horn (B) Ventral horn (C) Lateral horn (D) Gray matter (E) Pia mater

12. The Answer is C. The lateral horns, which contain sympathetic preganglionic neuron cell bodies, are present between the first thoracic and second lumbar spinal cord levels (T1-L2). The lateral horns of the second, third, and fourth sacral spinal cord levels (S2-S4) contain parasympathetic preganglionic neuron cell bodies. The entire spinal cord is surrounded by the pia mater and has the dorsal horn, ventral horn, and gray matter. Note that the fourth lumbar spinal cord level is not the same as the fourth vertebral level.

12. A construction worker falls feet first from a roof. He sustains a fracture of the groove on the undersurface of the sustentaculum tali of the calcaneus bone. Which of the following muscle tendons is most likely torn? (A) Flexor digitorum brevis (B) Flexor digitorum longus (C) Flexor hallucis brevis (D) Flexor hallucis longus (E) Tibialis posterior

12. The Answer is D. The tendon of the flexor hallucis longus muscle occupies first the groove on the posterior surface of the talus and then the groove on the undersurface of the sustentaculum tali. None of the other tendons would have been affected in such an injury.

12. An elderly man with a benign enlargement of his prostate experiences difficulty in urination, urinary frequency, and urgency. Which of the following lobes of the prostate gland is commonly involved in benign hypertrophy that obstructs the prostatic urethra? (A) Anterior lobe (B) Middle lobe (C) Right lateral lobe (D) Left lateral lobe (E) Posterior lobe

12. The answer is B. The middle lobe of the prostate gland is commonly involved in benign prostatic hypertrophy, resulting in obstruction of the prostatic urethra, whereas the posterior lobe is commonly involved in carcinomatous transformation. The anterior lobe contains little glandular tissue, and the two lateral lobes on either side of the urethra form the major part of the gland.

12. A 53-year-old man with a known history of emphysema is examined in the emergency department. Laboratory findings along with examination indicate that the patient is unable to exchange oxygen in the air and carbon dioxide in the blood. This exchange occurs in which portion of the respiratory system? (A) Bronchi (B) Alveolar (air) sac (C) Nasal cavity (D) Larynx (E) Trachea

12. The answer is B. The respiratory portion of the lung contains the alveolar (air) sacs or alveoli, which are surrounded by networks of pulmonary capillaries. Oxygen and carbon dioxide exchange occurs across the thin walls of the alveoli and blood capillaries with the aid of the diaphragm and thoracic cage. The nasal cavity, larynx, trachea, and bronchi are air-conducting portions.

12. A 24-year-old man falls from his motorcycle and lands in a creek. Death may result from bilateral severance of which of the following nerves? (A) Trigeminal nerve (B) Facial nerve (C) Vagus nerve (D) Spinal accessory nerve (E) Hypoglossal nerve

12. The answer is C. Bilateral severance of the vagus nerve (CN X) causes a loss of reflex control of circulation because of an increase in heart rate and blood pressure; poor digestion results because of decreased gastrointestinal motility and secretion; and difficulty in swallowing, speaking, and breathing occurs because of paralysis of laryngeal and pharyngeal muscles. All of these effects may result in death. Bilateral severance of other nerves does not cause death.

12. A 33-year-old man with a perforated gastric ulcer complains of excruciating pain in his stomach. It is observed that the pain comes from peritoneal irritation by gastric contents in the lesser sac. Which of the following nerves contain sensory nerve fibers that convey this sharp, stabbing pain? (A) Vagus nerves (B) Greater splanchnic nerves (C) Lower intercostal nerves (D) White rami communicantes (E) Gray rami communicantes

12. The answer is C. Pain sensation originating from peritoneal irritation by gastric contents in the lesser sac is carried by lower intercostals nerves. The vagus nerves carry sensory fibers associated with reflexes in the gastrointestinal (GI) tract. The greater splanchnic nerves and white rami communicantes carry pain (general visceral afferent [GVA]) fibers from the wall of the stomach and other areas of the GI tract. The gray rami communicantes contains no sensory fibers but contain sympathetic postganglionic fibers.

12. Following a penetrated injury in the submandibular triangle, the tongue of a 45-yearold patient deviates to the left on protrusion. Which of the following nerves is injured? (A) Right lingual nerve (B) Left lingual nerve (C) Right hypoglossal nerve (D) Left hypoglossal nerve (E) Left glossopharyngeal nerve

12. The answer is D. A lesion of the hypoglossal nerve causes deviation of the tongue toward the injured side on protrusion. The lingual and glossopharyngeal nerves do not supply the tongue muscles.

12. An 83-year-old man with a typical coronary circulation has been suffering from an embolism of the circumflex branch of the left coronary artery. This condition would result in ischemia of which of the following areas of the heart? (A) Anterior part of the left ventricle (B) Anterior interventricular region (C) Posterior interventricular region (D) Posterior part of the left ventricle (E) Anterior part of the right ventricle

12. The answer is D. The circumflex branch of the left coronary artery supplies the posterior portion of the left ventricle. The anterior interventricular artery supplies the anterior aspects of the right and left ventricles and the anterior interventricular septum.

13. A 24-year-old woman comes to a hospital to deliver her baby. Her obstetrician uses a caudal anesthesia during labor and childbirth to block the spinal nerves in the epidural space. Local anesthetic agents are most likely injected via which of the following openings? (A) Intervertebral foramen (B) Sacral hiatus (C) Vertebral canal (D) Dorsal sacral foramen (E) Ventral sacral foramen

13. The Answer is B. Caudal (epidural) anesthesia is used to block the spinal nerves in the epidural space by injecting local anesthetic agents via the sacral hiatus located between the sacral cornua. An intervertebral foramen transmits the dorsal and ventral primary rami of the spinal nerves. The vertebral canal accommodates the spinal cord. Dorsal and ventral sacral foramina transmit the dorsal and ventral primary rami of the sacral nerves.

13. A thoracic surgeon is going to collect a portion of the greater saphenous vein for coronary bypass surgery. He has observed that this vein runs (A) Posterior to the medial malleolus (B) Into the popliteal vein (C) Anterior to the medial condyles of the tibia and femur (D) Superficial to the fascia lata of the thigh (E) Along with the femoral artery

13. The Answer is D. The greater saphenous vein ascends superficial to the fascia lata. It courses anterior to the medial malleolus and posterior to the medial condyles of the tibia and femur and terminates in the femoral vein by passing through the saphenous opening. The small saphenous vein drains into the popliteal vein. The greater saphenous vein does not run along with the femoral artery.

13. A 26-year-old woman has an amenorrhea, followed by uterine bleeding, pelvic pain, and pelvic mass. Her obstetrician performed a thorough examination, and the patient was diagnosed as having an ectopic pregnancy. Which of the following organs is most likely to provide a normal site of fertilization? (A) Fundus of the uterus (B) Ampulla of the uterine tube (C) Fimbriae (D) Infundibulum of the uterine tube (E) Body of the uterus

13. The answer is B. Fertilization occurs in the ampulla of the uterine tube, and a fertilized oocyte forms a blastocyst by day 7 after fertilization and becomes embedded or implanted in the wall of the uterus during the progestational (secretory) phase of the menstrual cycle. Fertilization is the process beginning with the penetration of the secondary oocyte by the sperm and completed by fusion of the male and female pronuclei.

15. A 26-year-old heavyweight boxer was punched on his mandible, resulting in a slight subluxation (dislocation) of the atlantoaxial joint. The consequence of the injury was decreased range of motion at that joint. What movement would be most affected? (A) Extension (B) Flexion (C) Abduction (D) Adduction (E) Rotation

15. The Answer is E. The atlantoaxial joints are synovial joints that consist of two plane joints and one pivot joint and are involved primarily in rotation of the head. Other movements do not occur at this joint.

13. A 25-year-old man is involved in an automobile accident and slams his head into a concrete wall of a bridge. His CT scan reveals that the middle meningeal artery has ruptured but the meninges remain intact. Blood leaking from this artery enters which of the following spaces? (A) Subarachnoid space (B) Subdural space (C) Epidural space (D) Subpial space (E) Cranial dural sinuses

13. The answer is C. Rupture of the middle meningeal artery in the cranial cavity causes an epidural hemorrhage. Subarachnoid hemorrhage is due to rupture of cerebral arteries and veins. Subdural hematoma is due to rupture of bridging cerebral veins as they pass from the brain surface into one of the venous sinuses. Subpial hemorrhage is due to damage to the small vessels of the pia and brain tissue. Cranial dural sinuses normally contain venous blood.

13. A young boy is brought to the hospital after a bicycle accident and possible pelvic fracture. While awaiting a computed tomography (CT) scan of his pelvis, a physician proceeds with a focal neurologic examination. In testing the child's reflexes, which of the following nerves would carry afferent impulses of the cremasteric reflex? (A) Subcostal nerve (B) Lateral femoral cutaneous nerve (C) Genitofemoral nerve (D) Iliohypogastric nerve (E) Femoral nerve

13. The answer is C. Stimulation of the cremaster muscle draws the testis up from the scrotum toward the superficial inguinal ring. The efferent limb of the reflex arc is the genital branch of the genitofemoral nerve, whereas the afferent limb is the femoral branch of the genitofemoral nerve. The other nerves are not involved in the cremasteric reflex.

13. A 44-year-old man with a stab wound was brought to the emergency department, and a physician found that the patient was suffering from a laceration of his right phrenic nerve. Which of the following conditions has likely occurred? (A) Injury to only GSE fibers (B) Difficulty in expiration (C) Loss of sensation in the fibrous pericardium and mediastinal pleura (D) Normal function of the diaphragm (E) Loss of sensation in the costal part of the diaphragm

13. The answer is C. The phrenic nerve supplies the pericardium and mediastinal and diaphragmatic (central part) pleura and the diaphragm, an important muscle of inspiration. It contains general somatic efferent (GSE), general somatic afferent (GSA), and GVE (postganglionic sympathetic) fibers. The costal part of the diaphragm receives GSA fibers from the intercostal nerves.

13. A 59-year-old man is diagnosed with prostate cancer following a digital rectal examination. For the resection of prostate cancer, it is important to know that the prostatic ducts open into or on which of the following structures: (A) Membranous part of the urethra (B) Seminal colliculus (C) Spongy urethra (D) Prostatic sinus (E) Prostatic utricle

13. The answer is D. Ducts from the prostate gland open into the prostatic sinus, which is a groove on either side of the urethral crest. The prostate gland receives the ejaculatory duct, which opens into the prostatic urethra on the seminal colliculus (a prominent elevation of the urethral crest) just lateral to the prostatic utricle, which is a small blind pouch. The bulbourethral gland lies on the lateral side of the membranous urethra within the deep perineal space, but its duct opens into the bulbous portion of the spongy (penile) urethra.

13. A 47-year-old man cannot move his eyeball laterally. Which of the following conditions would cause such a clinical sign? (A) Tumor of the pituitary gland (B) Occlusion of the posterior cerebral artery (C) Infection in the maxillary sinus (D) Infection in the cavernous sinus (E) Tumor in the anterior cranial fossa

13. The answer is D. The abducens nerve, which innervates the lateral rectus muscle, runs through the middle of the cavernous sinus. The other conditions listed do not injure the abducens nerve. A tumor in the pituitary gland may injure the optic chiasma, causing bitemporal hemianopsia.

13. A 14-year-old boy falls on his outstretched hand and has a fracture of the scaphoid bone. The fracture is most likely accompanied by a rupture of which of the following arteries? (A) Brachial artery (B) Ulnar artery (C) Deep palmar arterial arch (D) Radial artery (E) Princeps pollicis artery

13. The answer is D. The scaphoid bone forms the floor of the anatomic snuffbox, through which the radial artery passes to enter the palm. The radial artery divides into the princeps pollicis artery and the deep palmar arch.

14. In a freak hunting accident, a 17-year-old boy was shot with an arrow that penetrated into his suboccipital triangle, injuring the suboccipital nerve between the vertebral artery and the posterior arch of the atlas. Which of the following muscles would be unaffected by such a lesion? (A) Rectus capitis posterior major (B) Semispinalis capitis (C) Splenius capitis (D) Obliquus capitis superior (E) Obliquus capitis inferior

14. The Answer is C. The splenius capitis is innervated by dorsal primary rami of the middle and lower cervical nerves. The suboccipital nerve (dorsal primary ramus of C1) supplies the muscles of the suboccipital area, including the rectus capitis posterior major, obliquus capitis superior and inferior, and the semispinalis capitis.

14. A 52-year-old woman slipped and fell and now complains of being unable to extend her leg at the knee joint. Which of the following muscles was paralyzed as a result of this accident? (A) Semitendinosus (B) Sartorius (C) Gracilis (D) Quadriceps femoris (E) Biceps femoris

14. The Answer is D. The quadriceps femoris muscle includes the rectus femoris muscle and the vastus medialis, intermedialis, and lateralis muscles. They extend the leg at the knee joint. The semitendinosus, semimembranosus, and biceps femoris muscles (the hamstrings) extend the thigh and flex the leg. The sartorius and gracilis muscles can flex the thigh and the leg.

14. A 29-year-old woman with a ruptured ectopic pregnancy is admitted to a hospital for culdocentesis. A long needle on the syringe is most efficiently inserted through which of the following structures? (A) Anterior fornix of the vagina (B) Posterior fornix of the vagina (C) Anterior wall of the rectum (D) Posterior wall of the uterine body (E) Posterior wall of the bladder

14. The answer is B. A needle should be inserted through the posterior fornix just below the posterior lip of the cervix while the patient is in the supine position to aspirate abnormal fluid in the cul-de-sac of Douglas (rectouterine pouch). Rectouterine excavation is not most efficiently aspirated by puncture of other structures.

14. A 29-year-old woman with abdominal pain was admitted to a local hospital, and examination shows that a retroperitoneal infection is affecting a purely endocrine gland. Which of the following structures is infected? (A) Ovary (B) Suprarenal gland (C) Pancreas (D) Liver (E) Stomach

14. The answer is B. The suprarenal gland is a retroperitoneal organ and is a purely endocrine gland. The pancreas is a retroperitoneal organ and contains endocrine cells, but it is not a purely endocrine gland. The liver and stomach contain endocrine cells, but they are not purely endocrine glands and also are surrounded by peritoneum. The ovary contains endocrine cells and is located in the pelvic cavity.

14. A 12-year-old boy walks in; he fell out of a tree and fractured the upper portion of his humerus. Which of the following nerves are intimately related to the humerus and are most likely to be injured by such a fracture? (A) Axillary and musculocutaneous (B) Radial and ulnar (C) Radial and axillary (D) Median and musculocutaneous (E) Median and ulnar

14. The answer is C. The axillary nerve passes posteriorly around the surgical neck of the humerus, and the radial nerve lies in the radial groove of the middle of the shaft of the humerus. The ulnar nerve passes behind the medial epicondyle, and the median nerve is vulnerable to injury by supracondylar fracture of the humerus, but these nerves lie close to or in contact with the lower portion of the humerus. The musculocutaneous nerve is not in direct contact with the humerus.

14. An 8-year-old boy with ASD presents to a pediatrician. This congenital heart defect shunts blood from the left atrium to the right atrium and causes hypertrophy of the right atrium, right ventricle, and pulmonary trunk. Which of the following veins opens into the hypertrophied atrium? (A) Middle cardiac vein (B) Small cardiac vein (C) Oblique cardiac vein (D) Anterior cardiac vein (E) Right pulmonary vein

14. The answer is D. The anterior cardiac vein drains into the right atrium. The middle, small, and oblique cardiac veins drain into the coronary sinus. The right and left pulmonary veins drain into the left atrium.

14. A 21-year-old man receives a penetrating knife wound in the abdomen and is injured in both the superior mesenteric artery and the vagus nerve. Which portion of the colon would most likely be impaired by this injury? (A) Ascending and descending colons (B) Transverse and sigmoid colons (C) Descending and sigmoid colons (D) Ascending and transverse colons (E) Transverse and descending colons

14. The answer is D. The ascending and transverse colons receive blood from the superior mesenteric artery and parasympathetic nerve fibers from the vagus nerve. However, the descending and sigmoid colons receive blood from the inferior mesenteric artery and the parasympathetic nerve fibers from the pelvic splanchnic nerve arising from sacral spinal nerves (S2-S4).

14. A young boy with a tooth abscess from a longstanding infection suffers damage of the lingual nerve as it enters the oral cavity. Which of the following structures contain cell bodies of injured nerve fibers? (A) Geniculate and otic ganglia (B) Trigeminal and submandibular ganglia (C) Trigeminal and dorsal root ganglia (D) Geniculate and trigeminal ganglia (E) Geniculate and pterygopalatine ganglia

14. The answer is D. The lingual nerve is joined by the chorda tympani in the infratemporal fossa. Therefore, the lingual nerve contains GSA fibers whose cell bodies are located in the trigeminal ganglion and SSA or taste fibers that have cell bodies located in the geniculate ganglion. In addition, the lingual nerve carries parasympathetic preganglionic GVE fibers that originated from the chorda tympani; the cell bodies are located in the superior salivatory nucleus in the pons. The chorda tympani and lingual nerves contain no fibers from the otic, submandibular, pterygopalatine, or dorsal root ganglia.

14. A 27-year-old paratrooper lands on a pine tree. Consequently, preganglionic parasympathetic nerves leaving the central nervous system are lacerated. Which of the following structures contain cell bodies of the damaged nerve fibers? (A) Cervical and sacral spinal cord (B) Cervical and thoracic spinal cord (C) Brain stem and cervical spinal cord (D) Thoracic and lumbar spinal cord (E) Brain stem and sacral spinal cord

14. The answer is E. Preganglionic neurons of the parasympathetic nervous system are located in the brain stem (cranial outflow) and sacral spinal cord segments S2 to S4 (sacral outflow). Preganglionic sympathetic neurons are located in the thoracic and lumbar spinal cord.

16. Which of the following nervous structures would most likely be affected by multiple sclerosis? (A) Trigeminal ganglion (B) Superior cervical ganglion (C) Optic nerve (D) Facial nerve (E) Spinal accessory nerve

16. The answer is C. Multiple sclerosis affects only axons in the CNS (spinal cord and brain) that have myelin sheaths formed by oligodendrocytes. The optic nerve is considered to be part of the CNS, as it is derived from an outpouching of the diencephalon. All other nervous structures are in the PNS and have their myelin sheaths formed by Schwan cells.

15. A patient experiences weakness in dorsiflexing and inverting the foot. Which of the following muscles is damaged? (A) Peroneus longus (B) Peroneus brevis (C) Tibialis anterior (D) Extensor digitorum longus (E) Peroneus tertius Questions 16 to 20: A 62-year-old woman slips and falls on the bathroom floor. As a result, she has a posterior dislocation of the hip joint and a fracture of the neck of the femur.

15. The Answer is C. The tibialis anterior can dorsiflex and invert the foot. The peroneus longus and brevis muscles can plantar flex and evert the foot, the peroneus tertius can dorsiflex and evert the foot, and the extensor digitorum longus can dorsiflex the foot and extend the toes.

15. A 37-year-old patient with severe chest pain, shortness of breath, and congestive heart failure was admitted to a local hospital. His coronary angiograms reveal a thrombosis in the circumflex branch of the left coronary artery. Which of the following conditions could result from the blockage of blood flow in the circumflex branch? (A) Tricuspid valve insufficiency (B) Mitral valve insufficiency (C) Ischemia of AV node (D) Paralysis of pectinate muscle (E) Necrosis of septomarginal trabecula

15. The answer is B. The circumflex branch of the left coronary artery supplies the left ventricle, and thus its blockage of blood flow results in necrosis of myocardium in the left ventricle, producing mitral valve insufficiency. The tricuspid valve, AV node, pectinate muscles, and septomarginal trabecula are present in the right atrium and ventricle. .

15. A 37-year-old man is suffering from carcinoma of the skin of the penis. Cancer cells are likely to metastasize directly to which of the following lymph nodes? (A) External iliac nodes (B) Internal iliac nodes (C) Superficial inguinal nodes (D) Aortic (lumbar) nodes (E) Deep inguinal nodes

15. The answer is C. The superficial inguinal nodes receive lymph from the penis, scrotum, buttocks, labium majus, and the lower parts of the vagina and anal canal. These nodes have efferent vessels that drain primarily into the external iliac and common iliac nodes and ultimately to the lumbar (aortic) nodes. The internal iliac nodes receive lymph from the upper part of the rectum, vagina, uterus, and other pelvic organs, and they drain into the common iliac nodes and then into the lumbar (aortic) nodes. Lymph vessels from the glans penis drain initially into the deep inguinal nodes and then into the external iliac nodes

15. A 42-year-old man with portal hypertension secondary to cirrhosis of the liver and subsequent massive ascites presents to the emergency department. He refuses to have a transjugular intrahepatic portosystemic shunt (TIPS) procedure and prefers surgery. Which of the following surgical connections is involved in the most practical method of shunting portal blood around the liver? (A) Superior mesenteric vein to the inferior mesenteric vein (B) Portal vein to the superior vena cava (C) Portal vein to the left renal vein (D) Splenic vein to the left renal vein (E) Superior rectal vein to the left colic vein

15. The answer is D. Portal hypertension can be reduced by diverting blood from the portal to the caval system. This is accomplished by connecting the splenic vein to the left renal vein or by creating a communication between the portal vein and the IVC. A connection between a hepatic vein and a branch of the portal vein can be accomplished by the transjugular intrahepatic portosystemic shunt (TIPS) procedure in the treatment of bleeding esophageal varices.

15. A 36-year-old woman received a firstdegree burn on her neck, arm, and forearm from a house fire. Which of the following skin structures or functions is most likely damaged or impaired? (A) GSE nerves (B) Parasympathetic general visceral efferent nerves (C) Trophic hormone production (D) Exocrine gland secretion (E) Vitamin A production

15. The answer is D. Skin has sweat glands and sebaceous glands, which are exocrine glands. Skin produces vitamin D, but it does not produce a trophic hormone and does not produce vitamin A. In addition, skin contains no GSE and parasympathetic GVE nerve fibers.

15. A man injures his wrist on broken glass. Which of the following structures entering the palm superficial to the flexor retinaculum may be damaged? (A) Ulnar nerve and median nerve (B) Median nerve and flexor digitorum profundus (C) Median nerve and flexor pollicis longus (D) Ulnar artery and ulnar nerve (E) Ulnar nerve and flexor digitorum superficialis

15. The answer is D. Structures entering the palm superficial to the flexor retinaculum include the ulnar nerve, ulnar artery, palmaris longus tendon, and palmar cutaneous branch of the median nerve. The median nerve, the flexor pollicis longus, and the flexor digitorum superficialis and profundus run deep to the flexor retinaculum.

15. A knife wound has severed the oculomotor nerve in a 45-year-old man. Which of the following conditions will occur because of this injury? (A) Constricted pupil (B) Abduction of the eyeball (C) Complete ptosis (D) Impaired lacrimal secretion (E) Paralysis of the ciliary muscle

15. The answer is E. The oculomotor nerve carries parasympathetic fibers to the ciliary and sphincter pupillae ciliary muscles; thus, a lesion of the oculomotor nerve leads to ciliary muscle paralysis and a dilated pupil. The abducens nerve supplies the lateral rectus, which is an abductor of the eye. The levator palpebrae superioris inserts on the tarsal plate in the upper eyelid, which is innervated by sympathetic fibers. Thus, a lesion of the oculomotor nerve does not cause complete ptosis. The secretomotor fibers for lacrimal secretion come through the pterygopalatine ganglion. Thus, severance of the oculomotor nerve has no effect on lacrimal secretion.

15. A 67-year-old woman comes to her physician complaining of visual loss. Her MRI scan shows an enlarged pituitary gland that lies in the sella turcica, immediately posterior and superior to which of the following structures? (A) Frontal sinus (B) Maxillary sinus (C) Ethmoid air cells (D) Mastoid air cells (E) Sphenoid sinus

15. The answer is E. The pituitary gland lies in the hypophyseal fossa of the sella turcica of the sphenoid bone, which lies immediately posterior and superior to the sphenoid sinus and medial to the cavernous sinus. The frontal sinus lies in the frontal bone; the maxillary sinus lies in the maxilla lateral to the lateral wall of the nasal cavity; the ethmoid sinus (composed of air cells) lies between the orbit and the nasal cavity; and the mastoid air cells lie in the mastoid process of the temporal bone.

16. A 20-year-old guard at the gate of the Royal King's palace blinks his eyes when a strong wind hits the cornea of his eye. The afferent fibers of the corneal reflex arc are carried by which of the following nerves? (A) Optic nerve (B) Lacrimal nerve (C) Nasociliary nerve (D) Zygomatic nerve (E) Oculomotor nerve

16. The answer is C. The afferent limb of the corneal reflex arc is the nasociliary nerve, and its efferent limb is the facial nerve. The other nerves are not involved in the reflex arc. The opening of the eye is conducted by the oculomotor nerve, but it is not a part of the corneal reflex.

16. A patient with Bennett fracture (a fracture of the base of the first metacarpal bone) experiences an impaired thumb movement. Which of the following intrinsic muscles of the thumb is most likely injured? (A) Abductor pollicis brevis (B) Flexor pollicis brevis (superficial head) (C) Opponens pollicis (D) Adductor pollicis (E) Flexor pollicis brevis (deep head)

16. The answer is C. The opponens pollicis inserts on the first metacarpal. All other intrinsic muscles of the thumb, including the abductor pollicis brevis, the flexor pollicis brevis, and the adductor pollicis muscles, insert on the proximal phalanges

16. Rupture of the ligamentum teres capitis femoris may lead to damage to a branch of which of the following arteries? (A) Medial circumflex femoral (B) Lateral circumflex femoral (C) Obturator (D) Superior gluteal (E) Inferior gluteal

16. The Answer is C. The obturator artery gives rise to an acetabular branch that runs in the round ligament of the head of the femur.

16. A crush injury of the vertebral column can cause the spinal cord to swell. Which structure would be trapped between the dura and the vertebral body by the swelling spinal cord? (A) Anterior longitudinal ligament (B) Alar ligament (C) Posterior longitudinal ligament (D) Cruciform ligament (E) Ligamentum nuchae

16. The Answer is C. The posterior longitudinal ligament interconnects the vertebral bodies and intervertebral disks posteriorly and runs anterior to the spinal cord within the vertebral canal. The ligamentum nuchae is formed by supraspinous ligaments that extend from the seventh cervical vertebra to the external occipital protuberance and crest. The anterior longitudinal ligament runs anterior to the vertebral bodies. The alar and cruciform ligaments also lie anterior to the spinal cord.

16. A 42-year-old woman who has had six children develops a weakness of the urogenital diaphragm. Paralysis of which of the following muscles would cause such a symptom? (A) Sphincter urethrae (B) Coccygeus (C) Superficial transversus perinei (D) Levator ani (E) Obturator internus

16. The answer is A. The urogenital diaphragm consists of the sphincter urethrae and deep transverse perineal muscles. Weakness of the muscles, ligaments, and fasciae of the pelvic floor, such as the pelvic diaphragm, urogenital diaphragm, and cardinal (transverse cervical) ligaments, occurs as a result of multiple child delivery, advancing age, and menopause. The pelvic diaphragm is composed of the levator ani and coccygeus muscles. The superficial transversus perinei is one of the superficial perineal muscles, and the obturator internus forms the lateral wall of the ischiorectal fossa.

16. After having a tonsillectomy, a 57-year-old man with a long history of chewing tobacco use is unable to detect taste on the posterior one-third of his tongue. Which of the following nerves has most likely been injured? (A) Internal laryngeal nerve (B) Lingual nerve (C) Glossopharyngeal nerve (D) Greater palatine nerve (E) Chorda tympani

16. The answer is C. The posterior one-third of the tongue receives both general and taste innervation from the lingual branch of the glossopharyngeal nerve. The internal laryngeal nerve supplies general and taste sensations to the epiglottis. The lingual nerve supplies general sensation to the anterior two-thirds of the tongue. The greater palatine nerve innervates the hard palate and the inner surface of the maxillary gingival. The chorda tympani supplies taste sensation to the anterior two-thirds of the tongue and preganglionic parasympathetic fibers to the submandibular ganglion for supplying the submandibular and sublingual glands.

16. A 78-year-old man is suffering from ischemia of the suprarenal glands. This condition results from rapid occlusion of direct branches of which of the following arteries? (A) Aorta, splenic, and inferior phrenic arteries (B) Renal, splenic, and inferior mesenteric arteries (C) Aorta, inferior phrenic, and renal arteries (D) Superior mesenteric, inferior mesenteric, and renal arteries (E) Aorta and hepatic and renal arteries

16. The answer is C. The suprarenal gland receives arteries from three sources. The superior suprarenal artery arises from the inferior phrenic artery, the middle suprarenal artery arises from the abdominal aorta, and the inferior suprarenal artery arises from the renal artery. The hepatic, superior mesenteric, inferior mesenteric, and splenic arteries do not supply the suprarenal gland.

16. A 75-year-old patient has been suffering from lung cancer located near the cardiac notch, a deep indentation on the lung. Which of the following lobes is most likely to be excised? (A) Superior lobe of the right lung (B) Middle lobe of the right lung (C) Inferior lobe of the right lung (D) Superior lobe of the left lung (E) Inferior lobe of the left lung

16. The answer is D. The cardiac notch is a deep indentation of the anterior border of the superior lobe of the left lung. Therefore, the right lung is not involved

17. Fracture of the neck of the femur results in avascular necrosis of the femoral head, probably resulting from lack of blood supply from which of the following arteries? (A) Obturator (B) Superior gluteal (C) Inferior gluteal (D) Medial femoral circumflex (E) Lateral femoral circumflex

17. The Answer is D. In adults, the chief arterial supply to the head of the femur is from the branches of the medial femoral circumflex artery. The lateral femoral circumflex artery may supply the femoral head by anastomosing with the medial femoral circumflex artery. The posterior branch of the obturator artery gives rise to the artery of the head of the femur, which runs in the round ligament of the femoral head and is usually insufficient to supply the head of the femur in adults but is an important source of blood to the femoral head in children. The superior and inferior gluteal arteries do not supply the head of the femur.

17. A 44-year-old woman comes to her physician and complains of headache and backache. On examination, she is found to have fluid accumulated in the spinal epidural space because of damage to blood vessels or meninges. Which of the following structures is most likely ruptured? (A) Vertebral artery (B) Vertebral vein (C) External vertebral venous plexus (D) Internal vertebral venous plexus (E) Lumbar cistern

17. The Answer is D. The internal vertebral venous plexus is located in the spinal epidural space. The vertebral artery and vein occupy the transverse foramina of the upper six cervical vertebrae. The external vertebral venous plexus consists of the anterior part, which lies in front of the vertebral column, and the posterior part, which lies on the vertebral arch. The lumbar cistern is the enlargement of the subarachnoid space between the inferior end of the spinal cord and the inferior end of the subarachnoid space.

17. A 43-year-old man has a benign tumor located near a gap between the arcuate pubic ligament and the transverse perineal ligament. Which of the following structures is most likely compressed by this tumor? (A) Perineal nerve (B) Deep dorsal vein of the penis (C) Superficial dorsal vein (D) Posterior scrotal nerve (E) Deep artery of the penis.

17. The answer is B. The deep dorsal vein, dorsal artery, and dorsal nerve of the penis pass through a gap between the arcuate pubic ligament and the transverse perineal ligament. The perineal nerve divides into a deep branch, which supplies all of the perineal muscles, and superficial branches as posterior scrotal nerves, which supply the scrotum. The superficial dorsal vein of the penis empties into the greater saphenous vein. The deep artery of the penis runs in the corpus cavernosum of the penis. .

17. A 27-year-old pianist with a known carpal tunnel syndrome experiences difficulty in finger movements. Which of the following intrinsic muscles of her hand is paralyzed? (A) Palmar interossei and adductor pollicis (B) Dorsal interossei and lateral two lumbricals (C) Lateral two lumbricals and opponens pollicis (D) Abductor pollicis brevis and palmar interossei (E) Medial two and lateral two lumbricals

17. The answer is C. The median nerve innervates the abductor pollicis brevis, opponens pollicis, and two lateral lumbricals. The ulnar nerve innervates all interossei (palmar and dorsal), the adductor pollicis, and the two medial lumbricals.

17. A thoracentesis is performed to aspirate an abnormal accumulation of fluid in a 37-yearold patient with pleural effusion. A needle should be inserted at the midaxillary line between which of the following two ribs so as to avoid puncturing the lung? (A) Ribs 1 and 3 (B) Ribs 3 and 5 (C) Ribs 5 and 7 (D) Ribs 7 and 9 (E) Ribs 9 and 11

17. The answer is D. A thoracentesis is performed for aspiration of fluid in the pleural cavity at or posterior to the midaxillary line, one or two intercostal spaces below the fluid level but not below the ninth intercostal space and, therefore, between ribs 7 and 9. Other intercostals spaces are not preferred.

17. A radiograph of a 32-year-old woman reveals a perforation in the posterior wall of the stomach in which the gastric contents have spilled into the lesser sac. The general surgeon has opened the lienogastric (gastrosplenic) ligament to reach the lesser sac and notes erosion of the ulcer into an artery. Which of the following vessels is most likely involved? (A) Splenic artery (B) Gastroduodenal artery (C) Left gastric artery (D) Right gastric artery (E) Left gastroepiploic artery

17. The answer is E. The left gastroepiploic artery runs through the lienogastric ligament, and hence, it is the artery most likely injured. The splenic artery is found in the lienorenal ligament. The right and left gastric arteries run within the lesser omentum. The gastroduodenal artery descends between the duodenum and the head of the pancreas.

17. A 71-year-old man suffers from a known benign tumor in the pterygoid canal. Which of the following nerve fibers could be injured by this condition? (A) Postganglionic parasympathetic fibers (B) Taste fibers from the epiglottis (C) General somatic afferent (GSA) fibers (D) Preganglionic sympathetic fibers (E) General visceral afferent (GVA) fibers

17. The answer is E. The nerve of the pterygoid canal (Vidian nerve) contains taste SVA fibers from the palate, GVA fibers, postganglionic sympathetic fibers, and preganglionic parasympathetic fibers

17. A 14-year-old boy hits his head on the asphalt road after falling off his skateboard. His radiograph reveals damage to the sella turcica. This is probably due to fracture of which of the following bones? (A) Frontal bone (B) Ethmoid bone (C) Temporal bone (D) Basioccipital bone (E) Sphenoid bone

17. The answer is E. The sella turcica is part of the sphenoid bone and lies superior to the sphenoid sinus. Therefore, none of the other bones listed is fractured.

18. If the acetabulum is fractured at its posterosuperior margin by dislocation of the hip joint, which of the following bones could be involved? (A) Pubis (B) Ischium (C) Ilium (D) Sacrum (E) Head of the femur

18. The Answer is C. The acetabulum is a cup-shaped cavity on the lateral side of the hip bone and is formed superiorly by the ilium, posteroinferiorly by the ischium, and anteromedially by the pubis. The sacrum and the head of the femur do not participate in the formation of the acetabulum.

18. A 69-year-old man has an abnormally increased curvature of the thoracic vertebral column. Which of the following conditions is the most likely diagnosis? (A) Lordosis (B) Spina bifida occulta (C) Meningocele (D) Meningomyelocele (E) Kyphosis

18. The Answer is E. Kyphosis (hunchback or humpback) is an abnormally increased thoracic curvature, usually resulting from osteoporosis. Lordosis is an abnormal accentuation of the lumbar curvature. Spina bifida occulta is failure of the vertebral arch to fuse (only bony defect). Meningocele is a protrusion of the meninges through the unfused arch of the vertebra, whereas meningomyelocele is a protrusion of the spinal cord and the meninges.

18. A newborn baby is readmitted to the hospital with hypoxia and upon testing is found to have pulmonary stenosis, dextraposition of the aorta, interventricular septal defect, and hypertrophy of the right ventricle. Which of the following is best described by these symptoms? (A) ASD (B) Patent ductus arteriosus (C) Tetralogy of Fallot (D) Aortic stenosis (E) Coarctation of the aorta

18. The answer is C. Tetralogy of Fallot is a combination of congenital cardiac defects consisting of (a) pulmonary stenosis, (b) dextraposition of the aorta (so that it overrides the ventricular septum and receives blood from the right ventricle), (c) ventricular septal defect (VSD), and (d) right ventricular hypertrophy. ASD is a congenital defect in the atrial septum, resulting from a patent foramen ovale. Patent ductus arteriosus shunts blood from the pulmonary trunk to the aorta, bypassing the lungs. Aortic stenosis is an abnormal narrowing of the aortic valve orifice, impeding the blood flow. Coarctation of the aorta is a congenital constriction of the aorta, commonly occurs just distal to the left subclavian artery, causing upper limb hypertension and diminished blood flow to the lower limbs and abdominal viscera.

18. A 35-year-old woman with a history of cholecystectomy arrives in the emergency department with intractable hiccups most likely caused by an abdominal abscess secondary to surgical infection. Which of the following nerves carries pain sensation caused by irritation of the peritoneum on the central portion of the inferior surface of the diaphragm? (A) Vagus nerve (B) Lower intercostal nerve (C) Phrenic nerve (D) Greater splanchnic nerve (E) Subcostal nerve

18. The answer is C. The diaphragm receives somatic motor fibers solely from the phrenic nerves. However, the peritoneum on the central part of the diaphragm receives sensory fibers from the phrenic nerve, and the peripheral part of the diaphragm receives such fibers from the lower intercostal nerves. The subcostal nerve supplies the peritoneum inferior to the diaphragm. The vagus and greater splanchnic nerves do not carry pain fibers from the peritoneum.

18. An obstetrician performs a median episiotomy on a woman before parturition to prevent uncontrolled tearing. If the perineal body is damaged, the function of which of the following muscles might be impaired? (A) Ischiocavernosus and sphincter urethrae (B) Deep transverse perineal and obturator internus (C) Bulbospongiosus and superficial transverse perineal (D) External anal sphincter and sphincter urethrae (E) Bulbospongiosus and ischiocavernosus

18. The answer is C. The perineal body (central tendon of the perineum) is a fibromuscular node at the center of the perineum. It provides attachment for the bulbospongiosus, the superficial and deep transverse perineal muscles, and the sphincter ani externus muscles. Other muscles (ischiocavernosus, sphincter urethrae, and obturator internus) are not attached to the perineal body

18. A 31-year-old roofer walks in with tenosynovitis resulting from a deep penetrated wound in the palm by a big nail. Examination indicates that he has an infection in the ulnar bursa. This infection most likely resulted in necrosis of which of the following tendons? (A) Tendon of the flexor carpi ulnaris (B) Tendon of the flexor pollicis longus (C) Tendon of the flexor digitorum profundus (D) Tendon of the flexor carpi radialis (E) Tendon of the palmaris longus

18. The answer is C. The ulnar bursa, or common synovial flexor sheath, contains the tendons of both the flexor digitorum superficialis and profundus muscles. The radial bursa envelops the tendon of the flexor pollicis longus. The tendons of the flexor carpi ulnaris and the palmaris longus are not contained in the ulnar bursa.

18. A 22-year-old patient has dryness of the corneal surface of his eye because of a lack of tears. Which of the following nerves may be damaged? (A) Proximal portion of the lacrimal nerve (B) Zygomatic branch of the facial nerve (C) Lesser petrosal nerve (D) Greater petrosal nerve (E) Deep petrosal nerve

18. The answer is D. The secretomotor fibers to the lacrimal gland are parasympathetic fibers that run in the facial, greater petrosal, Vidian (nerve of the pterygoid canal), maxillary, zygomatic (of maxillary), zygomaticotemporal, and lacrimal (terminal portion) nerves. The lesser petrosal nerve carries secretomotor (preganglionic parasympathetic) fibers to the parotid gland. The deep petrosal nerve contains postganglionic sympathetic fibers. The zygomatic branch of the facial nerve supplies the facial muscles.

18. The nerve accompanying the superior thyroid artery may be damaged during an operation on the thyroid gland. Which of the following functional defects may result from this injury? (A) Loss of sensation above the vocal cord (B) Loss of lateral rotation of the arytenoid cartilages (C) Paralysis of the vocalis muscle (D) Lack of abduction of the vocal cord (E) Decreased tension of the vocal cord

18. The answer is E. The superior thyroid artery is accompanied by the external laryngeal nerve, which innervates the cricothyroid muscle. Paralysis of this muscle due to a lesion of the external laryngeal nerve decreases tension of the vocal cord. Loss of sensation above the vocal cord is due to injury of the internal laryngeal nerve. The posterior cricoarytenoid muscle draws the muscular process of the arytenoid cartilage posteriorly and thereby rotates its vocal process laterally. Paralysis of the vocalis muscle is due to a lesion of the recurrent laryngeal nerve. Lack of abduction of the vocal cord results from paralysis of the posterior cricoarytenoid muscle.

19. During a snowstorm, a 52-year-old man is brought to the emergency department after a multiple car accident. Which of the following conditions is produced by a force that drives the trunk forward while the head lags behind in a rear-end automobile collision? (A) Scoliosis (B) Hangman fracture (C) Meningomyelocele (D) Whiplash injury (E) Herniated disk

19. The Answer is D. Whiplash injury of the neck is produced by a force that drives the trunk forward while the head lags behind. Scoliosis is a lateral deviation resulting from unequal growth of the spinal column. Hangman fracture is a fracture of the neural arch through the pedicle of the axis that may occur as a result of hanging or motor vehicle accidents. Meningomyelocele is a protrusion of the spinal cord and its meninges. A herniated disk compresses the spinal nerve roots when the nucleus pulposus is protruded through the annulus fibrosus.

19. The woman experiences weakness when abducting and medially rotating the thigh after this accident. Which of the following muscles is most likely damaged? (A) Piriformis (B) Obturator internus (C) Quadratus femoris (D) Gluteus maximus (E) Gluteus minimus

19. The Answer is E. The gluteus medius or minimus abducts and rotates the thigh medially. The piriformis, obturator internus, quadratus femoris, and gluteus maximus muscles can rotate the thigh laterally.

19. An 18-year-old boy involved in an automobile accident presents with an arm that cannot abduct. His paralysis is caused by damage to which of the following nerves? (A) Suprascapular and axillary (B) Thoracodorsal and upper subscapular (C) Axillary and musculocutaneous (D) Radial and lower subscapular (E) Suprascapular and dorsal scapular

19. The answer is A. The abductors of the arm are the deltoid and supraspinatus muscles, which are innervated by the axillary and suprascapular nerves, respectively. The thoracodorsal nerve supplies the latissimus dorsi, which can adduct, extend, and rotate the arm medially. The upper and lower subscapular nerves supply the subscapularis, and the lower subscapular nerve also supplies the teres major; both of these structures can adduct and rotate the arm medially. The musculocutaneous nerve supplies the flexors of the arm, and the radial nerve supplies the extensors of the arm. The dorsal scapular nerve supplies the levator scapulae and rhomboid muscles; these muscles elevate and adduct the scapula, respectively.

19. A 22-year-old man has a gonorrheal infection that has infiltrated the space between the inferior fascia of the urogenital diaphragm and the superficial perineal fascia. Which of the following structures might be inflamed? (A) Bulb of the penis (B) Bulbourethral gland (C) Membranous part of the male urethra (D) Deep transverse perineal muscle (E) Sphincter urethrae

19. The answer is A. The bulb of the penis is located in the superficial perineal space between the inferior fascia of the urogenital diaphragm and the membranous layer of the superficial perineal fascia (Colles fascia). All of the other structures are found in the deep perineal pouch.

19. A 37-year-old patient has an infectious inflammation of the dural venous sinus closest to the pituitary gland and a secondary thrombus formation. Which of the following is the most likely site of infection? (A) Straight sinus (B) Cavernous sinus (C) Superior petrosal sinus (D) Sigmoid sinus (E) Confluence of sinuses

19. The answer is B. The dural venous sinus nearest the pituitary gland is the cavernous sinus. Cavernous sinus thrombophlebitis is an infectious inflammation of the sinus that may produce meningitis, papilledema, exophthalmos, and ophthalmoplegia. The other sinuses listed are not closely associated with the pituitary gland.

19. A 16-year-old boy with a ruptured spleen comes to the emergency department for splenectomy. Soon after ligation of the splenic artery just distal to its origin, a surgical resident observes that the patient is healing normally. Normal blood flow would occur in which of the following arteries? (A) Short gastric arteries (B) Dorsal pancreatic artery (C) Inferior pancreaticoduodenal artery (D) Left gastroepiploic artery (E) Artery in the lienorenal ligament

19. The answer is C. The inferior pancreaticoduodenal artery is a branch of the superior mesenteric artery. All of other arteries are branches of the splenic artery

19. A 31-year-old hockey player is hit in the head by a puck. His radiogram shows a fracture of the foramen rotundum. Which of the following nerves would be damaged by this event? (A) Ophthalmic nerve (B) Mandibular nerve (C) Maxillary nerve (D) Optic nerve (E) Trochlear nerve

19. The answer is C. The maxillary nerve runs through the foramen rotundum; the ophthalmic nerve runs through the supraorbital fissure; the mandibular nerve passes through the foramen ovale; the optic nerve runs through the optic canal; and the trochlear nerve passes through the superior orbital fissure.

19. A 33-year-old patient is suffering from a sudden occlusion at the origin of the descending (thoracic) aorta. This condition would most likely decrease blood flow in which of the following intercostal arteries? (A) Upper six anterior (B) All of the posterior (C) Upper two posterior (D) Lower anterior (E) Lower six posterior

19. The answer is E. The first two posterior intercostal arteries are branches of the highest (superior) intercostal artery of the costocervical trunk; the remaining nine branches are from the thoracic aorta. The internal thoracic artery gives off the upper six anterior intercostal arteries and is divided into the superior epigastric and musculophrenic arteries, which gives off anterior intercostal arteries in the 7th, 8th, and 9th intercostal spaces and ends in the 10th intercostal space where it anastomoses with the deep circumflex iliac artery.

2. A patient with a deep knife wound in the buttock walks with a waddling gait that is characterized by the pelvis falling toward one side at each step. Which of the following nerves is damaged? (A) Obturator nerve (B) Nerve to obturator internus (C) Superior gluteal nerve (D) Inferior gluteal nerve (E) Femoral nerve

2. The Answer is C. The superior gluteal nerve innervates the gluteus medius muscle. Paralysis of this muscle causes gluteal gait, a waddling gait characterized by a falling of the pelvis toward the unaffected side at each step. The gluteus medius muscle normally functions to stabilize the pelvis when the opposite foot is off the ground. The inferior gluteal nerve innervates the gluteus maximus, and the nerve to the obturator internus supplies the obturator internus and superior gemellus muscles. The obturator nerve innervates the adductor muscles of the thigh, and the femoral nerve supplies the flexors of the thigh

2. A 16-year-old boy presents with double vision. Further examination reveals that he has difficulty in turning his eye inferolaterally and trouble going downstairs. Which of the following nerves is most likely damaged? (A) Oculomotor nerve (B) Optic nerve (C) Ophthalmic nerve (D) Trochlear nerve (E) Abducens nerve

2. The Answer is D. If the trochlear nerve is injured, the patient is unable to turn the eyeball inferolaterally and has trouble going downstairs due to paralysis of the superior oblique muscle. Lesion of the oculomotor nerve causes ptosis due to paralysis of the levator palpebrae superioris, dilation of the pupil due to paralysis of the sphincter pupillae, loss of accommodation due to paralysis of ciliary muscles, and loss of pupillary light reflex due to loss of the efferent limb of the pupillary light reflex. Lesion of the optic nerve causes blindness. Lesion of the ophthalmic nerve causes loss of cutaneous sensation on the face above the upper eyelid. Lesion of the abducens nerve causes internal strabismus in which the eyeball turns medially

2. A 23-year-old jockey falls from her horse and complains of headache, backache, and weakness. Radiologic examination would reveal blood in which of the following spaces if the internal vertebral venous plexus was ruptured? (A) Space deep to the pia mater (B) Space between the arachnoid and dura maters (C) Subdural space (D) Epidural space (E) Subarachnoid space

2. The Answer is D. The space between the vertebral canal and the dura mater is the epidural space, which contains the internal vertebral venous plexus. The spinal cord and blood vessels lie deep to the pia mater. The space between the arachnoid and dura maters is the subdural space, which contains a film of fluid. The subarachnoid space contains cerebrospinal fluid (CSF).

2. A 43-year-old female patient has been lying down on the hospital bed for more than 4 months. Her normal, quiet expiration is achieved by contraction of which of the following structures? (A) Elastic tissue in the lungs and thoracic wall (B) Serratus posterior superior muscles (C) Pectoralis minor muscles (D) Serratus anterior muscles (E) Diaphragm

2. The answer is A. Normal, quiet expiration is achieved by contraction of extensible tissue in the lungs and the thoracic wall. The serratus posterior superior muscles, diaphragm, pectoralis major, and serratus anterior are muscles of inspiration.

2. A 17-year-old boy suffers a traumatic groin injury during a soccer match. The urologist notices tenderness and swelling of the boy's left testicle that may be produced by thrombosis in which of the following veins? (A) Left internal pudendal vein (B) Left renal vein (C) Inferior vena cava (D) Left inferior epigastric vein (E) Left external pudendal vein

2. The answer is B. A tender swollen left testis may be produced by thrombosis in the left renal vein because the left testicular vein drains into the left renal vein. The right testicular vein drains into the inferior vena cava. The left internal pudendal vein empties into the left internal iliac vein. The left inferior epigastric vein drains into the left external iliac vein, and the left external pudendal vein empties into the femoral vein.

2. A 27-year-old woman with a goiter comes to the hospital for surgical treatment. The surgeon must ligate the superior laryngeal artery before surgically resecting the goiter, so care must be taken to avoid injury to which of the following nerves? (A) External laryngeal nerve (B) Internal laryngeal nerve (C) Superior laryngeal nerve (D) Hypoglossal nerve (E) Vagus nerve

2. The answer is B. The internal laryngeal nerve accompanies the superior laryngeal artery, whereas the external laryngeal nerve accompanies the superior thyroid artery. The superior laryngeal, hypoglossal, and vagus nerves are not closely associated with the superior laryngeal artery.

2. A 36-year-old woman with yellow pigmentation of the skin and sclerae presents at the outpatient clinic. Which of the following conditions most likely is the cause of her obstructive jaundice? (A) Aneurysm of the splenic artery (B) Perforated ulcer of the stomach (C) Obstruction of the main pancreatic duct (D) Cancer in the head of the pancreas (E) Cancer in the body of the pancreas

2. The answer is D. Because the bile duct traverses the head of the pancreas, cancer in the head of the pancreas obstructs the bile duct, resulting in jaundice. Aneurysm of the splenic artery, obstruction of the main pancreatic duct, a stomach ulcer, and cancer in the body of the pancreas are not closely associated with the bile duct. The tail of the pancreas is located at the hilus of the spleen, which lies far from the bile duct.

2. On the basis of the examination at her doctor's office, a patient is told that her parasympathetic nerves are damaged. Which of the following muscles would most likely be affected? (A) Muscles in the hair follicles (B) Muscles in blood vessels (C) Muscles that act at the elbow joint (D) Muscles in the gastrointestinal (GI) tract (E) Muscles enclosed by epimysium

2. The answer is D. Smooth muscles in the gastrointestinal tract are innervated by both parasympathetic and sympathetic nerves. Smooth muscles in the wall of the blood vessels and arrector pili muscles in hair follicles are innervated only by sympathetic nerves. Muscles that act at the elbow joint and muscles enclosed by epimysium are skeletal muscles that are innervated by somatic motor (general somatic efferent [GSE]) nerves

2. A patient comes in with a gunshot wound and requires surgery in which his thoracoacromial trunk needs to be ligated. Which of the following arterial branches would maintain normal blood flow? (A) Acromial (B) Pectoral (C) Clavicular (D) Deltoid (E) Superior thoracic

2. The answer is E. The superior thoracic artery is a direct branch of the axillary artery. The thoracoacromial trunk has four branches: the pectoral, clavicular, acromial, and deltoid.

20. The woman undergoes hip surgery. If all of the arteries that are part of the cruciate anastomosis of the upper thigh are ligated, which of the following arteries maintains blood flow? (A) Medial femoral circumflex (B) Lateral femoral circumflex (C) Superior gluteal (D) Inferior gluteal (E) First perforating

20. The Answer is C. The superior gluteal artery does not participate in the cruciate anastomosis of the thigh. The inferior gluteal artery, transverse branches of the medial and lateral femoral circumflex arteries, and an ascending branch of the first perforating artery form the cruciate anastomosis of the thigh.

20. A 37-year-old man is brought to the emergency department with a crushed second cervical vertebra (axis) that he suffered after a stack of pallets fell on him at work. Which of the following structures would be intact after the accident? (A) Alar ligament (B) Apical ligament (C) Semispinalis cervicis muscle (D) Rectus capitis posterior minor (E) Obliquus capitis inferior

20. The Answer is D. The rectus capitis posterior minor arises from the posterior tubercle of the atlas and inserts on the occipital bone below the inferior nuchal line. The alar ligament extends from the apex of the dens to the medial side of the occipital bone. The apical ligament extends from the dens of the axis to the anterior aspect of the foramen magnum of the occipital bone. The semispinalis cervicis arises from the transverse processes and inserts on the spinous processes. The obliquus capitis inferior originates from the spine of the axis and inserts on the transverse process of the atlas.

20. Muscles derived from the second (hyoid) pharyngeal arch are innervated by which of the following cranial nerves? (A) Trigeminal nerve (B) Facial nerve (C) Glossopharyngeal nerve (D) Vagus nerve (E) Accessory nerve

20. The answer is B. Muscles derived from the second (hyoid) pharyngeal arch are innervated by the facial nerve. Muscles derived from the first (mandibular) pharyngeal arch are innervated by the mandibular division of the trigeminal nerve. A muscle derived from the third pharyngeal arch is innervated by the glossopharyngeal nerve. Muscles derived from the fourth and sixth pharyngeal arches are innervated by the vagus nerve. Muscles of myotome origin that shrug shoulder and turn head are innervated by the accessory nerve.

20. A 56-year-old patient recently suffered a myocardial infarction in the area of the apex of the heart. The occlusion by atherosclerosis is in which of the following arteries? (A) Marginal artery (B) Right coronary artery at its origin (C) Anterior interventricular artery (D) Posterior interventricular artery (E) Circumflex branch of the left coronary artery

20. The answer is C. The apex of the heart typically receives blood from the anterior interventricular branch of the left coronary artery. The marginal artery supplies the right inferior margin of the right ventricle, the right coronary artery at its origin supplies the right atrium and ventricle, and the posterior interventricular artery and a circumflex branch of the left coronary artery supply the left ventricle.

20. A 9-year-old boy was admitted to the emergency department complaining of nausea, vomiting, fever, and loss of appetite. On examination, he was found to have tenderness and pain on the right lower quadrant. Based on signs and symptoms, the diagnosis of acute appendicitis was made. During an appendectomy performed at McBurney point, which of the following structures is most likely to be injured? (A) Deep circumflex femoral artery (B) Inferior epigastric artery (C) Iliohypogastric nerve (D) Genitofemoral nerve (E) Spermatic cord

20. The answer is C. The iliohypogastric nerve runs medially and inferiorly between the internal oblique and transverse abdominal muscles near the McBurney point, the point at the junction of the lateral one-third of the line between the anterior superior iliac spine and the umbilicus. Other structures are not found near the McBurney point

20. A 39-year-old man is unable to expel the last drops of urine from the urethra at the end of micturition because of paralysis of the external urethral sphincter and bulbospongiosus muscles. This condition may occur as a result of injury to which of the following nervous structures? (A) Pelvic plexus (B) Prostatic plexus (C) Pudendal nerve (D) Pelvic splanchnic nerve (E) Sacral splanchnic nerve

20. The answer is C. The perineal branch of the pudendal nerve supplies the external urethral sphincter and bulbospongiosus muscles in the male. All other nervous structures do not supply skeletal muscles but supply smooth muscles in the perineal and pelvic organs. The pelvic and prostatic plexuses contain both sympathetic and parasympathetic nerve fibers. The pelvic splanchnic nerve carries preganglionic parasympathetic fibers, whereas the sacral splanchnic nerve transmits preganglionic sympathetic fibers

20. A 17-year-old boy with a stab wound received multiple injuries on the upper part of the arm and required surgery. If the brachial artery were ligated at its origin, which of the following arteries would supply blood to the profunda brachii artery? (A) Lateral thoracic (B) Subscapular (C) Posterior humeral circumflex (D) Superior ulnar collateral (E) Radial recurrent

20. The answer is C. The posterior humeral circumflex artery anastomoses with an ascending branch of the profunda brachii artery, whereas the lateral thoracic and subscapular arteries do not. The superior ulnar collateral and radial recurrent arteries arise inferior to the origin of the profunda brachii artery

20. A young singer at the local music theater visits her physician and complains of vocal difficulties. On examination, she is unable to abduct the vocal cords during quiet breathing. Which of the following muscles is most likely paralyzed? (A) Vocalis muscle (B) Cricothyroid muscle (C) Oblique arytenoid muscle (D) Posterior cricoarytenoid muscle (E) Thyroarytenoid muscle

20. The answer is D. The posterior cricoarytenoid muscle is the only muscle that abducts the vocal cords during quiet breathing. All of the other laryngeal muscles adduct the vocal cords.

22. A 27-year-old baseball player is hit on his forearm by a high-speed ball during the World Series, and the muscles that form the floor of the cubital fossa appear to be torn. Which of the following groups of muscles have lost their functions? (A) Brachioradialis and supinator (B) Brachialis and supinator (C) Pronator teres and supinator (D) Supinator and pronator quadratus (E) Brachialis and pronator teres

22. The answer is B. The brachialis and supinator muscles form the floor of the cubital fossa. The brachioradialis and pronator teres muscles form the lateral and medial boundaries, respectively. The pronator quadratus is attached to the distal ends of the radius and the ulna.

21. A 34-year-old woman sustains a deep cut on the dorsum of the foot just distal to her ankle joint by a falling kitchen knife. A physician in the emergency department has ligated the dorsalis pedis artery proximal to the injured area. Which of the following conditions most likely occurs as a result of the injury? (A) Ischemia in the peroneus longus muscle (B) Aneurysm in the plantar arterial arch (C) Reduction of blood flow in the medial tarsal artery (D) Low blood pressure in the anterior tibial artery (E) High blood pressure in the arcuate artery

21. The Answer is C. Reduction of blood flow in the medial tarsal artery occurs because it is a branch of the dorsalis pedis artery, which begins at the ankle joint as the continuation of the anterior tibial artery. The anterior tibial and peroneal arteries supply the peroneus longus muscle. The deep plantar arterial arch is formed mainly by the lateral plantar artery. Blood pressure in the anterior tibial artery should be higher than normal. The arcuate artery should have a low blood pressure because it is a terminal branch of the dorsalis pedis artery.

21. A middle-aged coal miner injures his back after an accidental explosion. His magnetic resonance imaging (MRI) scan reveals that his spinal cord has shifted to the right because the lateral extensions of the pia mater were torn. Function of which of the following structures is most likely impaired? (A) Filum terminale internum (B) Coccygeal ligament (C) Denticulate ligament (D) Choroid plexus (E) Tectorial membrane

21. The Answer is C. The denticulate ligament is a lateral extension of the pia mater. The filum terminale (internum) is an inferior extension of the pia mater from the tip of the conus medullaris. The coccygeal ligament, which is also called the filum terminale externum or the filum of the dura, extends from the tip of the dural sac to the coccyx. The vascular choroid plexuses produce the cerebrospinal fluid (CSF) in the ventricles of the brain. The tectorial membrane is an upward extension of the posterior longitudinal ligaments from the body of the axis to the basilar part of the occipital bone.

21. A 21-year-old marine biologist asks about her first bimanual examination, and it is explained to her that the normal position of the uterus is (A) Anteflexed and anteverted (B) Retroflexed and anteverted (C) Anteflexed and retroverted (D) Retroverted and retroflexed (E) Anteverted and retroverted

21. The answer is A. The normal position of the uterus is anteverted (i.e., angle of 90 degrees at the junction of the vagina and cervical canal) and anteflexed (i.e., angle of 160 to 170 degrees at the junction of the cervix and body).

21. A 71-year-old woman often visits an emergency department with swallowing difficulties and subsequent choking while eating food. Which of the following pairs of muscles is most instrumental in preventing food from entering the larynx and trachea during swallowing? (A) Sternohyoid and sternothyroid muscles (B) Oblique arytenoid and aryepiglottic muscles (C) Inferior pharyngeal constrictor and thyrohyoid muscles (D) Levator veli palatini and tensor veli palatini muscles (E) Musculus uvulae and geniohyoid muscles

21. The answer is B. The oblique arytenoid and aryepiglottic muscles tilt the arytenoid cartilages and approximate them, assisting in closing of the larynx and preventing food from entering the larynx and trachea during the process of swallowing. The cricopharyngeus fibers of the inferior pharyngeal constrictors act as a sphincter that prevents air from entering the esophagus. Other muscles are not involved in closing or opening the airway.

21. A 54-year-old man with a long history of alcohol abuse presents to the emergency department with rapidly increasing abdominal distention most likely resulting from an alteration in portal systemic blood flow. Which of the following characteristics is associated with the portal vein or the portal venous system? (A) Lower blood pressure than in the IVC (B) Least risk of venous varices because of portal hypertension (C) Distention of the portal vein resulting from its numerous valves (D) Caput medusae and hemorrhoids caused by portal hypertension (E) Less blood flow than in the hepatic artery

21. The answer is D. Portal hypertension can cause esophageal varices, caput medusa, and hemorrhoids. The portal vein has higher pressure than systemic veins; the vein and its tributaries have no valves, or, if present, they are insignificant. In addition, the portal vein carries two to three times as much blood as the hepatic artery.

21. A 75-year-old woman was admitted to a local hospital, and bronchograms and radiographs revealed a lung carcinoma in her left lung. Which of the following structures or characteristics does the cancerous lung contain? (A) Horizontal fissure (B) Groove for superior vena cava (SVC) (C) Middle lobe (D) Lingula (E) Larger capacity than the right

21. The answer is D. The lingula is the tongue-shaped portion of the upper lobe of the left lung. The right lung has a groove for the horizontal fissure, superior vena cava (SVC), and middle lobe and has a larger capacity than the left lung.

21. A 23-year-old woman who receives a deep cut to her ring finger by a kitchen knife is unable to move the metacarpophalangeal joint. Which of the following pairs of nerves was damaged? (A) Median and ulnar (B) Radial and median (C) Musculocutaneous and ulnar (D) Ulnar and radial (E) Radial and axillary

21. The answer is D. The metacarpophalangeal joint of the ring finger is flexed by the lumbrical, palmar, and dorsal interosseous muscles, which are innervated by the ulnar nerve. The extensor digitorum, which is innervated by the radial nerve, extends this joint. The musculocutaneous and axillary nerves do not supply muscles of the hand. The median nerve supplies the lateral two lumbricals, which can flex metacarpophalangeal joints of the index and middle fingers.

22. A patient experiences paralysis of the muscle that originates from the femur and contributes directly to the stability of the knee joint. Which of the following muscles is involved? (A) Vastus lateralis (B) Semimembranosus (C) Sartorius (D) Biceps femoris (long head) (E) Rectus femoris

22. The Answer is A. The vastus lateralis muscles arise from the femur and all the other muscles originate from the hip (coxal) bone. The biceps femoris inserts on the fibula, and other muscles insert on the tibia; thus, all of them contribute to the stability of the knee joint.

22. A 25-year-old man with congenital abnormalities at birth has a lesion of the dorsal scapular nerve, making him unable to adduct his scapula. Which of the following muscles is most likely paralyzed? (A) Semispinalis capitis (B) Rhomboid major (C) Multifidus (D) Rotator longus (E) Iliocostalis

22. The Answer is B. The rhomboid major is a superficial muscle of the back; is innervated by the dorsal scapular nerve, which arises from the ventral primary ramus of the fifth cervical nerve; and adducts the scapula. The semispinalis capitis, multifidus, rotator longus, and iliocostalis muscles are deep muscles of the back, are innervated by dorsal primary rami of the spinal nerves, and have no attachment to the scapula.

22. After his bath but before getting dressed, a 4-year-old boy was playing with his puppy. The boy's penis was bitten by the puppy, and the deep dorsal vein was injured. The damaged vein (A) Lies superficial to Buck fascia (B) Drains into the prostatic venous plexus (C) Lies lateral to the dorsal artery of the penis (D) Is found in the corpus spongiosum (E) Is dilated during erection

22. The answer is B. The deep dorsal vein of the penis lies medial to the dorsal artery of the penis on the dorsum of the penis and deep to Buck fascia, drains into the prostatic plexus of veins, and is compressed against the underlying deep fascia of the penis during erection.

22. While examining radiographs and angiograms of a 52-year-old patient, a physician is trying to distinguish the jejunum from the ileum. He has observed that the jejunum has: (A) Fewer plicae circulares (B) Fewer mesenteric arterial arcades (C) Less digestion and absorption of nutrients (D) Shorter vasa recta (E) More fat in its mesentery

22. The answer is B. The jejunum has fewer mesenteric arterial arcades but longer vasa recta than the ileum. The plicae circulares (circular folds) are tall and closely packed in the jejunum and are low and sparse in the ileum, and the lower part of the ileum has no plicae circulares. More digestion and absorption of nutrients occurs in the jejunum than in the ileum, and less fat is found in the mesentery of the jejunum.

22. A 31-year-old woman complains of headache and dizziness after hitting a kitchen cabinet door with her head. Her MRI scan and venogram show a large blood clot in the great cerebral vein of Galen. The obstructed vein of the brain is a direct tributary of which of the following venous structures? (A) Emissary veins (B) Pterygoid venous plexus (C) Diploic veins (D) Dural venous sinuses (E) Internal jugular vein

22. The answer is D. The veins of the brain are direct tributaries of the dural venous sinuses. The emissary veins connect the dural venous sinuses with the veins of the scalp; the pterygoid venous plexus communicates with the cavernous sinus through an emissary vein; the diploic veins lie in channels in the diploë of the skull and communicate with the dural sinuses, the veins of the scalp, and the meningeal veins.

22. An 18-year-old girl is thrust into the steering wheel while driving and experiences difficulty in expiration. Which of the following muscles is most likely damaged? (A) Levator costarum (B) Innermost intercostal muscle (C) External intercostal muscle (D) Diaphragm (E) Muscles of the abdominal wall

22. The answer is E. The abdominal muscles are the major muscles of expiration, whereas the other distractors are muscles of inspiration.

23. After an automobile accident, a back muscle that forms the boundaries of the triangle of auscultation and the lumbar triangle receives no blood. Which of the following muscles might be ischemic? (A) Levator scapulae (B) Rhomboid minor (C) Latissimus dorsi (D) Trapezius (E) Splenius capitis

23. The Answer is C. The latissimus dorsi forms boundaries of the auscultation and lumbar triangles and receives blood from the thoracodorsal artery. The levator scapulae, rhomboid minor, and splenius capitis muscles do not form boundaries of these two triangles. The trapezius muscle forms a boundary of the auscultation triangle but not the lumbar triangle. The levator scapulae, rhomboid minor, and trapezius muscles receive blood from the transverse cervical artery. The splenius capitis muscle receives blood from the occipital and transverse cervical arteries.

23. A patient is involved in a motorcycle wreck that results in avulsion of the skin over the anterolateral leg and ankle. Which of the following structures is most likely destroyed with this type of injury? (A) Deep peroneal nerve (B) Extensor digitorum longus muscle tendon (C) Dorsalis pedis artery (D) Great saphenous vein (E) Superficial peroneal nerve

23. The Answer is E. The superficial peroneal nerve emerges between the peroneus longus and peroneus brevis muscles and descends superficial to the extensor retinaculum of the ankle on the anterolateral side of the leg and ankle, innervating the skin of the lower leg and foot. The great saphenous vein begins at the medial end of the dorsal venous arch of the foot and ascends in front of the medial malleolus and along the medial side of the tibia along with the saphenous nerve. Other structures pass deep to the extensor retinaculum

23. A 67-year-old woman with a long history of liver cirrhosis was seen in the emergency department. In this patient with portal hypertension, which of the following veins is most likely to be dilated? (A) Right colic vein (B) Inferior epigastric vein (C) Inferior phrenic vein (D) Suprarenal vein (E) Ovarian vein

23. The answer is A. The right colic vein belongs to the portal venous system and empties into the superior mesenteric vein, which joins the splenic vein to form the portal vein. The inferior epigastric, inferior phrenic, suprarenal, and ovarian veins belong to the systemic (or caval) venous system and drain directly or indirectly into the IVC.

24. A 26-year-old patient is admitted to a local hospital with a retroperitoneal infection. Which of the following arteries is most likely to be infected? (A) Left gastric artery (B) Proper hepatic artery (C) Middle colic artery (D) Sigmoid arteries (E) Dorsal pancreatic artery

24. The answer is E. The pancreas is a retroperitoneal organ, except for a small portion of its tail. The dorsal pancreatic artery would be the infected artery because it arises from the splenic artery and runs retroperitoneally along the superior border of the pancreas behind the peritoneum. The other arteries run within layers of the peritoneum. The left gastric arteries run within the lesser omentum; the proper hepatic artery runs within the free margin of the lesser omentum; the middle colic artery runs within the transverse mesocolon; the sigmoid arteries run within the sigmoid mesocolon.

23. A 23-year-old man complains of numbness on the medial side of the arm following a stab wound in the axilla. On examination, he is diagnosed with an injury of his medial brachial cutaneous nerve. In which of the following structures are the cell bodies of the damaged nerve involved in numbness located? (A) Sympathetic chain ganglion (B) Dorsal root ganglion (C) Anterior horn of the spinal cord (D) Lateral horn of the spinal cord (E) Posterior horn of the spinal cord

23. The answer is B. The medial brachial cutaneous nerve contains sensory (general somatic afferent [GSA]) fibers that have cell bodies in the dorsal root ganglia, and an injury of these GSA fibers causes numbness of the medial side of the arm. It also contains sympathetic postganglionic fibers that have cell bodies in the sympathetic chain ganglia. The anterior horn of the spinal cord contains cell bodies of skeletal motor (general somatic efferent [GSE]) fibers, and the lateral horn contains cell bodies of sympathetic preganglionic fibers. The posterior horn contains cell bodies of interneurons.

23. A 41-year-old woman overdoses on some prescription medications that have a common side effect of autonomic nerve stimulation. Which of the following conditions or actions results from stimulation of the parasympathetic fibers to the eyeball? (A) Enhanced vision for distant objects (B) Dilation of the pupil (C) Contraction of capillaries in the iris (D) Contraction of the ciliary muscle (E) Flattening of the lens

23. The answer is D. When the parasympathetic fibers to the eyeball are stimulated, the pupil constricts and the ciliary muscle contracts, resulting in a thicker lens and enhanced vision for near objects (accommodation). Dilation of the pupil, contraction of capillaries in the iris, and enhanced ability to see distant objects (flattening of the lens) result from stimulation of sympathetic nerves.

23. A 78-year-old patient presents with an advanced cancer in the posterior mediastinum. The surgeons are in a dilemma as to how to manage the condition. Which of the following structures is most likely damaged? (A) Brachiocephalic veins (B) Trachea (C) Arch of the azygos vein (D) Arch of the aorta (E) Hemiazygos vein

23. The answer is E. The hemiazygos vein is located in the posterior mediastinum. The brachiocephalic veins, trachea, and arch of the aorta are located in the superior mediastinum, whereas the arch of the azygos vein is found in the middle mediastinum.

23. A 62-year-old man is incapable of penile erection after rectal surgery with prostatectomy. The patient most likely has a lesion of which of the following nerves? (A) Dorsal nerve of the penis (B) Perineal nerve (C) Hypogastric nerve (D) Sacral splanchnic nerve (E) Pelvic splanchnic nerve

23. The answer is E. The pelvic splanchnic nerve contains preganglionic parasympathetic fibers, whereas the sacral splanchnic nerve contains preganglionic sympathetic fibers. Parasympathetic fibers are responsible for erection, whereas sympathetic fibers are involved with ejaculation. The right and left hypogastric nerves contain primarily sympathetic fibers and visceral sensory fibers. The dorsal nerve of the penis and the perineal nerve provide sensory nerve fibers.

24. A 38-year-old woman with a long history of shoulder pain is admitted to a hospital for surgery. Which of the following muscles becomes ischemic soon after ligation of the superficial or ascending branch of the transverse cervical artery? (A) Latissimus dorsi (B) Multifidus (C) Trapezius (D) Rhomboid major (E) Longissimus capitis

24. The Answer is C. The trapezius receives blood from the superficial branch of the transverse cervical artery. The latissimus dorsi receives blood from the thoracodorsal artery. The rhomboid major receives blood from the deep or descending branch of the transverse cervical artery. The multifidus and longissimus capitis receive blood from the segmental arteries.

24. A knife wound penetrates the superficial vein that terminates in the popliteal vein. Bleeding occurs from which of the following vessels? (A) Posterior tibial vein (B) Anterior tibial vein (C) Peroneal vein (D) Great saphenous vein (E) Lesser saphenous vein

24. The Answer is E. The lesser (small) saphenous vein ascends on the back of the leg in company with the sural nerve and terminates in the popliteal vein. The peroneal vein empties into the posterior tibial vein. The anterior and posterior tibial veins are deep veins and join to form the popliteal vein. The great saphenous vein drains into the femoral vein.

24. A 46-year-old patient comes to his doctor's office and complains of chest pain and headache. His computed tomography (CT) scan reveals a tumor located just superior to the root of the right lung. Blood flow in which of the following veins is most likely blocked by this tumor? (A) Hemiazygos vein (B) Arch of the azygos vein (C) Right subclavian vein (D) Right brachiocephalic vein (E) Accessory hemiazygos vein

24. The answer is B. The azygos vein arches over the root of the right lung and empties into the SVC. Other veins do not pass over the root of the right lung. .

24. A 23-year-old massage therapist who specializes in women's health attends a lecture at an annual conference on techniques of massage. She asks, "What structure is drained by the lumbar (aortic) lymph nodes?" Which of the following structures is the correct answer to this question? (A) Perineum (B) Lower part of the vagina (C) External genitalia (D) Ovary (E) Lower part of the anterior abdominal wall

24. The answer is D. The lymphatic vessels from the ovary ascend with the ovarian vessels in the suspensory ligament and terminate in the lumbar (aortic) nodes. Lymphatic vessels from the perineum, external genitalia, and lower part of the anterior abdominal wall drain into the superficial inguinal nodes.

24. A 38-year-old homebuilder was involved in an accident and is unable to supinate his forearm. Which of the following nerves are most likely damaged? (A) Suprascapular and axillary (B) Musculocutaneous and median (C) Axillary and radial (D) Radial and musculocutaneous (E) Median and ulnar

24. The answer is D. The supinator and biceps brachii muscles, which are innervated by the radial and musculocutaneous nerves, respectively, produce supination of the forearm. This is a question of two muscles that can supinate the forearm.

24. A 53-year-old woman with a severe middle ear infection comes to the hospital. On examination, the physician finds that the infection has injured the tympanic nerve. The damaged nerve: (A) Is a branch of the facial nerve (B) Contains postganglionic parasympathetic fibers (C) Synapses with fibers in the lesser petrosal nerve (D) Is a branch of the glossopharyngeal nerve (E) Forms the tympanic plexus in the external auditory meatus

24. The answer is D. The tympanic nerve, or Jacobson nerve, is a branch of the glossopharyngeal nerve, contains preganglionic parasympathetic fibers, and forms a tympanic plexus on the medial wall of the middle ear with sympathetic fibers. The tympanic nerve continues beyond the plexus as the lesser petrosal nerve, which transmits preganglionic parasympathetic fibers to the otic ganglion for synapse.

25. A 10-year-old boy falls from a tree house. The resultant heavy compression of the sole of his foot against the ground caused a fracture of the head of the talus. Which of the following structures is unable to function normally? (A) Transverse arch (B) Medial longitudinal arch (C) Lateral longitudinal arch (D) Tendon of the peroneus longus (E) Long plantar ligament

25. The Answer is B. The keystone of the medial longitudinal arch of the foot is the head of the talus, which is located at the summit between the sustentaculum tali and the navicular bone. The medial longitudinal arch is supported by the spring ligament and the tendon of the flexor hallucis longus muscle. The cuboid bone serves as the keystone of the lateral longitudinal arch, which is supported by the peroneus longus tendon and the long and short plantar ligaments. The transverse arch is formed by the navicular, the three cuneiform, the cuboid, and the five metatarsal bones and is supported by the peroneus longus tendon and the transverse head of the adductor hallucis.

25. A 25-year-old soldier suffers a gunshot wound on the lower part of his back and is unable to move his legs. A neurologic examination and magnetic resonance imaging (MRI) scan reveal injury of the cauda equina. Which of the following is most likely damaged? (A) Dorsal primary rami (B) Ventral primary rami (C) Dorsal roots of the thoracic spinal nerves (D) Ventral roots of the sacral spinal nerves (E) Lumbar spinal nerves

25. The Answer is D. The cauda equina is the collection of dorsal and ventral roots of the lower lumbar and sacral spinal nerves below the spinal cord. Dorsal and ventral primary rami and dorsal roots of the thoracic spinal nerves and lumbar spinal nerves do not participate in the formation of the cauda equina.

25. A pediatric surgeon has resected a structure that is a fibrous remnant of an embryonic or fetal artery in a 5-year-old child. Which of the following structures is most likely to be divided? (A) Lateral umbilical fold (B) Medial umbilical fold (C) Median umbilical fold (D) Ligamentum teres hepatis (E) Ligamentum venosum

25. The answer is B. The medial umbilical fold or ligament contains a fibrous remnant of the umbilical artery. The median umbilical fold contains a fibrous remnant of the urachus. The lateral umbilical fold (ligament) contains the inferior epigastric artery and vein, which are adult blood vessels. The ligamentum venosum contains a fibrous remnant of the ductus venosus, and the ligamentum teres hepatic contains a fibrous remnant of the left umbilical vein

25. A 13-year-old boy competing in a motocross competition falls from his bike and sustains massive head injuries. Which of the following cavities are separated from the middle cranial fossa by a thin layer of bone? (A) Auditory tube and bony orbit (B) Middle ear cavity and sphenoid sinus (C) Sigmoid sinus and frontal sinus (D) Sphenoid sinus and ethmoid sinus (E) Maxillary sinus and middle ear cavity

25. The answer is B. The middle ear cavity is separated from the middle cranial fossa by the tegmen tympani, a thin plate of the petrous part of the temporal bone. A part of the roof of the sphenoid bone forms the floor of the hypophyseal fossa. The other pairs of sinuses or bony cavities are not separated from the middle cranial cavity.

25. A 21-year-old patient with a stab wound reveals a laceration of the right vagus nerve proximal to the origin of the recurrent laryngeal nerve. Which of the following conditions would most likely result from this lesion? (A) Contraction of bronchial muscle (B) Stimulation of bronchial gland secretion (C) Dilation of the bronchial lumen (D) Decrease in cardiac rate (E) Constriction of coronary artery

25. The answer is C. The parasympathetic nerve fibers in the vagus nerve constrict the bronchial lumen, contract bronchial smooth muscle, stimulate bronchial gland secretion, decrease heart rate, and constrict the coronary artery. The vagus nerve also carries afferent fibers of pain, cough reflex, and stretch of the lung (during inspiration)

25. A sexually active adolescent presents with an infection within the ischiorectal fossa. Which of the following structures is most likely injured? (A) Vestibular bulb (B) Seminal vesicle (C) Greater vestibular gland (D) Inferior rectal nerve (E) Internal pudendal artery

25. The answer is D. The ischiorectal fossa contains the inferior rectal nerves and vessels and adipose tissue. The bulb of the vestibule and the great vestibular gland are located in the superficial perineal space, whereas the bulbourethral gland is found in the deep perineal space. The internal pudendal artery runs in the pudendal canal, but its branches pass through the superficial and deep perineal spaces.

25. A 31-year-old patient complains of sensory loss over the anterior and posterior surfaces of the medial third of the hand and the medial one and one-half fingers. He is diagnosed by a physician as having "funny bone" symptoms. Which of the following nerves is injured? (A) Axillary (B) Radial (C) Median (D) Ulnar (E) Musculocutaneous

25. The answer is D. The ulnar nerve supplies sensory fibers to the skin over the palmar and dorsal surfaces of the medial third of the hand and the medial one and one-half fingers. The median nerve innervates the skin of the lateral side of the palm; the palmar side of the lateral three and one-half fingers; and the dorsal side of the index finger, the middle finger, and one-half of the ring finger. The radial nerve innervates the skin of the radial side of the hand and the radial two and one-half digits over the proximal phalanx.

26. A 24-year-old woman complains of weakness when she extends her thigh and rotates it laterally. Which of the following muscles is paralyzed? (A) Obturator externus (B) Sartorius (C) Tensor fasciae latae (D) Gluteus maximus (E) Semitendinosus

26. The Answer is D. The gluteus maximus can extend and rotate the thigh laterally. The obturator externus rotates the thigh laterally. The sartorius can flex both the hip and knee joints. The tensor fasciae latae can flex and medially rotate the thigh. The semitendinosus can extend the thigh and medially rotate the leg.

26. A neonate appears severely cyanotic and breathing rapidly. Cardiac echocardiogram reveals that the aorta lies to the right of the pulmonary trunk. Which of the following is most likely occurred during development? (A) AP septum failed to develop in a spiral fashion (B) Excessive resorption of septum primum (C) Pulmonary valve atresia (D) Persistent truncus arteriosus (E) Coarctation of the aorta

26. The answer is A. Failure of the aorticopulmonary septum results in transposition of the great vessels, exhibiting that the aorta is to the right of the pulmonary trunk. Cyanosis is common in transposition of the great vessels. Excessive resorption of septum primum results in a secundum type of ASD. Pulmonary valve atresia may result in cyanosis, but it will not cause the aorta to be to the right of the pulmonary trunk. A persistent truncus arteriosus is caused by lack of development of the aorticopulmonary septum resulting in a single outflow track. Coarctation of the aorta is a severe narrowing of the aorta

26. A first-year resident in the urology department reviews pelvic anatomy before seeing patients. Which of the following statements is correct? (A) The dorsal artery of the penis supplies the glans penis. (B) The seminal vesicles store spermatozoa. (C) The duct of the bulbourethral gland opens into the membranous urethra. (D) The duct of the greater vestibular gland opens into the vagina. (E) The anterior lobe of the prostate gland is prone to carcinomatous transformation.

26. The answer is A. The dorsal artery of the penis supplies the glans penis. The seminal vesicles store no spermatozoa. The duct of the bulbourethral gland opens into the bulbous portion of the spongy urethra, whereas the greater vestibular gland opens into the vestibule between the labium minora and the hymen. The anterior lobe of the prostate is devoid of glandular substance, the middle lobe is prone to benign hypertrophy, and the posterior lobe is prone to carcinomatous transformation.

26. A patient with a deep stab wound in the middle of the forearm has impaired movement of the thumb. Examination indicates a lesion of the anterior interosseous nerve. Which of the following muscles is paralyzed? (A) Flexor pollicis longus and brevis (B) Flexor pollicis longus and opponens pollicis (C) Flexor digitorum profundus and pronator quadratus (D) Flexor digitorum profundus and superficialis (E) Flexor pollicis brevis and pronator quadratus

26. The answer is C. The anterior interosseous nerve is a branch of the median nerve and supplies the flexor pollicis longus, half of the flexor digitorum profundus, and the pronator quadratus. The median nerve supplies the pronator teres, flexor digitorum superficialis, palmaris longus, and flexor carpi radialis muscles. A muscular branch (the recurrent branch) of the median nerve innervates the thenar muscles.

26. A 32-year-old house painter suffers from a head injury after falling off a ladder and has bleeding in his head. During intraoperative testing, the neurosurgeon notes loss of general sensation in the dura of the middle cranial fossa. Which of the following nerves has been affected? (A) Vagus nerve (B) Facial nerve (C) Hypoglossal nerve (D) Trigeminal nerve (E) Glossopharyngeal nerve

26. The answer is D. The cranial dura in the middle cranial fossa is innervated by the maxillary and mandibular divisions of the trigeminal nerve, the dura in the anterior cranial fossa is innervated by the ophthalmic division of the trigeminal nerve, and the dura in the posterior cranial fossa is innervated by the vagus and hypoglossal (C1 through the hypoglossal) nerves. The facial and glossopharyngeal nerves do not supply the cranial dura.

26. A 57-year-old patient has a tumor in the body of the pancreas that obstructs the inferior mesenteric vein just before joining the splenic vein. Which of the following veins is most likely to be enlarged? (A) Middle colic vein (B) Left gastroepiploic vein (C) Inferior pancreaticoduodenal vein (D) Ileocolic vein (E) Left colic vein

26. The answer is E. The left colic vein is a tributary of the inferior mesenteric vein. The middle colic, inferior pancreaticoduodenal, and ileocolic veins drain into the superior mesenteric vein. The left gastroepiploic vein empties into the splenic vein.

27. A patient with hereditary blood clotting problems presents with pain in the back of her knee. An arteriograph reveals a blood clot in the popliteal artery at its proximal end. Which of the following arteries will allow blood to reach the foot? (A) Anterior tibial (B) Posterior tibial (C) Peroneal (D) Lateral circumflex femoral (E) Superior medial genicular

27. The Answer is D. If the proximal end of the popliteal artery is blocked, blood may reach the foot by way of the descending branch of the lateral circumflex femoral artery, which participates in the anastomosis around the knee joint. Other blood vessels are direct or indirect branches of the popliteal artery.

27. A 43-year-old woman presents with a prolapsed uterus. Repair of a prolapsed uterus requires knowledge of the supporting structures of the uterus. Which of the following structures plays the most important role in the support of the uterus? (A) Levator ani (B) Sphincter urethrae (C) Uterosacral ligament (D) Ovarian ligament (E) Arcuate pubic ligament

27. The answer is A. The pelvic diaphragm, particularly the levator ani, provides the most important support for the uterus, although the urogenital diaphragm and the uterosacral and ovarian ligaments support the uterus. The arcuate pubic ligament arches across the inferior aspect of the pubic symphysis

27. A 29-year-old patient comes in; he cannot flex the distal interphalangeal (DIP) joint of the index finger. His physician determines that he has nerve damage from a supracondylar fracture. Which of the following conditions is also a symptom of this nerve damage? (A) Inability to flex the DIP joint of the ring finger (B) Atrophy of the hypothenar eminence (C) Loss of sensation over the distal part of the second digit (D) Paralysis of all the thumb muscles (E) Loss of supination

27. The answer is C. The flexor digitorum profundus muscle flexes the DIP joints of the index and middle fingers and is innervated by the median nerve, which also supplies sensation over the distal part of the second digit. The same muscle flexes the DIP joints of the ring and little fingers but receives innervation from the ulnar nerve, which also innervates the hypothenar muscles. The median nerve innervates the thenar muscles. The radial nerve innervates the supinator, abductor pollicis longus, and extensor pollicis longus and brevis muscles. The ulnar nerve innervates the adductor pollicis. The musculocutaneous nerve supplies the biceps brachii that can supinate the arm.

27. During a carotid endarterectomy of a 57-year-old man who suffered a stroke, the carotid sinus is damaged. A third-year medical student in surgical rotation notices that the injured structure: (A) Is located at the origin of the external carotid artery (B) Is innervated by the facial nerve (C) Functions as a chemoreceptor (D) Is stimulated by changes in blood pressure (E) Communicates freely with the cavernous sinus

27. The answer is D. The carotid sinus, a spindle-shaped dilatation of the origin of the internal carotid artery, is a pressoreceptor that is stimulated by changes in blood pressure. The carotid sinus is at the origin of the internal carotid artery and is innervated by the carotid sinus branch of the glossopharyngeal nerve and nerve to the carotid body of the vagus nerve. It is not a venous sinus and thus does not communicate with the cavernous sinus. The carotid body functions as a chemoreceptor.

27. An elderly man with prostatic hypertrophy returns to his urologist with another case of epididymitis. An acute infection involving the dartos muscle layer of the scrotum most likely leads to an enlargement of which of the following lymph nodes? (A) Preaortic nodes (B) Lumbar nodes (C) External iliac nodes (D) Superficial inguinal nodes (E) Common iliac nodes

27. The answer is D. The superficial inguinal lymph nodes receive lymph from the scrotum, penis, buttocks, and lower part of the anal canal, and their efferent vessels enter primarily to the external iliac nodes and ultimately to the lumbar (aortic) nodes. The deep inguinal nodes receive lymph from the testis and upper parts of the vagina and anal canal, and their efferent vessels enter the external iliac nodes.

27. A 12-year-old boy was admitted to a local hospital with a known history of heart problems. His left ventricular hypertrophy could result from which of the following conditions? (A) A constricted pulmonary trunk (B) An abnormally small left AV opening (C) Improper closing of the pulmonary valves (D) An abnormally large right AV opening (E) Stenosis of the aorta

27. The answer is E. Stenosis of the aorta can cause left ventricular hypertrophy. Right ventricular hypertrophy may occur as a result of pulmonary stenosis, pulmonary and tricuspid valve defects, or mitral valve stenosis.

28. A 72-year-old woman complains of a cramplike pain in her thigh and leg. She was diagnosed as having a severe intermittent claudication. Following surgery, an infection was found in the adductor canal, damaging the enclosed structures. Which of the following structures remains intact? (A) Femoral artery (B) Femoral vein (C) Saphenous nerve (D) Great saphenous vein (E) Nerve to the vastus medialis

28. The Answer is D. The great saphenous nerve remains intact because it is not in the adductor canal. The adductor canal contains the femoral vessels, the saphenous nerve, and the nerve to the vastus medialis.

28. A 16-year-old boy presents to the emergency department with rupture of the penile urethra. Extravasated urine from this injury can spread into which of the following structures? (A) Scrotum (B) Ischiorectal fossa (C) Pelvic cavity (D) Testis (E) Thigh

28. The answer is A. Extravasated urine from the penile urethra below the perineal membrane spreads into the superficial perineal space, scrotum, penis, and anterior abdominal wall. However, it does not spread into the testis, ischiorectal fossa, pelvic cavity, and thigh because Scarpa fascia ends by firm attachment to the fascia lata of the thigh.

28. During a game, a 26-year-old baseball player is hit in the head by a baseball, which fractures the optic canal. Which of the following pairs of structures is most likely to be damaged? (A) Optic nerve and ophthalmic vein (B) Ophthalmic vein and ophthalmic nerve (C) Ophthalmic artery and optic nerve (D) Ophthalmic nerve and optic nerve (E) Ophthalmic artery and ophthalmic vein

28. The answer is C. The optic canal transmits the optic nerve and ophthalmic artery. The ophthalmic nerve and ophthalmic vein enter the orbit through the superior orbital fissure.

28. A 27-year-old man with cubital tunnel syndrome complains of numbness and tingling in the ring and little finger and back and sides of his hand because of damage to a nerve in the tunnel at the elbow. Which of the following muscles is most likely to be paralyzed? (A) Flexor digitorum superficialis (B) Opponens pollicis (C) Two medial lumbricals (D) Pronator teres (E) Supinator

28. The answer is C. The ulnar nerve innervates the two medial lumbricals. However, the median nerve innervates the two lateral lumbricals, the flexor digitorum superficialis, the opponens pollicis, and the pronator teres muscles.

28. A patient with cirrhosis is scheduled for liver transplant surgery. During the operation rounds, the transplant physician explains to his residents that one of the reasons a surgeon must pay close attention to the anatomic location of the liver is that this organ: (A) Receives blood only from the hepatic arteries (B) Manufactures red blood cells in an adult (C) Drains bile from the quadrate lobe into the right hepatic duct (D) Drains venous blood into the hepatic veins (E) Functions to concentrate and store bile

28. The answer is D. The liver receives blood from the hepatic artery and portal vein and drains its venous blood into the hepatic veins. The liver manufactures red blood cells in the fetus. The liver plays important roles in bile production and secretion. The quadrate lobe drains bile into the left hepatic duct, not the right hepatic duct, whereas the caudate lobe drains bile into the right and left hepatic ducts. The gallbladder functions to concentrate and store bile.

28. A 31-year-old man was involved in a severe automobile accident and suffered laceration of the left primary bronchus. The damaged primary bronchus: (A) Has a larger diameter than the right primary bronchus (B) Often receives more foreign bodies than the right primary bronchus (C) Gives rise to the eparterial bronchus (D) Is longer than the right primary bronchus (E) Runs under the arch of the azygos vein

28. The answer is D. The right primary bronchus is shorter than the left one and has a larger diameter. More foreign bodies enter it via the trachea because it is more vertical than the left primary bronchus. The right primary bronchus runs under the arch of the azygos vein and gives rise to the eparterial bronchus.

29. A basketball player was hit in the thigh by an opponent's knee. Which of the following arteries is likely to compress and cause ischemia because of the bruise and damage to the extensor muscles of the leg? (A) Popliteal (B) Deep femoral (C) Anterior tibial (D) Posterior tibial (E) Peroneal

29. The Answer is C. A muscular spasm or hypertrophy of the extensor muscles of the leg may compress the anterior tibial artery, causing ischemia. The popliteal artery supplies muscles of the popliteal fossa. The deep femoral artery supplies deep muscles of the thigh. The posterior tibial and peroneal arteries supply muscles of the posterior and lateral compartments of the leg.

29. A 41-year-old woman is brought to the emergency department by her family because of acute onset of right upper quadrant pain, nausea, and vomiting. For this case, it is important to remember that the bile duct: (A) Drains bile into the second part of the duodenum (B) Can be blocked by cancer in the body of the pancreas (C) Joins the main pancreatic duct, which carries hormones (D) Is formed by union of the right and left hepatic duct (E) Lies posterior to the portal vein in the right free edge of the lesser omentum

29. The answer is A. The bile duct is formed by union of the common hepatic and cystic ducts, lies lateral to the proper hepatic artery and anterior to the portal vein in the right free margin of the lesser omentum, traverses the head of the pancreas, and drains bile into the second part of the duodenum at the greater papilla. The endocrine part of the pancreas secretes the hormones insulin and glucagon, which are transported through the bloodstream. The main pancreatic duct carries pancreatic juice containing enzymes secreted from the exocrine part of the pancreas.

29. A 62-year-old woman who is a heavy smoker has an advanced lung cancer that spread into her right third posterior intercostal space posterior to the midaxillary line. If cancer cells are carried in the venous drainage, they would travel first to which of the following veins? (A) SVC (B) Right superior intercostal vein (C) Right brachiocephalic vein (D) Azygos vein (E) Hemiazygos vein

29. The answer is B. The superior intercostal vein is formed by the union of the second, third, and fourth posterior intercostal veins and drains into the azygos vein on the right and the brachiocephalic vein on the left. The azygos vein drains into the SVC. The hemiazygos vein usually drains into the azygos vein.

29. A secretary comes in to your office complaining of pain in her wrists from typing all day. You determine that she likely has carpal tunnel syndrome. Which of the following conditions would help you determine the diagnosis? (A) Inability to adduct the little finger (B) Inability to flex the DIP joint of the ring finger (C) Flattened thenar eminence (D) Loss of skin sensation of the medial one and one-half fingers (E) Atrophied adductor pollicis muscle

29. The answer is C. The carpal tunnel contains the median nerve and the tendons of flexor pollicis longus, flexor digitorum profundus, and flexor digitorum superficialis muscles. Carpal tunnel syndrome results from injury to the median nerve, which supplies the thenar muscle. Thus, injury to this nerve causes the flattened thenar eminence. The middle finger has no attachment for the adductors. The ulnar nerve innervates the medial half of the flexor digitorum profundus muscle, which allows flexion of the DIP joints of the ring and little fingers. The ulnar nerve supplies the skin over the medial one and one-half fingers and adductor pollicis muscle.

29. A 43-year-old man has new onset of difficulty with speaking. Examination by the ENT resident reveals problems in elevating the hyoid bone and floor of the mouth, secondary to paralysis of the posterior belly of the digastric muscle. Which of the following nerves is most likely involved? (A) Accessory nerve (B) Trigeminal nerve (C) Ansa cervicalis (D) Facial nerve (E) Glossopharyngeal nerve

29. The answer is D. The digastric posterior belly is innervated by the facial nerve, whereas the digastric anterior belly is innervated by the trigeminal nerve. The accessory nerve supplies the sternocleidomastoid and trapezius muscles. The ansa cervicalis innervates the infrahyoid (or strap) muscles. The glossopharyngeal nerve supplies the stylopharyngeus muscle.

29. A 23-year-old woman visits her obstetrician for an annual checkup. During vaginal examination, which of the following structures may be palpated? (A) Apex of the urinary bladder (B) Fundus of the uterus (C) Terminal part of the round ligament of the uterus (D) Body of the clitoris (E) Uterine cervix

29. The answer is E. In addition to the uterine cervix, the uterus, uterine tubes, ovaries, and ureters can be palpated. The apex of the urinary bladder is the anterior end of the bladder; thus, it cannot be palpated. The fundus of the uterus is the anterosuperior part of the uterus. The terminal part of the round ligament of the uterus emerges from the superficial inguinal ring and becomes lost in the subcutaneous tissue of the labium majus.

3. A 31-year-old man with a penetrating injury to the posterior triangle of the neck is unable to shrug his shoulder and turn the head to the opposite side. Which of the following nerves is most likely damaged? (A) Trigeminal nerve (B) Facial nerve (C) Glossopharyngeal nerve (D) Accessory nerve (E) Hypoglossal nerve

3. The Answer is D. Accessory nerve passes through the posterior cervical triangle and is responsible for shrugging the shoulder and turn the head to the opposite side. The trigeminal nerve carries sensory fibers for the face and motor fibers for the muscles of mastication. The facial nerve carries motor fibers to the muscles of facial expression, secretomotor fibers to lacrimal, submandibular, sublingual, and nasal glands, and taste fibers from the anterior two-thirds of the tongue. The glossopharyngeal nerve conveys motor fibers to the stylopharyngeus muscle and taste fibers from the posterior one-third of the tongue. The hypoglossal nerve carries motor fibers for the muscles of tongue movement.

3. A 42-year-old woman with metastatic breast cancer is known to have tumors in the intervertebral foramina between the fourth and fifth cervical vertebrae and between the fourth and fifth thoracic vertebrae. Which of the following spinal nerves may be damaged? (A) Fourth cervical and fourth thoracic nerves (B) Fifth cervical and fifth thoracic nerves (C) Fourth cervical and fifth thoracic nerves (D) Fifth cervical and fourth thoracic nerves (E) Third cervical and fourth thoracic nerves

3. The Answer is D. All cervical spinal nerves exit through the intervertebral foramina above the corresponding vertebrae, except the eighth cervical nerves, which run inferior to the seventh cervical vertebra. All other spinal nerves exit the intervertebral foramina below the corresponding vertebrae. Therefore, the fifth cervical nerve passes between the fourth and fifth cervical vertebrae, and the fourth thoracic nerve runs between the fourth and fifth thoracic vertebrae.

3. A patient is unable to prevent anterior displacement of the femur on the tibia when the knee is flexed. Which of the following ligaments is most likely damaged? (A) Anterior cruciate (B) Fibular collateral (C) Patellar (D) Posterior cruciate (E) Tibial collateral

3. The Answer is D. The posterior cruciate ligament is important because it prevents forward displacement of the femur on the tibia when the knee is flexed. The anterior cruciate ligament prevents backward displacement of the femur on the tibia.

3. A 2-year-old boy presents with pain in his groin that has been increasing in nature over the past few weeks. He is found to have a degenerative malformation of the transversalis fascia during development. Which of the following structures on the anterior abdominal wall is likely defective? (A) Superficial inguinal ring (B) Deep inguinal ring (C) Inguinal ligament (D) Sac of a direct inguinal hernia (E) Anterior wall of the inguinal canal

3. The answer is B. The deep inguinal ring lies in the transversalis fascia, just lateral to the inferior epigastric vessels. The superficial inguinal ring is in the aponeurosis of the external oblique muscle. The inguinal ligament and the anterior wall of the inguinal canal are formed by the aponeurosis of the external oblique muscle. The sac of a direct inguinal hernia is formed by the peritoneum.

3. A 46-year-old male patient with high blood pressure was examined in the emergency department, and his physician found a leakage of blood from the blood vessel that normally carries richly oxygenated blood. Which of the following vessels would most likely be damaged? (A) Superior vena cava (B) Pulmonary arteries (C) Pulmonary veins (D) Portal vein (E) Coronary sinus

3. The answer is C. Pulmonary veins return oxygenated blood to the heart from the lungs. Pulmonary arteries carry deoxygenated blood from the heart to the lungs for oxygen renewal. The portal vein carries deoxygenated blood with nutrients from the intestine to the liver. The superior vena cava and coronary sinus carry deoxygenated blood to the right atrium.

3. A 23-year-old man received a gunshot wound, and his greater splanchnic nerve was destroyed. Which of the following nerve fibers would be injured? (A) General somatic afferent (GSA) and preganglionic sympathetic fibers (B) General visceral afferent (GVA) and postganglionic sympathetic fibers (C) GVA and preganglionic sympathetic fibers (D) General somatic efferent (GSE) and postganglionic sympathetic fibers (E) GVA and GSE fibers

3. The answer is C. The greater splanchnic nerves contain general visceral afferent (GVA) and preganglionic sympathetic general visceral efferent (GVE) fibers

3. On a busy Saturday night in Chicago, a 16-year-old boy presents to the emergency department with a stab wound from a knife that entered the pelvis above the piriformis muscle. Which of the following structures is most likely to be damaged? (A) Sciatic nerve (B) Internal pudendal artery (C) Superior gluteal nerve (D) Inferior gluteal artery (E) Posterior femoral cutaneous nerve

3. The answer is C. The superior gluteal nerve leaves the pelvis through the greater sciatic foramen, above the piriformis. The sciatic nerve, internal pudendal vessels, inferior gluteal vessels and nerve, and posterior femoral cutaneous nerve leave the pelvis below the piriformis.

3. A 19-year-old woman complains of numbness of the nasopharynx after surgical removal of the adenoid. A lesion of which of the following nerves would be expected? (A) Maxillary nerve (B) Superior cervical ganglion (C) External laryngeal nerve (D) Glossopharyngeal nerve (E) Vagus nerve

3. The answer is D. The glossopharyngeal nerve supplies sensory innervation to the mucosa of the upper pharynx, whereas the vagus nerve supplies sensory innervation to the lower pharynx and larynx. The maxillary nerve supplies sensory innervation to the face below the level of the eye and above the level of the upper lip and the palate and nasal mucosa. The superior cervical ganglion contributes to a formation of the pharyngeal plexus but contains no afferent fibers. The external laryngeal nerve innervates the cricothyroid and inferior pharyngeal constrictor muscles.

3. A 29-year-old man comes in with a stab wound, cannot raise his arm above horizontal, and exhibits a condition known as "winged scapula." Which of the following structures of the brachial plexus would most likely be damaged? (A) Medial cord (B) Posterior cord (C) Lower trunk (D) Roots (E) Upper trunk

3. The answer is D. Winged scapula is caused by paralysis of the serratus anterior muscle that results from damage to the long thoracic nerve, which arises from the roots of the brachial plexus (C5-C7).

Questions 31 to 35: A 20-year-old college student receives a severe blow on the inferolateral side of the left knee joint while playing football. Radiographic examination reveals a fracture of the head and neck of the fibula. 31. Which of the following nerves is damaged? (A) Sciatic (B) Tibial (C) Common peroneal (D) Deep peroneal (E) Superficial peroneal 32. After injury to this nerve, which of the following muscles could be paralyzed? (A) Gastrocnemius (B) Popliteus (C) Extensor hallucis longus (D) Flexor digitorum longus (E) Tibialis posterior

31. The Answer is C. The common peroneal nerve is vulnerable to injury as it passes behind the head of the fibula and then winds around the neck of the fibula and pierces the peroneus longus muscle, where it divides into the deep and superficial peroneal nerves. In addition, the deep and superficial peroneal nerves pass superficial to the neck of the fibula in the substance of the peroneus longus muscle and are less susceptible to injury than the common peroneal nerve. Other nerves are not closely associated with the head and neck of the fibula. 32. The Answer is C. The extensor hallucis longus is innervated by the deep peroneal nerve, whereas other muscles are innervated by the posterior tibial nerve

31. A 72-year-old man comes to his physician for an annual checkup. Which of the following structures is most readily palpated during rectal examination? (A) Prostate gland (B) Epididymis (C) Ejaculatory duct (D) Ureter (E) Testis

31. The answer is A. The prostate gland may be palpated on rectal examination. The ejaculatory duct runs within the prostate gland and cannot be felt. In the male, the pelvic part of the ureter lies lateral to the ductus deferens and enters the posterosuperior angle of the bladder, where it is situated anterior to the upper end of the seminal vesicle, and thus, it cannot be palpated during rectal examination. However, in the female, the ureter can be palpated during vaginal examination because it runs near the uterine cervix and the lateral fornix of the vagina to enter the posterosuperior angle of the bladder. The testes are examined during a routine annual checkup but obviously not during a rectal examination

31. The pupil in the eye of a 43-year-old patient remains small even when room lighting is dim. Which of the following nerves would be injured? (A) Trochlear nerve (B) Superior cervical ganglion (C) Oculomotor nerve (D) Ophthalmic nerve (E) Abducens nerve

31. The answer is B. The superior cervical ganglion is damaged. When the pupil remains small in a dimly lit room, it is an indication that postganglionic sympathetic fibers that originate from the superior cervical ganglion and innervate the dilator pupillae (radial muscles of the iris) are damaged. Other nerves contain no sympathetic fibers, but the oculomotor nerve contains preganglionic parasympathetic fibers.

31. A 19-year-old man with a ruptured appendix is sent to the emergency department for surgery. To cut off the blood supply to the appendix (if collateral circulation is discounted), a surgeon should ligate which of the following arteries? (A) Middle colic artery (B) Right colic artery (C) Ileocolic artery (D) Inferior mesenteric artery (E) Common iliac artery

31. The answer is C. The appendicular artery is a branch of the ileocolic artery. The other arteries do not supply the appendix. The middle colic and right colic arteries are branches of the superior mesenteric artery. The inferior mesenteric artery passes to the left behind the peritoneum and distributes to the descending and sigmoid colons and the upper portion of the rectum. The common iliac arteries are bifurcations from the aorta.

31. A 37-year-old man is brought to the emergency department complaining of severe chest pain. His angiogram reveals thromboses of both brachiocephalic veins just before entering the superior vena cava. This condition would most likely cause a dilation of which of the following veins? (A) Azygos (B) Hemiazygos (C) Right superior intercostal (D) Left superior intercostal (E) Internal thoracic

31. The answer is D. The left superior intercostal vein is formed by the second, third, and fourth posterior intercostal veins and drains into the left brachiocephalic vein. The right superior intercostal vein drains into the azygos vein, which in turn drains into the SVC. The hemiazygos vein drains into the azygos vein, whereas the internal thoracic vein empties into the brachiocephalic vein.

31. The victim of an automobile accident has a destructive injury of the proximal row of carpal bones. Which of the following bones is most likely damaged? (A) Capitate (B) Hamate (C) Trapezium (D) Triquetrum (E) Trapezoid

31. The answer is D. The proximal row of carpal bones consists of the scaphoid, lunate, triquetrum, and pisiform bones, whereas the distal row consists of the trapezium, trapezoid, capitate, and hamate bones.

32. A 48-year-old college football coach undergoes a radical prostatectomy for a malignant tumor in his prostate. Following surgery, he is incapable of achieving an erection. Which of the following nerves is most likely damaged during the surgery? (A) Sacral splanchnic nerve (B) Pelvic splanchnic nerve (C) Pudendal nerve (D) Dorsal nerve of the penis (E) Posterior scrotal nerve

32. The answer is B. Parasympathetic preganglionic fibers in the pelvic splanchnic nerve are responsible for erection of the penis. Sympathetic preganglionic fibers in the sacral splanchnic nerve are responsible for ejaculation. The pudendal nerve supplies the external anal sphincter and perineal muscles and supplies GSA fibers to the perineal region. The dorsal nerve of the penis is a terminal branch of the pudendal nerve and supplies sensation of the penis. The posterior scrotal nerves are superficial branches of the perineal nerve and supply sensory fibers to the scrotum.

32. A pharyngeal (gag) reflex is the contraction of the pharyngeal constrictor muscles that is elicited by touching the back of a patient's pharynx (e.g., with a tongue depressor). Afferent nerve fibers that innervated the pharyngeal mucosa are branches of which of the following nerves? (A) Trigeminal nerve (B) Facial nerve (C) Glossopharyngeal nerve (D) Vagus nerve (E) Hypoglossal nerve

32. The answer is C. The afferent limb of the pharyngeal (gag) reflex is a pharyngeal branch of the glossopharyngeal nerve, whereas the vagus nerve mediates the efferent limb. The trigeminal, facial, and hypoglossal nerves are not involved in the gag reflex.

32. Because of an inflammatory bowel disease (Crohn disease) and a small bowel obstruction leading to bowel ischemia, an elderly woman requires bypass of her ileum and jejunum and is scheduled for a gastrocolostomy. The surgeon will ligate all arteries that send branches to the stomach. Which of the following arteries may be spared? (A) Splenic artery (B) Gastroduodenal artery (C) Inferior pancreaticoduodenal artery (D) Left gastroepiploic artery (E) Proper hepatic artery

32. The answer is C. The inferior pancreaticoduodenal artery does not supply the stomach. All of the other arteries supply the stomach. Gastrocolostomy is used to establish a communication between the stomach and colon, bypassing the small intestine when the patient has Crohn disease (inflammation disease) and small bowel obstruction

32. A patient has a torn rotator cuff of the shoulder joint as the result of an automobile accident. Which of the following muscle tendons is intact and has normal function? (A) Supraspinatus (B) Subscapularis (C) Teres major (D) Teres minor (E) Infraspinatus

32. The answer is C. The rotator cuff consists of the tendons of the supraspinatus, infraspinatus, subscapularis, and teres minor muscles. It stabilizes the shoulder joint by holding the head of the humerus in the glenoid cavity during movement. The teres major inserts on the medial lip of the intertubercular groove of the humerus.

32. A cardiologist is on clinical rounds with her medical students. She asks them, "During the cardiac cycle, which of the following events occurs?" (A) AV valves close during diastole (B) Aortic valve closes during systole (C) Pulmonary valve opens during diastole (D) Blood flow in coronary arteries is maximal during diastole (E) Aortic valve closes at the same time as AV valve

32. The answer is D. During diastole, the AV valves open, and the aortic and pulmonary valves close; whereas during systole, the AV valves close, and the aortic and pulmonary valves open.

Questions 31 to 35: A 20-year-old college student receives a severe blow on the inferolateral side of the left knee joint while playing football. Radiographic examination reveals a fracture of the head and neck of the fibula. 33. If the lateral (fibular) collateral ligament is torn by this fracture, which of the following conditions may occur? (A) Abnormal passive abduction of the extended leg (B) Abnormal passive adduction of the extended leg (C) Anterior displacement of the femur on the tibia (D) Posterior displacement of the femur on the tibia (E) Maximal flexion of the leg

33. The Answer is B. The lateral (fibular) collateral ligament prevents adduction at the knee. Therefore, a torn lateral collateral ligament can be recognized by abnormal passive adduction of the extended leg. Abnormal passive abduction of the extended leg may occur when the medial (tibial) collateral ligament is torn. The anterior cruciate ligament prevents posterior displacement of the femur on the tibia; the posterior cruciate ligament prevents anterior displacement of the femur on the tibia. In addition, the posterior cruciate ligament is taut when the knee is fully flexed.

33. A 38-year-old woman with peptic ulcer disease of the stomach experiences severe abdominal pain. Which of the following nervous structures is most likely involved? (A) Greater splanchnic nerve (B) Ventral roots of the spinal nerve (C) Lower intercostal nerve (D) Vagus nerve (E) Gray ramus communicans

33. The answer is A. The greater splanchnic nerve carries pain fibers from the upper GI tract. Neither the ventral roots of the spinal nerves nor the gray rami communicantes contain sensory nerve fibers. The vagus nerve contains sensory fibers associated with reflexes, but it does not contain pain fibers. The lower intercostal nerves carry general somatic afferent (GSA) pain fibers from the diaphragm, abdominal wall, and peritoneum but not GVA pain fibers from the GI tract.

33. While performing a pelvic exenteration, the surgical oncologist notices a fractured or ruptured boundary of the pelvic inlet. Which of the following structures is most likely damaged? (A) Promontory of the sacrum (B) Anterior-inferior iliac spine (C) Inguinal ligament (D) Iliac crest (E) Arcuate pubic ligament

33. The answer is A. The pelvic inlet (pelvic brim) is bounded by the promontory and the anterior border of the ala of the sacrum, the arcuate line of the ilium, the pectineal line, the pubic crest, and the superior margin of the pubic symphysis.

33. A benign tumor in the orbit of a 49-year-old man compresses a structure that runs through both the superior orbital fissure and the common tendinous ring. Which of the following structures is most likely damaged? (A) Frontal nerve (B) Lacrimal nerve (C) Trochlear nerve (D) Abducens nerve (E) Ophthalmic vein

33. The answer is D. The abducens nerve enters the orbit through the superior orbital fissure and the common tendinous ring. The trochlear, lacrimal, and frontal nerves and the ophthalmic vein enter the orbit through the superior orbital fissure outside the common tendinous ring.

33. A patient complains of having pain with repeated movements of his thumb (claudication). His physician performs the Allen test and finds an insufficiency of the radial artery. Which of the following conditions would be a result of the radial artery stenosis? (A) A marked decrease in the blood flow in the superficial palmar arterial arch (B) Decreased pulsation in the artery passing superficial to the flexor retinaculum (C) Ischemia of the entire extensor muscles of the forearm (D) A marked decrease in the blood flow in the princeps pollicis artery (E) A low blood pressure in the anterior interosseous artery

33. The answer is D. The radial artery divides into the princeps pollicis artery and the deep palmar arterial arch. Thus, stenosis of the radial artery results in a decreased blood flow in the princeps pollicis artery. The superficial palmar arterial arch is formed primarily by the ulnar artery, which passes superficial to the flexor retinaculum. The extensor compartment of the forearm receives blood from the posterior interosseous artery, which arises from the common interosseous branch of the ulnar artery. However, the radial and radial recurrent arteries supply the brachioradialis and the extensor carpi radialis longus and brevis.

33. Coronary angiographs of a 44-year-old male patient reveal an occlusion of the circumflex branch of the left coronary artery. This patient has been suffering from myocardial infarction in which of the following areas? (A) Right and left ventricles (B) Right and left atria (C) Interventricular septum (D) Apex of the heart (E) Left atrium and ventricle

33. The answer is E. The left atrium and ventricle receive blood from the circumflex branch of the left coronary artery. The interventricular septum and the apex of the heart are supplied by the anterior interventricular branch of the left coronary artery. The right ventricle receives blood from the anterior interventricular artery and the marginal branch of the right coronary artery. The right atrium receives blood from the right coronary artery.

Questions 31 to 35: A 20-year-old college student receives a severe blow on the inferolateral side of the left knee joint while playing football. Radiographic examination reveals a fracture of the head and neck of the fibula. 34. Which of the following arteries could also be damaged by this fracture? (A) Popliteal (B) Posterior tibial (C) Anterior tibial (D) Peroneal (E) Lateral inferior genicular

34. The Answer is C. The anterior tibial artery, which arises from the popliteal artery, enters the anterior compartment by passing through the gap between the fibula and tibia at the upper end of the interosseous membrane. The other arteries would not be affected because they are not closely associated with the head and neck of the fibula.

34. A patient bleeding from the shoulder secondary to a knife wound is in fair condition because there is vascular anastomosis around the shoulder. Which of the following arteries is most likely a direct branch of the subclavian artery that is involved in the anastomosis? (A) Dorsal scapular artery (B) Thoracoacromial artery (C) Circumflex scapular artery (D) Transverse cervical artery (E) Suprascapular artery

34. The answer is A. The dorsal scapular artery arises directly from the third part of the subclavian artery and replaces the deep (descending) branch of the transverse cervical artery. The suprascapular and transverse cervical arteries are branches of the thyrocervical trunk of the subclavian artery. The thoracoacromial artery is a short trunk from the first or second part of the axillary artery and has pectoral, clavicular, acromial, and deltoid branches.

34. A patient has a small but solid tumor in the mediastinum, which is confined at the level of the sternal angle. Which of the following structures would most likely be found at this level? (A) Bifurcation of the trachea (B) Beginning of the ascending aorta (C) Middle of the aortic arch (D) Articulation of the third rib with the sternum (E) Superior border of the superior mediastinum

34. The answer is A. The sternal angle is the junction of the manubrium and the body of the sternum. It is located at the level where the second rib articulates with the sternum, the trachea bifurcates into the right and left bronchi, and the aortic arch begins and ends. It marks the end of the ascending aorta and the beginning of the descending aorta, and it forms the inferior border of the superior mediastinum.

34. A 37-year-old man feels a little discomfort when moving his tongue, pharynx, and larynx. Physical examination indicates that the muscles attached to the styloid process are paralyzed. Which of the following groups of cranial nerves are damaged? (A) Facial, glossopharyngeal, and hypoglossal nerves (B) Hypoglossal, vagus, and facial nerves (C) Glossopharyngeal, trigeminal, and vagus nerves (D) Vagus, spinal accessory, and hypoglossal nerves (E) Facial, glossopharyngeal, and vagus nerves

34. The answer is A. The styloid process provides attachments for the stylohyoid, styloglossus, and stylopharyngeus muscles. The stylohyoid muscle is innervated by the facial nerve, the styloglossus muscle is innervated by the hypoglossal nerve, and the stylopharyngeus muscle is innervated by the glossopharyngeal nerve. No other muscles are attached to the styloid process.

34. A 32-year-old patient with multiple fractures of the pelvis has no cutaneous sensation in the urogenital triangle. The function of which of the following nerves is most likely to be spared? (A) Ilioinguinal nerve (B) Iliohypogastric nerve (C) Posterior cutaneous nerve of the thigh (D) Pudendal nerve (E) Genitofemoral nerve

34. The answer is B. The iliohypogastric nerve innervates the skin above the pubis. The skin of the urogenital triangle is innervated by the pudendal nerve, perineal branches of the posterior femoral cutaneous nerve, anterior scrotal or labial branches of the ilioinguinal nerve, and the genital branch of the genitofemoral nerve.

34. A 3-year-old boy is diagnosed as having a persistent processus vaginalis in its middle portion. Which of the following conditions is most likely to be associated with this developmental anomaly? (A) Direct inguinal hernia (B) Gubernaculum testis (C) Hematocele (D) Hydrocele (E) Cryptorchidism

34. The answer is D. If a middle portion of the processus vaginalis persists, it forms a congenital hydrocele. If the entire processus vaginalis persists, it develops a congenital indirect inguinal hernia. Gubernaculum testis is the fetal ligament that connects the bottom of the fetal testis to the developing scrotum. Hematocele is an effusion of blood into the cavity of the tunica vaginalis. Cryptorchidism is failure of the testis to descend from the abdomen to the scrotum.

Questions 31 to 35: A 20-year-old college student receives a severe blow on the inferolateral side of the left knee joint while playing football. Radiographic examination reveals a fracture of the head and neck of the fibula. 35. Which of the following conditions would occur from this fracture? (A) Ischemia in the gastrocnemius (B) Loss of plantar flexion (C) Trendelenburg sign (D) Anterior tibial compartment syndrome (E) Flat foot.

35. The Answer is D. Anterior tibial compartment syndrome is characterized by ischemic necrosis of the muscles of the anterior tibial compartment of the leg resulting from damage to the anterior tibial artery. The gastrocnemius receives blood from sural branches of the popliteal artery. Loss of plantar flexion is due to necrosis of the posterior muscles of the leg, which are supplied by the posterior tibial and peroneal arteries. Trendelenburg sign is caused by weakness or paralysis of the gluteus medius and minimus muscles. Flat foot results from the collapse of the medial longitudinal arch of the foot.

35. A high school basketball player experiences a sudden difficulty in breathing and is brought to an emergency department. When a low tracheotomy is performed below the isthmus of the thyroid, which of the following vessels may be encountered? (A) Inferior thyroid artery (B) Inferior thyroid vein (C) Costocervical trunk (D) Superior thyroid artery (E) Right brachiocephalic vein

35. The answer is B. A low tracheotomy is a surgical incision of the trachea through the neck and below the isthmus of the thyroid gland. The inferior thyroid veins drain the thyroid gland, descend in front of the trachea, and enter the brachiocephalic veins. Consequently, these veins are closely associated with the isthmus of the thyroid gland. Other blood vessels are not closely related with the front of the trachea and the isthmus of the thyroid gland.

35. A 37-year-old house painter fell from a ladder and fractured his left third rib and the structures with which it articulated. Which of the following structures would most likely be damaged? (A) Manubrium of the sternum (B) Body of the second thoracic vertebra (C) Spinous process of the third thoracic vertebra (D) Body of the fourth thoracic vertebra (E) Transverse process of the second thoracic vertebra

35. The answer is B. The third rib articulates with the body of the sternum, bodies of the second and third thoracic vertebrae, and transverse process of the third thoracic vertebra.

35. During a breast examination of a 56-year old woman, the physician found a palpable mass in her breast. Which of the following characteristics of breast cancer and its diagnosis is correct? (A) Elevated nipple (B) Polymastia (C) Shortening of the clavipectoral fascia (D) Dimpling of the overlying skin (E) Enlargement of the breast

35. The answer is D. Breast cancer may cause dimpling of the overlying skin because of shortening of the suspensory (Cooper) ligaments and inverted or retracted nipple because of pulling on the lactiferous ducts. Polymastia is a condition in which more than two breasts are present.

35. A 22-year-old victim of an automobile accident has received destructive damage to structures that form the boundary of the perineum. Which of the following structures is spared? (A) Pubic arcuate ligament (B) Tip of the coccyx (C) Ischial tuberosities (D) Sacrospinous ligament (E) Sacrotuberous ligament

35. The answer is D. The sacrospinous ligament forms a boundary of the lesser sciatic foramen. The pubic arcuate ligament, tip of the coccyx, ischial tuberosities, and sacrotuberous ligament all form part of the boundary of the perineum.

35. Examination of a 54-year-old man reveals an isolated tumor located at the porta hepatis. This tumor most likely compresses which of the following structures? (A) Cystic duct (B) Hepatic veins (C) Common hepatic artery (D) Left gastric artery (E) Branches of the portal vein

35. The answer is E. The porta hepatis is the transverse fissure (doorway) in the liver and contains the hepatic ducts, hepatic arteries, and branches of the portal vein. The other structures are not found in the porta hepatis.

36. A construction worker is hit on the leg with a concrete block and is subsequently unable to plantar flex and invert his foot. Which of the following muscles is most likely damaged? (A) Extensor digitorum longus (B) Tibialis anterior (C) Tibialis posterior (D) Peroneus longus (E) Peroneus brevis

36. The Answer is C. The tibialis posterior can plantar flex and invert the foot. The extensor digitorum longus can dorsiflex and evert the foot, the tibialis anterior can dorsiflex and invert the foot, and the peroneus longus and brevis can plantar flex and evert the foot.

36. A patient with a stab wound receives a laceration of the musculocutaneous nerve. Which of the following conditions is most likely to have occurred? (A) Lack of sweating on the lateral side of the forearm (B) Inability to extend the forearm (C) Paralysis of brachioradialis muscle (D) Loss of tactile sensation on the arm (E) Constriction of blood vessels on the hand

36. The answer is A. The musculocutaneous nerve contains sympathetic postganglionic fibers that supply sweat glands and blood vessels on the lateral side of the forearm as the lateral antebrachial cutaneous nerve. The musculocutaneous nerve does not supply the extensors of the forearm and the brachioradialis. This nerve also supplies tactile sensation on the lateral side of the forearm but not the arm and supplies blood vessels on the lateral side of the forearm but not the hand.

36. A 32-year-old man undergoes vasectomy as a means of permanent birth control. A physician performing the vasectomy by making an incision on each side of the scrotum should remember which of the following statements most applicable to the scrotum? (A) It is innervated by the ilioinguinal and genitofemoral nerves. (B) It receives blood primarily from the testicular artery. (C) Its venous blood drains primarily into the renal vein on the left. (D) Its lymphatic drainage is primarily into upper lumbar nodes. (E) Its dartos tunic is continuous with the perineal membrane.

36. The answer is A. The scrotum is innervated by branches of the ilioinguinal, genitofemoral, pudendal, and posterior femoral cutaneous nerves. The scrotum receives blood from the posterior scrotal branches of the internal pudendal arteries and the anterior scrotal branches of the external pudendal arteries, but it does not receive blood from the testicular artery. Similarly, the scrotum is drained by the posterior scrotal veins into the internal pudendal vein. The lymph vessels from the scrotum drain into the superficial inguinal nodes, whereas the lymph vessels from the testis drain into the upper lumbar nodes. The dartos tunic is continuous with the membranous layer of the superficial perineal fascia (Colles fascia).

36. A patient is rushed to the operating room for an emergent cholecystectomy (resection of a gallbladder) because of cholecystitis. While locating landmarks before surgical resection of an infected gallbladder, the surgeon recalls a portal-caval anastomosis. Which of the following pairs of veins form a portal-caval anastomosis? (A) Hepatic veins and IVC (B) Superior and middle rectal vein (C) Left and right gastric vein (D) Inferior and superficial epigastric veins (E) Suprarenal and renal veins

36. The answer is B. Portal-caval anastomoses occur between the left gastric vein and esophageal vein of the azygos, the superior rectal and middle or inferior rectal veins, paraumbilical and superficial epigastric veins, and retrocolic veins and twigs of the renal vein. The hepatic veins and the IVC are systemic or caval veins. The left and right gastric veins belong to the portal venous system. The inferior and superficial epigastric veins and the suprarenal and renal veins are systemic veins.

36. A 45-year-old woman presents with a tumor confined to the posterior mediastinum. This could result in compression of which of the following structures? (A) Trachea (B) Descending aorta (C) Arch of the aorta (D) Arch of the azygos vein (E) Phrenic nerve

36. The answer is B. The descending aorta is found in the posterior mediastinum. The superior mediastinum contains the trachea and arch of the aorta, and the middle mediastinum contains the ascending aorta, arch of the azygos vein, and main bronchi. The phrenic nerve runs in the middle mediastinum

36. A 59-year-old man complains of numbness in the anterior cervical triangle. Therefore, damage has occurred to which of the following nerves? (A) Phrenic nerve (B) Greater auricular nerve (C) Transverse cervical nerve (D) Supraclavicular nerve (E) Lesser occipital nerve

36. The answer is C. The transverse cervical nerve turns around the posterior border of the sternocleidomastoid and innervates the skin of the anterior cervical triangle. The phrenic nerve, a branch of the cervical plexus, contains motor and sensory fibers but no cutaneous nerve fibers. The greater auricular nerve innervates the skin behind the auricle and on the parotid gland. The supraclavicular nerve innervates the skin over the clavicle and the shoulder. The lesser occipital nerve innervates the scalp behind the auricle.

37. The obturator nerve and the sciatic (tibial portion) nerve of a 15-year-old boy are transected as a result of a motorcycle accident. This injury would result in complete paralysis of which of the following muscles? (A) Rectus femoris (B) Biceps femoris, short head (C) Pectineus (D) Adductor magnus (E) Sartorius

37. The Answer is D. The adductor magnus is innervated by both the obturator and sciatic (tibial portion) nerves. Hence, a lesion here could cause paralysis. The rectus femoris and sartorius are innervated by the femoral nerve. The biceps femoris long head is innervated by the tibial portion of the sciatic nerve, whereas the short head is innervated by the common peroneal portion of the sciatic nerve. The pectineus is innervated by both the femoral and obturator nerves.

37. A 37-year-old woman complains of a bearing- down sensation in her womb and an increased frequency of and burning sensation on urination. On examination by her gynecologist, she is diagnosed with a uterine prolapse. Which of the following structures provides the primary support for the cervix of the uterus? (A) External anal sphincter (B) Broad ligament of the uterus (C) Cardinal (transverse cervical) ligament (D) Round ligament of the uterus (E) Suspensory ligament of the ovary

37. The answer is C. The cardinal (transverse cervical) ligament provides the major ligamentous support for the uterus. The sphincter ani externus does not support the uterus. The broad and round ligaments of the uterus provide minor supports for the uterus. The suspensory ligament of the ovary does not support the uterus.

37. A 53-year-old man has difficulty with breathing through his nose. On examination, his physician finds that he has swelling of the mucous membranes of the superior nasal meatus. Which opening of the paranasal sinuses is most likely plugged? (A) Middle ethmoidal sinus (B) Maxillary sinus (C) Posterior ethmoidal sinus (D) Anterior ethmoidal sinus (E) Frontal sinus

37. The answer is C. The posterior ethmoidal sinus opens into the superior nasal meatus. The maxillary, frontal, and anterior and middle ethmoidal sinuses drain into the middle nasal meatus.

37. Mrs. Jones is undergoing a routine colonoscopy for colon cancer prevention. The gastroenterologist finds a Meckel diverticulum. Which of the following statements is true about the diverticulum? (A) It is found 2 ft distal to the ileocecal junction (B) It is located on the mesenteric side of the ileum (C) It occurs in approximately 20% of the population (D) It is a persistent remnant of the embryonic yolk stalk (E) It may contain renal and suprarenal tissues

37. The answer is D. The Meckel diverticulum is a persistent remnant of the yolk stalk (vitelline duct) and located 2 ft proximal to the ileocecal junction on the antimesenteric border of the ileum. It is approximately 2 in. long, occurs in approximately 2% of the population, and contains two types of mucosal (gastric and pancreatic) tissues in its wall.

37. A 62-year-old patient with pericardial effusion comes to a local hospital for aspiration of pericardial fluid by pericardiocentesis. The needle is inserted into the pericardial cavity through which of the following intercostal spaces adjacent to the sternum? (A) Right fourth intercostal space (B) Left fourth intercostal space (C) Right fifth intercostal space (D) Left fifth intercostal space (E) Right sixth intercostal space

37. The answer is D. To aspirate pericardial fluid, the needle should be inserted into the pericardial cavity through the fifth intercostals space just left to the sternum. Because of the cardiac notch, the needle misses the pleura and lungs, but it penetrates the pericardium. Lung tissues lie beneath the fourth and sixth intercostal spaces.

37. A 20-year-old man fell from the parallel bar during the Olympic trial. A neurologic examination reveals that he has a lesion of the lateral cord of the brachial plexus. Which of the following muscles is most likely weakened by this injury? (A) Subscapularis (B) Teres major (C) Latissimus dorsi (D) Teres minor (E) Pectoralis major

37. The answer is E. The pectoralis major is innervated by the lateral and medial pectoral nerves originating from the lateral and medial cords of the brachial plexus, respectively. The subscapularis, teres major, latissimus dorsi, and teres minor muscles are innervated by nerves originating from the posterior cord of the brachial plexus.

38. A 24-year-old woman presents to her physician with weakness in flexing the hip joint and extending the knee joint. Which muscle is most likely involved in this scenario? (A) Sartorius (B) Gracilis (C) Rectus femoris (D) Vastus medialis (E) Semimembranosus

38. The Answer is C. The rectus femoris flexes the thigh and extends the leg. The sartorius can flex both the hip and knee joints. The gracilis adducts and flexes the thigh and flexes the leg, the vastus medialis extends the knee joint, and the semimembranosus extends the hip joint and flexes the knee joint.

38. The attending faculty in the coronary intensive care unit demonstrates to his students a normal heart examination. The first heart sound is produced by near-simultaneous closure of which of the following valves? (A) Aortic and tricuspid (B) Aortic and pulmonary (C) Tricuspid and mitral (D) Mitral and pulmonary (E) Tricuspid and pulmonary

38. The answer is C. The first heart sound ("lub") is produced by the closure of the tricuspid and mitral valves, whereas the second heart sound ("dub") is produced by the closure of the aortic and pulmonary valves.

38. A woman is delivering a breech baby. The obstetrician decides that it is best to perform a mediolateral episiotomy. Which of the following structures should the obstetrician avoid incising? (A) Vaginal wall (B) Superficial transverse perineal muscle (C) Bulbospongiosus (D) Levator ani (E) Perineal membrane

38. The answer is D. An obstetrician should avoid incising the levator ani and the external anal sphincter. The levator ani is the major part of the pelvic diaphragm, which forms the pelvic floor and supports all of the pelvic organs. None of the other choices applies here.

38. A 24-year-old carpenter suffers a crush injury of his entire little finger. Which of the following muscles is most likely to be spared? (A) Flexor digitorum profundus (B) Extensor digitorum (C) Palmar interossei (D) Dorsal interossei (E) Lumbricals

38. The answer is D. The dorsal interossei are abductors of the fingers. The little finger has no attachment for the dorsal interosseous muscle because it has its own abductor. Therefore, the dorsal interosseous muscle is not affected. Other muscles are attached to the little finger; thus, they are injured.

38. A 54-year-old man comes to a hospital with abdominal pain, jaundice, loss of appetite, and weight loss. On examination of his radiograms and CT scans, a physician finds a slowly growing tumor in the uncinate process of the pancreas. Which of the following structures is most likely compressed by this tumor? (A) Main pancreatic duct (B) Splenic artery (C) Portal vein (D) Superior mesenteric artery (E) Superior pancreaticoduodenal artery

38. The answer is D. The uncinate process of the pancreas is a projection of the lower part of its head to the left behind the superior mesenteric vessels. The superior pancreaticoduodenal artery runs between the duodenum and the head of the pancreas. The main pancreatic duct runs transversely through the entire pancreas superior to the uncinate process. The splenic artery runs along the superior border of the pancreas. The portal vein runs behind the neck of the pancreas

38. A 61-year-old woman is found to have ocular lymphoma invading her optic canal. Which of the following structures would most likely be damaged? (A) Ophthalmic vein (B) Ophthalmic nerve (C) Oculomotor nerve (D) Trochlear nerve (E) Ophthalmic artery

38. The answer is E. The optic canal transmits the ophthalmic artery and optic nerve. The ophthalmic nerve, ophthalmic vein, and oculomotor and trochlear nerves enter the orbit through the superior orbital fissure.

39. A 17-year-old boy was stabbed during a gang fight, resulting in the transection of the obturator nerve. Which of the following muscles is completely paralyzed? (A) Pectineus (B) Adductor magnus (C) Adductor longus (D) Biceps femoris (E) Semimembranosus

39. The Answer is C. The adductor longus is innervated by only the obturator nerve. Thus, injury here could completely paralyze the adductor longus. The pectineus is innervated by both the obturator and femoral nerves. The adductor magnus is innervated by both the obturator nerve and tibial part of the sciatic nerve. The biceps femoris is innervated by the tibial portion (long head) and common peroneal portion (short head) of the sciatic nerve. The semimembranosus is innervated by the tibial portion of the sciatic nerve

39. A 76-year-old man with swallowing difficulties undergoes imaging for a possible mass. The CT scan image at the level of the cricothyroid ligament in his neck should show which of the following structures? (A) Inferior laryngeal nerves (B) External carotid arteries (C) Inferior thyroid veins (D) Thyrocervical trunks (E) Internal laryngeal nerves

39. The answer is A. A CT scan through the cricothyroid ligament shows the inferior laryngeal nerves, which are the terminal portion of the recurrent laryngeal nerves above the lower border of the cricoid cartilage. The external carotid arteries and the internal laryngeal nerves lie above the cricothyroid ligament, and the inferior thyroid veins and the thyrocervical trunks lie below the ligament.

39. During pelvic surgery, a surgeon notices severe bleeding from the artery that remains within the true pelvis. Which of the following arteries is most likely to be injured? (A) Iliolumbar artery (B) Obturator artery (C) Uterine artery (D) Internal pudendal artery (E) Inferior gluteal artery

39. The answer is C. Of all the arteries listed, the uterine artery remains within the pelvic cavity.

39. A 7-year-old boy falls from a tree house and is brought to the emergency department of a local hospital. On examination, he has weakness in rotating his arm laterally because of an injury of a nerve. Which of the following conditions is most likely to cause a loss of this nerve function? (A) Injury to the lateral cord of the brachial plexus (B) Fracture of the anatomic neck of the humerus (C) Knife wound on the teres major muscle (D) Inferior dislocation of the head of the humerus (E) A tumor in the triangular space in the shoulder region

39. The answer is D. Inferior dislocation of the head of the humerus may damage the axillary nerve, which arises from the posterior cord of the brachial plexus, runs through the quadrangular space accompanied by the posterior humeral circumflex vessels around the surgical neck of the humerus, and supplies the deltoid and teres minor, which are lateral rotators of the arm.

39. A 27-year-old patient with Marfan syndrome has an aneurysm of the aortic arch. This may compress which of the following structures? (A) Right vagus nerve (B) Left phrenic nerve (C) Right sympathetic trunk (D) Left recurrent laryngeal nerve (E) Left greater splanchnic nerve

39. The answer is D. The left recurrent laryngeal nerve loops around the arch of the aorta near the ligamentum arteriosum, whereas the right recurrent laryngeal nerve hooks around the right subclavian artery. All other nerves are not closely associated with the aortic arch.

39. A 6-year-old boy comes to his pediatrician with a lump in the groin near the thigh and pain in the groin. On examination, the physician makes a diagnosis of a direct inguinal hernia because the herniated tissue: (A) Enters the deep inguinal ring (B) Lies lateral to the inferior epigastric artery (C) Is covered by spermatic fasciae (D) Descends into the scrotum (E) Develops after birth

39. The answer is E. A direct hernia is acquired (develops after birth), whereas an indirect inguinal hernia is congenital. The direct hernia does not enter the deep inguinal ring but occurs through the posterior wall of the inguinal canal, lies medial to the inferior epigastric artery, is covered only by peritoneum, and does not descend into the scrotum.

4. A 41-year-old man was involved in a fight and felt weakness in extending the knee joint. On examination, he was diagnosed with a lesion of the femoral nerve. Which of the following symptoms would be a result of this nerve damage? (A) Paralysis of the psoas major muscle (B) Loss of skin sensation on the lateral side of the foot (C) Loss of skin sensation over the greater trochanter (D) Paralysis of the vastus lateralis muscle (E) Paralysis of the tensor fasciae latae

4. The answer is D. The femoral nerve innervates the quadratus femoris, sartorius, and vastus muscles. Therefore, damage to this nerve results in paralysis of these muscles. The second and third lumbar nerves innervate the psoas major muscle, the sural nerve innervates the skin on the lateral side of the foot, the iliohypogastric nerve and superior cluneal nerves supply the skin over the greater trochanter, and the superior gluteal nerve innervates the tensor fasciae latae.

4. A 39-year-old woman with headaches presents to her primary care physician with a possible herniated disk. Her magnetic resonance imaging (MRI) scan reveals that the posterolateral protrusion of the intervertebral disk between L4 and L5 vertebrae would most likely affect nerve roots of which of the following spinal nerves? (A) Third lumbar nerve (B) Fourth lumbar nerve (C) Fifth lumbar nerve (D) First sacral nerve (E) Second sacral nerve

4. The Answer is C. A posterolateral herniation of the intervertebral disk at disk level L4 to L5 affects the fifth lumbar nerve root but rarely affects the fourth lumbar nerve root because of a progressive descending obliquity of the fourth and fifth lumbar nerve roots. The first seven cervical nerves exit above the corresponding vertebra, and the eighth cervical nerve exits below the seventh cervical vertebra because there are eight cervical nerves but only seven cervical vertebrae. The rest of the spinal nerves exit below their corresponding vertebrae.

4. A 23-year-old woman suffers from a fracture of the jugular foramen by car accident. Which of the following nerves is/are most likely damaged? (A) Cranial nerve V2 (B) Cranial nerve VI (C) Cranial nerves VII and VIII (D) Cranial nerves IX, X, and XI (E) Cranial nerve XII

4. The Answer is D. The jugular foramen transmits cranial nerves, IX, X, and XI, along with the internal jugular vein. The cranial nerve V2 runs through the foramen rotundum. The cranial nerve VI passes through the superior orbital fissure. The cranial nerves VII and VIII courses through the internal auditory meatus. The cranial nerve XII passes through the hypoglossal canal.

4. A 16-year-old patient received a stab wound, and axons of the general somatic efferent (GSE) neurons to the shoulder muscles were severed. The damaged axons: (A) Would carry impulses toward the cell bodies (B) Would carry impulses away from the cell bodies (C) Would carry pain impulses (D) Are several in numbers for multipolar neurons (E) Are found primarily in the gray matter

4. The answer is B. The axons of the neurons carry impulses away from the cell bodies, and dendrites carry impulses to the cell bodies. The axons contain sensory or motor fibers. Multipolar neurons have several dendrites and one axon. The GSE neurons do not carry sensory impulses. The gray matter of the central nervous system consists largely of neuron cell bodies, dendrites, and neuroglia, whereas the white matter consists largely of axons and neuroglia.

4. A 29-year-old man comes to a local hospital with duodenal peptic ulcer and complains of cramping epigastric pain. Which of the following structures harbors the cell bodies of abdominal pain fibers? (A) Lateral horn of the spinal cord (B) Anterior horn of the spinal cord (C) Dorsal root ganglion (D) Sympathetic chain ganglion (E) Celiac ganglion

4. The answer is C. Cell bodies of the abdominal pain fibers are located in the dorsal root ganglion. The lateral horn of the spinal cord contains cell bodies of sympathetic preganglionic nerve fibers; the anterior horn contains cell bodies of general somatic efferent (GSE) fibers. The sympathetic chain ganglion contains cell bodies of sympathetic postganglionic fibers, which supply blood vessels, sweat glands, and hair follicles. The celiac ganglion contains cell bodies of sympathetic postganglionic fibers, which supply the visceral organs such as stomach and intestine.

4. A 22-year-old woman receives a deep cut in the inguinal canal 1 in. lateral to the pubic tubercle. Which of the following ligaments is lacerated within the inguinal canal? (A) Suspensory ligament of the ovary (B) Ovarian ligament (C) Mesosalpinx (D) Round ligament of the uterus (E) Rectouterine ligament

4. The answer is D. The round ligament of the uterus is found in the inguinal canal along its course. The other ligaments do not pass through the inguinal canal.

4. During surgery on a 56-year-old man for a squamous cell carcinoma of the neck, the surgeon notices profuse bleeding from the deep cervical artery. Which of the following arteries must be ligated immediately to stop bleeding? (A) Inferior thyroid artery (B) Transverse cervical artery (C) Thyrocervical trunk (D) Costocervical trunk (E) Ascending cervical artery

4. The answer is D. The surgeon should ligate the costocervical trunk because it divides into the deep cervical and superior intercostal arteries. The thyrocervical trunk gives off the suprascapular, transverse cervical, and inferior thyroid artery. The ascending cervical artery is a branch of the inferior thyroid artery.

4. A 17-year-old boy was involved in a gang fight, and a stab wound severed the white rami communicantes at the level of his sixth thoracic vertebra. This injury would result in degeneration of nerve cell bodies in which of the following structures? (A) Dorsal root ganglion and anterior horn of the spinal cord (B) Sympathetic chain ganglion and dorsal root ganglion (C) Sympathetic chain ganglion and posterior horn of the spinal cord (D) Dorsal root ganglion and lateral horn of the spinal cord (E) Anterior and lateral horns of the spinal cord

4. The answer is D. The white rami communicantes contain preganglionic sympathetic GVE fibers and GVA fibers, whose cell bodies are located in the lateral horn of the spinal cord and the dorsal root ganglia. The sympathetic chain ganglion contains cell bodies of the postganglionic sympathetic nerve fibers. The anterior horn of the spinal cord contains cell bodies of the GSE fibers. The dorsal root ganglion contains cell bodies of GSA and GVA fibers.

4. A 16-year-old patient has weakness flexing the metacarpophalangeal joint of the ring finger and is unable to adduct the same finger. Which of the following muscles is most likely paralyzed? (A) Flexor digitorum profundus (B) Extensor digitorum (C) Lumbrical (D) Dorsal interosseous (E) Palmar interosseous

4. The answer is E. The dorsal and palmar interosseous and lumbrical muscles can flex the metacarpophalangeal joints and extend the interphalangeal joints. The palmar interosseous muscles adduct the fingers, while the dorsal interosseous muscles abduct the fingers. The flexor digitorum profundus flexes the distal interphalangeal (DIP) joints.

40. A 32-year-old carpenter fell from the roof. The lateral longitudinal arch of his foot was flattened from fracture and displacement of the keystone for the arch. Which of the following bones is damaged? (A) Calcaneus (B) Cuboid bone (C) Head of the talus (D) Medial cuneiform (E) Navicular bone

40. The Answer is B. The keystone for the lateral longitudinal arch is the cuboid bone, whereas the keystone for the medial longitudinal arch is the head of the talus. The calcaneus, navicular, and medial cuneiform bones form a part of the medial longitudinal arch, but they are not keystones. The calcaneus also forms a part of the lateral longitudinal arch.

40. A 49-year-old woman is diagnosed as having a large lump in her right breast. Lymph from the cancerous breast drains primarily into which of the following nodes? (A) Apical nodes (B) Anterior (pectoral) nodes (C) Parasternal (internal thoracic) nodes (D) Supraclavicular nodes (E) Nodes of the anterior abdominal wall

40. The answer is B. Lymph from the breast drains mainly (75%) to the axillary nodes, more specifically to the anterior (pectoral) nodes.

40. The muscles that are of branchiomeric origin are paralyzed in a 26-year-old patient. A lesion of which of the following nerves would cause muscle dysfunction? (A) Oculomotor nerve (B) Trochlear nerve (C) Trigeminal nerve (D) Abducens nerve (E) Hypoglossal nerve

40. The answer is C. SVE nerve fibers originate from the first branchial arch (trigeminal), the second arch (facial), the third arch (glossopharyngeal), and the fourth and sixth arches (vagus). Nerves that supply the muscles of the eyeball (oculomotor, trochlear, and abducens) and tongue (hypoglossal) are not of branchiomeric origin.

40. A 47-year-old man with a known atrial fibrillation returns to see his cardiologist for follow-up of his cardiac health. The right atrium is important in this case because it: (A) Receives blood from the oblique cardiac vein (B) Is associated with the apex of the heart (C) Contains the SA node (D) Receives the right pulmonary vein (E) Is hypertrophied by pulmonary stenosis

40. The answer is C. The SA and AV nodes are in the wall of the right atrium and are not associated with the apex of the heart. The oblique cardiac vein drains into the coronary sinus, and the pulmonary veins empty into the left atrium. The right ventricle is hypertrophied by the pulmonary stenosis.

40. A neurosurgeon performs a surgical resection of a rare meningeal tumor in the sacral region. He tries to avoid an injury of the nerve that arises from the lumbosacral plexus and remains within the abdominal or pelvic cavity. To which of the following nerves should he pay particular attention? (A) Ilioinguinal nerve (B) Genitofemoral nerve (C) Lumbosacral trunk (D) Femoral nerve (E) Lateral femoral cutaneous nerve

40. The answer is C. The lumbosacral trunk is formed by part of the ventral ramus of the fourth lumbar nerve and the ventral ramus of the fifth lumbar nerve. This trunk contributes to the formation of the sacral plexus by joining the ventral ramus of the first sacral nerve in the pelvic cavity and does not leave the pelvic cavity. All other nerves leave the abdominal and pelvic cavities.

40. A 21-year-old man developed a hernia after lifting heavy boxes while moving into his new house. During the repair of his resulting hernia, the urologist recalls that the genitofemoral nerve: (A) Runs in front of the quadratus lumborum (B) Is a branch of the femoral nerve (C) Supplies the testis (D) Passes through the deep inguinal ring (E) Gives rise to an anterior scrotal branch

40. The answer is D. The genitofemoral nerve descends on the anterior surface of the psoas muscle and gives rise to a genital branch, which enters the inguinal canal through the deep inguinal ring to supply the cremaster muscle, and a femoral branch, which supplies the skin of the femoral triangle. The genitofemoral nerve is not a branch of the femoral nerve but arises from the lumbar plexus and does not supply the testis. It is the ilioinguinal nerve that gives rise to an anterior scrotal branch.

41. While playing football, a 19-year-old college student receives a twisting injury to his knee when being tackled from the lateral side. Which of the following conditions most likely has occurred? (A) Tear of the medial meniscus (B) Ruptured fibular collateral ligament (C) Tenderness on pressure along the fibular collateral ligament (D) Injury of the posterior cruciate ligament (E) Swelling on the back of the knee joint

41. The Answer is A. The "unhappy triad" of the knee joint is characterized by tear of the medial meniscus, rupture of the tibial collateral ligament, and rupture of the anterior cruciate ligament. This injury may occur when a cleated shoe, as worn by football players, is planted firmly in the turf and the knee is struck from the lateral side. Tenderness along the medial collateral ligament and over the medial meniscus and swelling on the front of the joint are due to excessive production of synovial fluid, which fills the joint cavity and the suprapatellar bursa

41. A 17-year-old boy fell from his motorcycle and complains of numbness of the lateral part of the arm. Examination reveals that the axillary nerve is severed. Which of the following types of axons is most likely spared? (A) Postganglionic sympathetic axons (B) Somatic afferent axons (C) Preganglionic sympathetic axons (D) General somatic efferent axons (E) General visceral afferent axons

41. The answer is C. The axillary nerve contains no preganglionic sympathetic general visceral efferent (GVE) fibers, but it contains postganglionic sympathetic GVE fibers. The axillary nerve also contains GSA, GSE, and general visceral afferent (GVA) fibers.

41. A 57-year-old patient has a heart murmur resulting from the inability to maintain constant tension on the cusps of the AV valve. Which of the following structures is most likely damaged? (A) Crista terminalis (B) Septomarginal trabecula (C) Chordae tendineae (D) Pectinate muscle (E) Anulus fibrosus

41. The answer is C. The chordae tendineae are tendinous strands that extend from the papillary muscles to the cusps of the valve. The papillary muscles and chordae tendineae prevent the cusps from being everted into the atrium during ventricular contraction.

41. An oncologist is reviewing a CT scan of a 74-year-old man with newly diagnosed hepatocellular carcinoma. He locates the affected quadrate lobe of the liver that: (A) Lies between the IVC and ligamentum venosum (B) Receives blood from the right hepatic artery (C) Drains bile into the left hepatic duct (D) Is a medial superior segment (E) Is functionally a part of the right lobe

41. The answer is C. The quadrate lobe of the liver drains bile into the left hepatic duct and receives blood from the left hepatic artery. It lies between the gallbladder fossa and the ligamentum teres hepatic, is a medial inferior segment, and is a part of the left lobe.

41. During surgery for a malignant parotid tumor in a 69-year-old woman, the main trunk of the facial nerve is lacerated. Which of the following muscles is paralyzed? (A) Masseter muscle (B) Stylopharyngeus muscle (C) Anterior belly of the digastric muscle (D) Buccinator muscle (E) Tensor tympani

41. The answer is D. The buccinator muscle is innervated by the facial nerve. The masseter, anterior belly of the digastric, and tensor tympani muscles are innervated by the mandibular division of the trigeminal nerve. The stylopharyngeus muscle is innervated by the glossopharyngeal nerve.

41. After repair of a ruptured diverticulum, a 31-year-old patient begins to spike with fever and complains of abdominal pain. An infection in the deep perineal space would most likely damage which of the following structures? (A) Ischiocavernosus muscles (B) Superficial transverse perineal muscles (C) Levator ani (D) Sphincter urethrae (E) Bulbospongiosus

41. The answer is D. The sphincter urethrae are found in the deep perineal space, whereas the other structures are located in the superficial perineal space.

42. A patient has weakness when flexing both her thigh and leg. Which of the following muscles is most likely injured? (A) Rectus femoris (B) Semitendinosus (C) Biceps femoris (D) Sartorius (E) Adductor longus

42. The Answer is D. The sartorius can flex and rotate the thigh laterally, and flex and rotate the leg medially. The rectus femoris flexes the thigh and extends the leg. The semimembranosus extends the thigh and flexes and rotates the leg medially. The biceps femoris extends the thigh and flexes and rotates the leg laterally. The adductor longus adducts and flexes the thigh.

42. A 58-year-old man is presented with edema of the lower limb and enlarged superficial veins of the abdominal wall. Examination of radiographs and angiograms reveals obstruction of the IVC just proximal to the entrance of the renal vein. This venous blockage may result in dilation of which of the following veins? (A) Left suprarenal vein (B) Right inferior phrenic vein (C) Right hepatic vein (D) Left gastric vein (E) Portal vein

42. The answer is A. The veins distal to obstruction are dilated, but the veins proximal to obstruction are not dilated but have low blood pressure. The suprarenal vein drains into the left renal vein and thus is dilated because of high pressure. The right phrenic and right hepatic veins drain into the IVC above the obstruction. The left gastric vein joins the portal vein, which enters the liver.

42. A mother with diabetes gives birth to a baby who is diagnosed as having dextroposition of the aorta and the pulmonary trunk with cyanosis and shortness of breath. Which of the following structures is required to remain patent until surgical correction of the deformity? (A) Umbilical arteries (B) Umbilical vein (C) Ductus arteriosus (D) Ductus venosus (E) Sinus venosus

42. The answer is C. A patent ductus arteriosus shunts blood from the pulmonary trunk to the aorta, partially bypassing the lungs, and thus allowing mixed blood to reach the body tissues and causing cyanosis. Dextroposition or transposition of the great arteries must be accompanied by a VSD or a patent ductus arteriosus for the infant to survive. The transposition causes oxygenated blood to pass from the left ventricle into the pulmonary trunk and then into the lungs, but deoxygenated blood travels from the right ventricle into the aorta and then into the systemic circulation.

42. During a gang fight, a 17-year-old boy is punched, and his nasal septum is broken. Which of the following structures would be damaged? (A) Septal cartilage and nasal bone (B) Inferior concha and vomer (C) Vomer and perpendicular plate of ethmoid (D) Septal cartilage and middle concha (E) Cribriform plate and frontal bone

42. The answer is C. The nasal septum is formed primarily by the vomer, the perpendicular plate of ethmoid bone, and the septal cartilage. The superior, middle, and inferior conchae form the lateral wall of the nasal cavity. The ethmoid (cribriform plate), nasal, frontal, and sphenoid (body) bones form the roof. The floor is formed by the palatine process of the maxilla and the horizontal plate of the palatine bone.

42. A radiologist interprets a lymphangiogram for a 29-year-old patient with metastatic carcinoma. Upper lumbar nodes most likely receive lymph from which of the following structures? (A) Lower part of the anal canal (B) Labium majus (C) Clitoris (D) Testis (E) Scrotum

42. The answer is D. Lymphatic vessels from the testis and epididymis ascend along the testicular vessels in the spermatic cord through the inguinal canal and continue upward in the abdomen to drain into the upper lumbar nodes. The lymph from the other structures drains into the superficial inguinal lymph nodes.

42. A construction worker suffers a destructive injury of the structures related to the anatomic snuffbox. Which of the following structures would most likely be damaged? (A) Triquetral bone (B) Trapezoid bone (C) Extensor indicis tendon (D) Abductor pollicis brevis tendon (E) Radial artery

42. The answer is E. The radial artery lies on the floor of the anatomic snuffbox. Other structures are not related to the snuffbox. The tendons of the extensor pollicis longus, extensor pollicis brevis, and abductor pollicis longus muscles form the boundaries of the anatomic snuffbox. The scaphoid and trapezium bones form its floor.

43. A 35-year-old man has difficulty in dorsiflexing the foot. Which of the following muscles is most likely damaged? (A) Tibialis posterior (B) Flexor digitorum longus (C) Tibialis anterior (D) Peroneus longus (E) Peroneus brevis

43. The Answer is C. The tibialis anterior muscle can dorsiflex the foot, whereas all other muscles are able to plantar flex the foot.

43. A physical fitness trainer for a young Hollywood movie star explains the reasons for 100 stomach crunches a day. The young star, a medical student before 'hitting it big,' reaffirms to his trainer that the lateral margin of the rectus abdominis, the muscle responsible for a washboard stomach, defines which of the following structures? (A) Linea alba (B) Linea semilunaris (C) Linea semicircularis (D) Transversalis fascia (E) Falx inguinalis

43. The answer is B. The linea semilunaris is a curved line along the lateral border of the rectus abdominis. The linea alba is a tendinous median raphe between the two rectus abdominis muscles. The linea semicircularis is an arcuate line of the rectus sheath, which is the lower limit of the posterior layer of the rectus sheath. The falx inguinalis (conjoint tendon) is formed by aponeuroses of the internal oblique and transverse abdominal muscles (otherwise known as the transversalis fascia).

43. A rock climber falls on his shoulder, resulting in a chipping off of the lesser tubercle of the humerus. Which of the following structures would most likely have structural and functional damage? (A) Supraspinatus muscle (B) Infraspinatus muscle (C) Subscapularis muscle (D) Teres minor muscle (E) Coracohumeral ligament

43. The answer is C. The subscapularis muscle inserts on the lesser tubercle of the humerus. The supraspinatus, infraspinatus, and teres minor muscles insert on the greater tubercle of the humerus. The coracohumeral ligament attaches to the greater tubercle.

43. During early development of the respiratory system, the laryngotracheal tube maintains communication with the primitive foregut. Which of the following embryonic structures is most likely responsible for partitioning these two embryonic structures? (A) Tracheoesophageal folds (B) Tracheoesophageal fistula (C) Tracheoesophageal septum (D) Laryngotracheal diverticulum (E) Laryngotracheal septum

43. The answer is C. The tracheoesophageal septum is formed by the fusion of the tracheoesophageal folds in the midline. This septum divides the foregut into a ventral portion, the laryngotracheal tube (primordium of the larynx, trachea, bronchi, and lungs), and a dorsal portion (primordium of the oropharynx and esophagus).

43. A 58-year-old woman comes to the hospital and complains of progressive loss of voice, numbness, loss of taste on the back part of her tongue, and difficulty in shrugging her shoulders. Her MRI scan reveals a dural meningioma that compresses the nerves leaving the skull. These nerves leave the skull through which of the following openings? (A) Foramen spinosum (B) Foramen rotundum (C) Internal auditory meatus (D) Jugular foramen (E) Foramen lacerum

43. The answer is D. A loss of voice is due to an injury to the recurrent laryngeal nerve of the vagus nerve; numbness and loss of taste on the posterior part of the tongue are due to a lesion of the glossopharyngeal nerve; an inability to shrug the shoulder is due to damage of the accessory nerve. These three CNs exit the skull through the jugular foramen. The foramen spinosum transmits the middle meningeal artery. The foramen rotundum transmits the maxillary division of the trigeminal nerve. The internal auditory meatus transmits the facial and vestibulocochlear nerves. The foramen lacerum transmits nothing, but its upper part is traversed by the internal carotid artery with sympathetic nerve plexus.

43. A 49-year-old woman has a large mass on the pelvic brim. Which of the following structures is most likely compressed by this mass when crossing the pelvic brim? (A) Deep dorsal vein of the penis (B) Uterine tube (C) Ovarian ligament (D) Uterine artery (E) Lumbosacral trunk

43. The answer is E. All of the listed structures do not cross the pelvic brim except the lumbosacral trunk, which arises from L4 and L5, enters the true pelvis by crossing the pelvic brim, and contributes to the formation of the sacral plexus. The deep dorsal vein of the penis enters the pelvic cavity by passing under the symphysis pubis between the arcuate and transverse perineal ligaments.

44. An injury to the leg of a golfer results in loss of the ability to invert the foot. Which of the following muscles is most likely paralyzed? (A) Tibialis posterior (B) Peroneus longus (C) Peroneus brevis (D) Peroneus tertius (E) Extensor digitorum longus

44. The Answer is A. The tibialis posterior inverts the foot. The peroneus longus, brevis, and tertius and extensor digitorum longus can evert the foot.

44. A 32-year-old patient has a tension pneumothorax that can be treated with needle aspiration. To avoid an injury of the intercostal neurovascular bundle, the needle may be inserted in which of the following locations? (A) Above the upper border of the ribs (B) Deep to the upper border of the ribs (C) Beneath the lower border of the ribs (D) Between the external and internal intercostals (E) Through the transversus thoracis muscle

44. The answer is A. The intercostal veins, arteries, and nerves run in the costal groove beneath the inferior border of the ribs between the internal and innermost layers of muscles. The transversus thoracis muscles are situated in the internal surface of the lower anterior thoracic wall.

44. A 26-year-old man comes to a hospital with fever, nausea, pain, and itching in the perineal region. On examination by a urologist, he is diagnosed as having infected bulbourethral (Cowper) glands. Which of the following structures is/are affected by this infection? (A) Superficial perineal space (B) Sphincter urethrae (C) Production of sperm (D) Testis (E) Seminal vesicles

44. The answer is B. The bulbourethral glands lie on either side of the membranous urethra, embedded in the sphincter urethrae. Their ducts open into the bulbous part of the penile urethra. Semen—a thick, yellowish-white, viscous, spermatozoa-containing fluid—is a mixture of the secretions of the testes, seminal vesicles, prostate, and bulbourethral glands. Sperm, or spermatozoa, are produced in the seminiferous tubules of the testis and mature in the head of the epididymis. The seminal vesicles are lobulated glandular structures, produce the alkaline constituent of the seminal fluid that contains fructose and choline, and lie inferior and lateral to the ampullae of the ductus deferens against the fundus (base) of the bladder

44. A 21-year-old woman presents to her physician with a swelling on her neck. On examination, she is diagnosed with an infection within the carotid sheath. Which of the following structures would be damaged? (A) Vagus nerve and middle cervical ganglion (B) Internal carotid artery and recurrent laryngeal nerve (C) Internal jugular vein and vagus nerve (D) Sympathetic trunk and common carotid artery (E) External carotid artery and ansa cervicalis

44. The answer is C. The carotid sheath contains the internal jugular vein, vagus nerve, and common and internal carotid arteries. The recurrent laryngeal nerve lies in a groove between the trachea and esophagus. The sympathetic trunk, with superior and middle cervical ganglia, lies behind the carotid sheath. The external carotid artery is not contained within the carotid sheath. The ansa cervicalis lies superficial to or within the carotid sheath.

44. A 22-year-old female Macarena dancer fell from the stage and complains of elbow pain and inability to supinate her forearm. Which of the following nerves are most likely injured from this accident? (A) Median and ulnar nerves (B) Axillary and radial nerves (C) Radial and musculocutaneous nerves (D) Ulnar and axillary nerves (E) Musculocutaneous and median nerves

44. The answer is C. The supinator and biceps brachii muscles supinate the forearm. The supinator is innervated by the radial nerve, and the biceps brachii is innervated by the musculocutaneous nerve.

44. During surgical treatment of portal hypertension in a 59-year-old man with liver cirrhosis, a surgeon inadvertently lacerates the dilated paraumbilical veins. The veins must be repaired to allow collateral flow. Which of the following ligaments is most likely severed? (A) Lienorenal ligament (B) Lienogastric ligament (C) Gastrophrenic ligament (D) Ligamentum teres hepatis (E) Ligamentum venosum

44. The answer is D. The paraumbilical veins and the ligamentum teres hepatis are contained in the free margin of the falciform ligament. The lienorenal ligament contains the splenic vessels and a small portion of the tail of the pancreas. The lienogastric ligament contains the left gastroepiploic and short gastric vessels. The gastrophrenic ligament contains no named structures. The hepatoduodenal ligament, a part of the lesser omentum, contains the bile duct, proper hepatic artery, and portal vein in its free margin.

45. An orthopedic surgeon ligates the posterior tibial artery at its origin. Which of the following arteries has no blood flow immediately after the ligation? (A) Peroneal (B) Dorsalis pedis (C) Superior medial genicular (D) Anterior tibial (E) Descending genicular

45. The Answer is A. The peroneal artery is a branch of the posterior tibial artery. The dorsalis pedis artery begins anterior to the ankle as the continuation of the anterior tibial artery. The superior medial genicular artery is a branch of the popliteal artery, and the descending genicular artery arises from the femoral artery.

45. A 21-year-old man is involved in a highspeed motor vehicle accident. As a result, he has extensive damage to his sphincter urethra. Which of the following best describes the injured sphincter urethra? (A) Smooth muscle (B) Innervated by the perineal nerve (C) Lying between the perineal membrane and Colles fascia (D) Enclosed in the pelvic fascia (E) Part of the pelvic diaphragm

45. The answer is B. The sphincter urethra is a striated muscle that lies in the deep perineal space and forms a part of the urogenital diaphragm but not the pelvic diaphragm. It is not enclosed in the pelvic fascia. It is innervated by a deep (muscular) branch of the perineal nerve.

45. A 43-year-old woman is admitted to the hospital because of deep abdominal pain in her epigastric region. On examination, it is observed that a retroperitoneal infection erodes an artery that runs along the superior border of the pancreas. Which of the following arteries is likely injured? (A) Right gastric artery (B) Left gastroepiploic artery (C) Splenic artery (D) Gastroduodenal artery (E) Dorsal pancreatic artery

45. The answer is C. The splenic artery arises from the celiac trunk, runs along the superior border of the pancreas, and enters the spleen through the lienorenal ligament and the hilus of the spleen. The right gastric artery runs along the lesser curvature of the stomach, and the left gastroepiploic artery runs along the greater curvature of the stomach. The gastroduodenal artery runs behind the first part of the duodenum. The dorsal pancreatic artery descends behind the neck of the pancreas and divides into right and left branches to supply the pancreas.

45. An angiogram of a 45-year-old man shows an occlusion of the costocervical trunk. This obstruction could produce a marked decrease in the blood flow in which of the following arteries? (A) Superior thoracic artery (B) Transverse cervical artery (C) Ascending cervical artery (D) Deep cervical artery (E) Inferior thyroid artery

45. The answer is D. The costocervical trunk gives rise to the deep cervical and superior intercostal arteries. The superior thoracic artery arises from the axillary artery. The transverse cervical, inferior thyroid, and suprascapular arteries arise from the thyrocervical trunk. The ascending cervical artery arises from the inferior thyroid artery.

45. A 9-month-old girl was admitted to the children's hospital with tachypnea (fast breathing) and shortness of breath. Physical examination further exhibits tachycardia (fast heart rate), a bounding peripheral pulse, and her angiographs reveal a patent ductus arteriosus. Which of the following embryonic arterial structures is most likely responsible for the origin of the patent ductus arteriosus? (A) Right fourth arch (B) Left fifth arch (C) Right fifth arch (D) Left sixth arch (E) Right sixth arch

45. The answer is D. The left sixth aortic arch is responsible for the development of both the ductus arteriosus and the pulmonary arteries. The ductus arteriosus closes functionally in an infant soon after birth, with anatomic closure requiring several weeks.

Questions 45 to 47: A 37-year-old female patient has a fracture of the clavicle. The junction of the middle and lateral thirds of the bone exhibits overriding of the medial and lateral fragments. The arm is rotated medially, but it is not rotated laterally. 45. The lateral portion of the fractured clavicle is displaced downward by which of the following? (A) Deltoid and trapezius muscles (B) Pectoralis major and deltoid muscles (C) Pectoralis minor muscle and gravity (D) Trapezius and pectoralis minor muscles (E) Deltoid muscle and gravity

45. The answer is E. The lateral fragment of the clavicle is displaced downward by the pull of the deltoid muscle and gravity. The medial fragment is displaced upward by the pull of the sternocleidomastoid muscle. None of the other muscles are involved.

46. Before knee surgery, a surgeon ligates arteries participating in the anastomosis around the knee joint. Which of the following arteries is most likely spared? (A) Lateral superior genicular (B) Medial inferior genicular (C) Descending branch of the lateral femoral circumflex (D) Saphenous branch of the descending genicular (E) Anterior tibial recurrent

46. The Answer is D. The descending genicular artery gives off the articular branch, which enters the anastomosis around the knee joint, and the saphenous branch, which is not involved in the anastomosis but supplies the superficial tissue and skin on the medial side of the knee. Other arteries are involved in the anastomosis around the knee joint.

46. A 7-day-old baby is diagnosed as having congenital neonatal emphysema, which is caused by collapsed bronchi because of failure of bronchial cartilage development. Bronchial cartilages are derived from which of the following derivations? (A) Ectoderm (B) Mesoderm (C) Endoderm (D) Proctodeum (E) Neuroectoderm

46. The answer is B. Bronchial cartilages, smooth muscles, and connective tissue are derived from the mesoderm. The bronchial epithelium and glands are derived from the endoderm.

46. A 6-month-old male infant is admitted to the children's hospital because he has no testis in his scrotum. During physical examination, the pediatrician palpated the testis in the inguinal canal. What is the diagnosis of this condition? (A) Male pseudohermaphroditism (B) Hypospadias (C) Epispadias (D) Cryptorchid testis (E) Chordee

46. The answer is D. Cryptorchid testis is called an undescended testis, which is located in the inguinal region. Male pseudohermaphroditism is a condition in which the affected individual is a genetic and gonadal male with genital anomalies. Hypospadias occurs when the spongy urethra opens on the underside of the penis, frequently associated with the chordee, which is a ventral curvature of the penis. Epispadias occurs when the urethra opens on the dorsal surface of the penis.

46. A 57-year-old man comes to the local hospital with fever, headache, nausea, and vomiting. Laboratory tests reveal an infection, and radiologic examination localizes the infection to the cavernous sinus. Which of the following nerves would be unaffected by this condition? (A) Oculomotor nerves (B) Abducens nerves (C) Trochlear nerves (D) Mandibular nerves (E) Ophthalmic nerves

46. The answer is D. The mandibular division of the trigeminal nerve does not lie in the wall of the cavernous sinus, whereas the oculomotor, abducens, trochlear, and ophthalmic nerves do.

Questions 45 to 47: A 37-year-old female patient has a fracture of the clavicle. The junction of the middle and lateral thirds of the bone exhibits overriding of the medial and lateral fragments. The arm is rotated medially, but it is not rotated laterally. 46. Which of the following muscles causes upward displacement of the medial fragment? (A) Pectoralis major (B) Deltoid (C) Trapezius (D) Sternocleidomastoid (E) Scalenus anterior

46. The answer is D. The sternocleidomastoid muscle is attached to the superior border of the medial third of the clavicle, and the medial fragment of a fractured clavicle is displaced upward by the pull of the muscle.

46. A 19-year-old young woman with a long history of irritable bowel syndrome presents for the possibility of surgical resection of the gastrointestinal (GI) tract where the vagal parasympathetic innervation terminates. Which of the following sites is most appropriate for surgical resection? (A) Duodenojejunal junction (B) Ileocecal junction (C) Right colic flexure (D) Left colic flexure (E) Anorectal junction

46. The answer is D. The vagus nerve supplies parasympathetic nerve fibers to the GI tract and terminates approximately at the left colic flexure (junction of the transverse colon and the descending colon). The duodenojejunal junction, ileocecal junction, and right colic flexure are supplied by the vagus nerve. The descending colon, sigmoid colon, rectum, anal canal, and anorectal junction are supplied by the pelvic splanchnic nerve for parasympathetic innervation.

47. A 25-year-old gladiator sustains a penetrating injury that severs the superficial peroneal nerve. This will most likely cause paralysis of which of the following muscles? (A) Peroneus tertius (B) Peroneus brevis (C) Flexor hallucis longus (D) Tibialis anterior (E) Tibialis posterior

47. The Answer is B. The peroneus brevis muscle is innervated by the superficial peroneal nerve. The peroneus tertius and tibialis anterior muscles are innervated by the deep peroneal nerve. The flexor hallucis longus and tibialis posterior muscles are innervated by the tibial nerve.

47. A 7-year-old girl has difficulty breathing through her nose and is brought to her pediatrician. On examination, she is diagnosed with adenoids. Which of the following tonsils is enlarged? (A) Palatine tonsil (B) Pharyngeal tonsil (C) Tubal tonsil (D) Lingual tonsil (E) Eustachian tonsil

47. The answer is B. The enlarged pharyngeal tonsil is called an adenoid. An adenoid obstructs passage of air from the nasal cavities through the choanae into the nasopharynx, thus causing difficulty in nasal breathing and phonation. The tubal tonsil is also called the eustachian tonsil. The palatine tonsil is called the faucial tonsil. The submerged tonsil is a palatine tonsil that is shrunken and atrophied and is partly or entirely hidden by the palatoglossal arch.

Questions 45 to 47: A 37-year-old female patient has a fracture of the clavicle. The junction of the middle and lateral thirds of the bone exhibits overriding of the medial and lateral fragments. The arm is rotated medially, but it is not rotated laterally. 47. Which of the following conditions is most likely to occur secondary to the fractured clavicle? (A) A fatal hemorrhage from the brachiocephalic vein (B) Thrombosis of the subclavian vein, causing a pulmonary embolism (C) Thrombosis of the subclavian artery, causing an embolism in the ascending aorta (D) Damage to the upper trunk of the brachial plexus (E) Damage to the long thoracic nerve, causing the winged scapula

47. The answer is B. The fractured clavicle may damage the subclavian vein, resulting in a pulmonary embolism; cause thrombosis of the subclavian artery, resulting in embolism of the brachial artery; or damage the lower trunk of the brachial plexus.

47. A 58-year-old man is admitted to the hospital with severe abdominal pain, nausea, and vomiting resulting in dehydration. Emergency CT scan reveals a tumor located between the celiac trunk and the superior mesenteric artery. Which of the following structures is likely compressed by this tumor? (A) Fundus of the stomach (B) Neck of the pancreas (C) Transverse colon (D) Hepatopancreatic ampulla (E) Duodenojejunal junction

47. The answer is B. The pyloric canal and the neck of the pancreas are situated anterior to the abdominal aorta between the origin of the celiac trunk and the superior mesenteric artery. The transverse colon passes anterior to the superior mesenteric artery and the third part of the duodenum. The other structures are not located in front of the aorta.

47. An obstetrician is about to perform a pudendal block so a woman can experience less pain when she delivers her child. He recalls what he learned in medical school about this nerve. Which of the following statements is correct? (A) It passes superficial to the sacrotuberous ligament (B) It innervates the testis and epididymis in a male (C) It provides motor fibers to the coccygeus (D) It can be blocked by injecting an anesthetic near the inferior margin of the ischial spine (E) It arises from the lumbar plexus

47. The answer is D. The pudendal nerve, which arises from the sacral plexus, provides sensory innervation to the labium majus (or scrotum in a male). It leaves the pelvis through the greater sciatic foramen and enters the perineum through the lesser sciatic foramen near the inferior margin of the ischial spine. Therefore, it can be blocked by injection of an anesthetic near the inferior margin of the ischial spine.

48. A patient presents with a thrombosis in the popliteal vein. This thrombosis most likely causes reduction of blood flow in which of the following veins? (A) Greater saphenous (B) Lesser saphenous (C) Femoral (D) Posterior tibial (E) Anterior tibial

48. The Answer is C. The popliteal vein drains blood into the femoral vein; thus, blood flow in the femoral vein is reduced. The great saphenous vein drains into the upper part of the femoral vein. Other veins empty into the popliteal vein.

48. A trauma surgeon in the emergency department at a local center examines a 14-year-old boy with extensive pelvic injuries after a hit and run accident. The surgeon inspects the ischiorectal fossa because it (A) Accumulates urine leaking from rupture of the bulb of the penis (B) Contains the inferior rectal vessels (C) Has a pudendal canal along its medial wall (D) Is bounded anteriorly by the sacrotuberous ligament (E) Contains a perineal branch of the fifth lumbar nerve

48. The answer is B. The ischiorectal fossa is bounded posteriorly by the gluteus maximus and the sacrotuberous ligament. It contains fat, the inferior rectal nerve and vessels, and perineal branches of the posterior femoral cutaneous nerve. The pudendal canal runs along its lateral wall. Urine leaking from a ruptured bulb of the penis does not spread into the ischiorectal fossa because Scarpa fascia ends by firm attachment to the fascia lata of the thigh.

48. An emergent hernia repair is scheduled. As the attending physician is driving to the hospital, the medical student assisting on the case quickly reviews his anatomy atlas and is trying to commit to memory that the internal oblique abdominis muscle contributes to the formation of which of the following structures? (A) Inguinal ligament (B) Deep inguinal ring (C) Falx inguinalis (conjoint tendon) (D) Internal spermatic fascia (E) Reflected inguinal ligament

48. The answer is C. The falx inguinalis (conjoint tendon) is formed by the aponeuroses of the internal oblique and transverse muscles of the abdomen. The inguinal ligament is formed by aponeurosis of the external oblique abdominal muscle, and the reflected inguinal ligament is formed by certain fibers of the inguinal ligament reflected from the pubic tubercle upward toward the linea alba. The deep inguinal ring lies in the transversalis fascia, and the internal spermatic fascia is formed by the transversalis fascia.

48. A 59-year-old woman with pain at the side of her skull comes to the emergency department. An emergent head CT scan shows a large lesion in the internal auditory meatus. This condition may progress and damage which of the following pairs of structures? (A) Vagus and glossopharyngeal nerves (B) Internal carotid and vertebral arteries (C) Internal jugular vein and trigeminal nerve (D) Facial and vestibulocochlear nerves (E) Hypoglossal and accessory nerves

48. The answer is D. The internal auditory meatus transmits the facial and vestibulocochlear nerves. The jugular foramen transmits the glossopharyngeal, vagus, and accessory nerves and the internal jugular vein. The ophthalmic, maxillary, and mandibular divisions of the trigeminal nerve run through the superior orbital fissure, foramen rotundum, and foramen ovale, respectively. The hypoglossal nerve runs through the hypoglossal canal.

Questions 48 to 50: A 21-year-old man injures his right arm in an automobile accident. Radiographic examination reveals a fracture of the medial epicondyle of the humerus. 48. Which of the following nerves is most likely injured as a result of this accident? (A) Axillary (B) Musculocutaneous (C) Radial (D) Median (E) Ulnar

48. The answer is E. The ulnar nerve runs down the medial aspect of the arm and behind the medial epicondyle in a groove, where it is vulnerable to damage by fracture of the medial epicondyle. Other nerves are not in contact with the medial epicondyle.

49. A 21-year-old tennis player comes to an emergency room and complains of pain in the knee joint. On examination, he has an infection inside the knee joint capsule but outside the synovial cavity. Which of the following structures is preserved from this infection? (A) Anterior cruciate ligament (B) Posterior cruciate ligament (C) Lateral meniscus (D) Lateral collateral ligament (E) Medial meniscus

49. The Answer is D. The lateral (fibular) collateral ligament extends between the lateral femoral epicondyle and the head of the fibula and is not attached to the lateral meniscus. All other ligaments lie outside the synovial cavity but within the joint capsule.

49. After ingesting a toxic substance found in her friend's home, a 12-year-old girl is unable to close her lips. Which of the following muscles may be paralyzed? (A) Levator labii superioris (B) Zygomaticus minor (C) Orbicularis oris (D) Lateral pterygoid (E) Depressor labii inferioris

49. The answer is C. The lips are closed by the orbicularis oris muscles. The lips are opened by the levator labii superioris, zygomaticus minor, and depressor labii inferioris muscles. The lateral pterygoid muscle can open the mouth by depressing the lower jaw.

49. An elderly man with prostatitis is seen at an internal medicine clinic. The seminal colliculus of his prostate gland is infected, and its fine openings are closed. Which of the following structures is/are most likely to be disturbed? (A) Ducts of the prostate gland (B) Prostatic utricle (C) Ducts of the bulbourethral glands (D) Ejaculatory ducts (E) Duct of the seminal vesicles

49. The answer is D. The ejaculatory ducts, which open onto the seminal colliculus, may be injured. The prostate ducts open into the urethral sinus, the bulbourethral ducts open into the bulbous part of the penile urethra, and the ducts of the seminal vesicle join the ampulla of the ductus deferens to form the ejaculatory duct. The prostatic utricle is a minute pouch on the summit of the seminal colliculus.

49. A 9-year-old girl has crashed into her neighbor's brick fence while riding her bike and is brought to the emergency department with a great deal of abdominal pain. Her radiogram and angiogram show laceration of the superior mesenteric artery immediately distal to the origin of the middle colic artery. If collateral circulation is discounted, which of the following organs may become ischemic? (A) Descending colon (B) Duodenum (C) Pancreas (D) Ascending colon (E) Transverse colon

49. The answer is D. The right colic and ileocolic arteries arise from the superior mesenteric artery distal to the origin of the middle colic artery. The right colic artery may arise from the ileocolic artery and supplies the ascending colon. The duodenum and pancreas receive blood from the inferior pancreaticoduodenal artery and superior pancreaticoduodenal artery. The pancreas is also supplied by the splenic artery of the celiac trunk. The transverse colon receives blood from the middle colic artery. The descending colon is supplied by the left colic artery, which is a branch of the inferior mesenteric artery.

Questions 48 to 50: A 21-year-old man injures his right arm in an automobile accident. Radiographic examination reveals a fracture of the medial epicondyle of the humerus. 49. Which of the following muscles is most likely paralyzed as a result of this accident? (A) Extensor pollicis brevis (B) Abductor pollicis longus (C) Abductor pollicis brevis (D) Adductor pollicis (E) Opponens pollicis

49. The answer is D. The ulnar nerve innervates the adductor pollicis muscle. The radial nerve innervates the abductor pollicis longus and extensor pollicis brevis muscles, whereas the median nerve innervates the abductor pollicis brevis and opponens pollicis muscles

5. A 47-year-old woman is unable to invert her foot after she stumbled on her driveway. Which of the following nerves are most likely injured? (A) Superficial and deep peroneal (B) Deep peroneal and tibial (C) Superficial peroneal and tibial (D) Medial and lateral plantar (E) Obturator and tibial

5. The Answer is B. The deep peroneal and tibial nerves innervate the chief evertors of the foot, which are the tibialis anterior, tibialis posterior, triceps surae, and extensor hallucis longus muscles. The tibialis anterior and extensor hallucis longus muscles are innervated by the deep peroneal nerve, and the tibialis posterior and triceps surae are innervated by the tibial nerve.

5. A 57-year-old woman comes into her physician's office complaining of fever, nausea, vomiting, and the worst headache of her life. Tests and physical examination suggest hydrocephalus (widening ventricles) resulting from a decrease in the absorption of cerebrospinal fluid (CSF). A decrease of flow in the CSF through which of the following structures would be responsible for these findings? (A) Choroid plexus (B) Vertebral venous plexus (C) Arachnoid villi (D) Internal jugular vein (E) Subarachnoid trabeculae

5. The Answer is C. Cerebrospinal fluid (CSF) is absorbed into the venous system primarily through the arachnoid villi projecting into the cranial dural venous sinuses, particularly the superior sagittal sinus. CSF is produced by the choroid plexuses of the ventricles of the brain and is circulated in the subarachnoid space, in which subarachnoid trabeculae are also found. The vertebral venous plexus and internal jugular vein are not involved in the absorption of CSF.

5. A 17-year-old boy is involved in a gang fight and receives a penetrating injury to the neck. Which of the following conditions is most likely exhibited by this misadventure? (A) Internal strabismus (B) Trouble going down the stairs (C) Constricted pupil (D) Inability close the eye (E) Deviation of tongue toward lesion side

5. The Answer is C. Lesion of sympathetic nerves in the cervical region results in a constricted pupil due to paralysis of the dilator pupillae. Internal strabismus is caused by a lesion of the abducens nerve. Lesion of the trochlear nerve results in difficulty going downstairs. Inability to close the eye is due to a lesion of the facial nerve. Lesion of the hypoglossal nerve causes deviation of the tongue toward the lesion side.

5. A 29-year-old carpenter sustains severe injuries of the pelvic splanchnic nerve by a deep puncture wound, which has become contaminated. The injured parasympathetic preganglionic fibers in the splanchnic nerve are most likely to synapse in which of the following ganglia? (A) Ganglia in or near the viscera or pelvic plexus (B) Sympathetic chain ganglia (C) Collateral ganglia (D) Dorsal root ganglia (E) Ganglion impar

5. The answer is A. The pelvic splanchnic nerves carry preganglionic parasympathetic general visceral efferent fibers that synapse in the ganglia of the inferior hypogastric plexus and in terminal ganglia in the muscular walls of the pelvic organs. The sympathetic preganglionic fibers synapse in the sympathetic chain (paravertebral) ganglia or in the collateral (prevertebral) ganglia. The dorsal root ganglia contain cell bodies of general somatic afferent (GSA) and general visceral afferent (GVA) fibers and have no synapsis. The two sympathetic trunks unite and terminate in the ganglion impar (coccygeal ganglion), which is the most inferior, unpaired ganglion located in front of the coccyx.

5. A 42-year-old obese woman with seven children is brought to a local hospital by her daughter. Physical examination and her radiograph reveal that large gallstones have ulcerated through the posterior wall of the fundus of the gallbladder into the intestine. Which of the following parts of the intestine is most likely to initially contain gallstones? (A) Cecum (B) Ascending colon (C) Transverse colon (D) Descending colon (E) Sigmoid colon

5. The answer is C. The fundus of the gallbladder is in contact with the transverse colon, and thus, gallstones erode through the posterior wall of the gallbladder and enter the transverse colon. They are passed naturally to the rectum through the descending colon and sigmoid colon. Gallstones lodged in the body of the gallbladder may ulcerate through the posterior wall of the body of the gallbladder into the duodenum (because the gallbladder body is in contact with the duodenum) and may be held up at the ileocecal junction, producing an intestinal obstruction.

5. A 27-year-old patient presents with an inability to draw the scapula forward and downward because of paralysis of the pectoralis minor. Which of the following would most likely be a cause of his condition? (A) Fracture of the clavicle (B) Injury to the posterior cord of the brachial plexus (C) Fracture of the coracoid process (D) Axillary nerve injury (E) Defects in the posterior wall of the axilla

5. The answer is C. The pectoralis minor inserts on the coracoid process, originates from the second to the fifth ribs, and is innervated by the medial and lateral pectoral nerves that arise from the medial and lateral cords of the brachial plexus. It depresses the shoulder and forms the anterior wall of the axilla. The pectoralis minor has no attachment on the clavicle.

5. A 27-year-old cardiac patient with an irregular heartbeat visits her doctor's office for examination. Where should the physician place the stethoscope to listen to the sound of the mitral valve? (A) Over the medial end of the second left intercostal space (B) Over the medial end of the second right intercostal space (C) In the left fourth intercostal space at the midclavicular line (D) In the left fifth intercostal space at the midclavicular line (E) Over the right half of the lower end of the body of the sternum

5. The answer is D. The mitral valve (left atrioventricular [AV] valve) produces the apical beat (thrust) of the heart, which is most audible over the left fifth intercostal space at the midclavicular line. The pulmonary valve is most audible over the medial end of the second left intercostal space, the aortic valve is most audible over the medial end of the second right intercostal space, and the right AV valve is most audible over the right half of the lower end of the body of the sternum.

5. A 16-year-old patient received a laceration of the posterior intercostal nerves by a penetrated knife blade. A pathologist obtained needle biopsy tissues and observed numerous degenerated cell bodies of the unipolar or pseudounipolar neurons. Which of the following structures would most likely provide the abnormal cell morphology? (A) Ventral horn of the spinal cord (B) Lateral horn of the spinal cord (C) Dorsal horn of the spinal cord (D) Dorsal root ganglion (E) Sympathetic chain ganglion

5. The answer is D. Ventral, lateral, and dorsal horns and sympathetic chain ganglia contain multipolar neurons, whereas the dorsal root ganglion contains unipolar or pseudounipolar neurons. A laceration of the intercostal nerve injures GSE, postganglionic sympathetic general visceral efferent (GVE), general visceral afferent (GVA), and general somatic afferent (GSA) fibers, whose cell bodies are located in the anterior horn, sympathetic chain ganglia, and dorsal root ganglia.

5. A 17-year-old boy receives an injury to the phrenic nerve by a knife wound in the neck. The damaged nerve passes by which of the following structures in the neck? (A) Anterior to the subclavian vein (B) Posterior to the subclavian artery (C) Deep to the brachial plexus (D) Medial to the common carotid artery (E) Superficial to the anterior scalene muscle

5. The answer is E. The phrenic nerve descends on the superficial surface of the anterior scalene muscle and passes into the thorax posterior to the subclavian vein, anterior to the subclavian artery, and lateral to the common carotid artery. The brachial plexus passes deep to the anterior scalene muscle.

50. A 14-year-old gymnastic silver medalist falls from the parallel bar and complains of pains from the knee and ankle joints. On physical examination, her physician found that the muscle responsible for flexing the leg at the knee joint and plantar flexing the foot is severely weakened. Which of the following muscles involved in both movements was most likely damaged in this accident? (A) Tibialis posterior (B) Gastrocnemius (C) Soleus (D) Peroneus longus (E) Flexor digitorum longus

50. The Answer is B. The gastrocnemius can flex the knee joint and also plantar flex the foot. The tibialis posterior can plantar flex and invert the foot. The soleus can plantar flex the foot. The peroneus longus can plantar flex and evert the foot. The flexor digitorum longus can plantar flex the foot and flex the lateral four toes.

50. A 37-year-old man receives a direct blow to his head and is brought to an emergency department. His radiograph shows a fracture of the floor of the middle cranial cavity, causing severance of the greater petrosal nerve. Which of the following conditions could be produced by this injury? (A) Increased lacrimal gland secretion (B) Loss of taste sensation in the epiglottis (C) Dryness in the nose and palate (D) Decreased parotid gland secretion (E) Loss of sensation in the pharynx

50. The answer is C. The greater petrosal nerve carries parasympathetic (preganglionic) fibers, which are secretomotor fibers, to the lacrimal glands and mucous glands in the nasal cavity and palate; carries taste fibers from the palate; and carries GVA fibers from the nasal cavity, palate, and roof of the oral cavity but not from the pharynx and larynx. Therefore, a lesion of the greater petrosal nerve causes dryness in the nose and palate and decreased lacrimal secretion. Decreased parotid gland secretion is due to a lesion of the lesser petrosal nerve. Taste sensation in the epiglottis is carried by the internal laryngeal branch of the superior laryngeal nerve. General visceral sensation in the pharynx is carried by the glossopharyngeal nerve.

50. A 53-year-old woman with known kidney disease presents to a hospital because her pain has become increasingly more severe. A physician performing kidney surgery must remember that: (A) The left kidney lies a bit lower than the right one (B) The perirenal fat lies external to the renal fascia (C) The renal fascia does not surround the suprarenal gland (D) The left renal vein runs anterior to both the aorta and the left renal artery (E) The right renal artery is shorter than the left renal artery

50. The answer is D. The left renal vein runs anterior to both the aorta and the left renal artery. The renal fascia lies external to the perirenal fat and internal to the pararenal fat, and it also surrounds the suprarenal gland. The right renal artery runs behind the IVC and is longer than the left renal artery. Because of the large size of the right lobe of the liver, the right kidney lies a little lower than the left kidney

50. A general surgeon is giving a lecture to a team of surgery residents. She describes characteristics of structures above the pectinate line of the anal canal, which include (A) Stratified squamous epithelium (B) Venous drainage into the caval system (C) Lymphatic drainage into the superficial inguinal nodes (D) Visceral sensory innervation (E) External hemorrhoids

50. The answer is D. The pectinate line is a point of demarcation between visceral and somatic portions of the anal canal. Characteristics above the pectinate line include columnar epithelium, venous drainage into the portal system, lymphatic drainage into the internal iliac nodes, visceral sensory innervation, and internal hemorrhoids.

Questions 48 to 50: A 21-year-old man injures his right arm in an automobile accident. Radiographic examination reveals a fracture of the medial epicondyle of the humerus. 50. After this injury, the patient is unable to do which of the following? (A) Flex the proximal interphalangeal joint of his ring finger (B) Flex the DIP joint of his index finger (C) Feel sensation on his middle finger (D) Abduct his thumb (E) Adduct his index finger

50. The answer is E. The fingers are adducted by the palmar interosseous muscles; abduction is performed by the dorsal interosseous muscles. The palmar and dorsal interosseous muscles are innervated by the ulnar nerve. The proximal interphalangeal joints are flexed by the flexor digitorum superficialis, which is innervated by the median nerve. However, the DIP joints of the index and middle fingers are flexed by the flexor digitorum profundus, which is innervated by the median nerve (except the medial half of the muscle, which is innervated by the ulnar nerve). The median nerve supplies sensory innervation on the palmar aspect of the middle finger. The abductor pollicis brevis is innervated by the median nerve; the abductor pollicis longus is innervated by the radial nerve.

51. A neonatal baby was born with diabetes mellitus due to an inadequate production of insulin. Cells in the endocrine portion of the pancreas that secrete insulin, glucagon, and somatostatin are derived from which of the following? (A) Ectoderm (B) Mesoderm (C) Endoderm (D) Proctodeum (E) Neural crest cells

51. Answer is C. Cells in the islets of Langerhans, an endocrine portion of the pancreas, are derived from the endoderm of the caudal foregut (from the liver diverticulum). Proctodeum is an invagination of the ectoderm of the terminal part of the hindgut.

51. A 28-year-old basketball player falls while rebounding and is unable to run and jump. On physical examination, he has pain and weakness when extending his thigh and flexing his leg. Which muscle involved in both movements is most likely injured? (A) Short head of biceps femoris (B) Adductor magnus (C) Semitendinosus (D) Sartorius (E) Gracilis

51. The Answer is C. The semitendinosus extends the thigh and flexes the leg. The short head of the biceps femoris flexes the leg. The adductor magnus adducts, flexes, and extends the thigh. The sartorius and gracilis can flex the thigh and leg.

Questions 51 to 55: A 10-year-old boy falls off his bike, has difficulty in moving his shoulder, and is brought to the emergency department. His radiograph and angiograph reveal fracture of the surgical neck of his humerus and bleeding from the point of the fracture. 51. Which of the following nerves is most likely injured as a result of this accident? (A) Musculocutaneous (B) Axillary (C) Radial (D) Median (E) Ulnar

51. The answer is B. The axillary nerve runs posteriorly around the surgical neck of the humerus and is vulnerable to injury such as fracture of the surgical neck of the humerus or inferior dislocation of the humerus. The other nerves listed are not in contact with the surgical neck of the humerus.

51. A 5-month-old boy is admitted to the children's hospital because of urine being expelled from the dorsal aspect of the penis. Which of the following embryologic structures failed to fuse in this patient? (A) Labioscrotal swellings or folds (B) Urogenital sinus (C) Spongy urethra (D) Phallus (E) Urethral folds

51. The answer is C. A developmental defect in the spongy urethra results in epispadias, causing the patient to pass urine through an opening on the dorsum of the penis. Labioscrotal swellings form the scrotum in males and the labia majora in females. Urogenital sinus forms the urinary bladder, urethra, prostate, and bulbourethral glands in males, and the bladder, urethra, lower vagina, and greater vestibular glands in females. The phallus (genital tubercle) forms the penis in males and the clitoris in females. Urethral (urogenital) folds form the spongy urethra and a portion of the shaft of the penis in males and the labia minora in females.

51. A 65-year-old man with multiple vision problems comes to the local eye clinic. The pupillary light reflex can be eliminated by cutting which of the following nerves? (A) Short ciliary, ophthalmic, and oculomotor nerves (B) Long ciliary, optic, and short ciliary nerves (C) Oculomotor, short ciliary, and optic nerves (D) Optic and long ciliary nerves and ciliary ganglion (E) Ophthalmic and optic nerves and ciliary ganglion

51. The answer is C. The efferent limbs of the reflex are involved in the pupillary light reflex (i.e., constriction of the pupil in response to illumination of the retina) and are composed of parasympathetic preganglionic fibers in the oculomotor nerve, parasympathetic fibers and ganglionic cells in the ciliary ganglion, and parasympathetic postganglionic fibers in the short ciliary nerves. The afferent limbs of this reflex are optic nerve fibers. The long ciliary nerves contain postganglionic sympathetic fibers. The ophthalmic nerve contains GSA fibers.

52. During development, the midgut artery appears to be markedly narrowed at its origin. Which of the following structures is derived from the midgut and may receive inadequate blood supply? (A) Gallbladder (B) Stomach (C) Descending colon (D) Ascending colon (E) Rectum

52. Answer is D. The ascending colon is derived from the midgut. The gallbladder and stomach are derived from the foregut, and the descending colon and rectum are derived from the hindgut.

52. A 52-year-old woman comes to an orthopedic surgeon complaining of an uncomfortable feeling in her knee and ankle joints. After a thorough examination, she is diagnosed as having arthritis with mild discomfort with passive movements. The muscles acting at the ankle joint appear normal with good strength. Which muscle can both dorsiflex and invert her foot? (A) Peroneus longus (B) Peroneus brevis (C) Peroneus tertius (D) Extensor hallucis longus (E) Extensor digitorum longus

52. The Answer is D. The extensor hallucis longus can dorsiflex and invert the foot. The peroneus longus, peroneus tertius, and extensor digitorum longus can dorsiflex and evert foot. The peroneus brevis can plantar flex and evert the foot.

52. A 78-year-old man has carcinoma of the rectum. The cancer is likely to metastasize via the veins into which of the following structures? (A) Spleen (B) Kidney (C) Liver (D) Duodenum (E) Suprarenal gland

52. The answer is C. Cancer cells from rectal cancer are likely to metastasize to the liver via the superior rectal, inferior mesenteric, splenic, and portal veins. Cancer cells are not directly spread to the other organs listed. The spleen and duodenum drain their venous blood to the portal venous system, and the kidney and suprarenal gland empty into the caval (inferior vena cava) system.

Questions 51 to 55: A 10-year-old boy falls off his bike, has difficulty in moving his shoulder, and is brought to the emergency department. His radiograph and angiograph reveal fracture of the surgical neck of his humerus and bleeding from the point of the fracture. 52. Following this accident, the damaged nerve causes difficulty in abduction, extension, and lateral rotation of his arm. Cell bodies of the injured nerve involved in movement of his arm are located in which of the following structures? (A) Dorsal root ganglion (B) Sympathetic chain ganglion (C) Anterior horn of the spinal cord (D) Lateral horn of the spinal cord (E) Posterior horn of the spinal cord

52. The answer is C. The (injured) axillary nerve contains GSE fibers whose cell bodies are located in the anterior horn of the spinal cord, and these GSE fibers supply the deltoid and teres minor muscles. The axillary nerve also contains GSA and GVA fibers, whose cell bodies are located in the dorsal root ganglia, and sympathetic postganglionic fibers, whose cell bodies are located in sympathetic chain ganglia. The lateral horn of the spinal cord between T1 and L2 contains cell bodies of sympathetic preganglionic fibers. The posterior horn of the spinal cord contains cell bodies of interneurons.

52. A 12-year-old boy has difficulty in breathing because he is choking on food. A school nurse performs the Valsalva maneuver to expel air from his lungs and thus dislodge the food. When that fails, she performs a needle cricothyrotomy, which would open into which of the following regions? (A) Rima glottidis (B) Laryngeal vestibule (C) Laryngeal ventricle (D) Infraglottic cavity (E) Piriform recess

52. The answer is D. The infraglottic cavity extends from the rima glottidis to the lower border of the cricoid cartilage. The rima glottidis is the space between the vocal folds and arytenoid cartilages. The vestibule extends from the laryngeal inlet to the vestibular folds. The ventricle extends between the vestibular fold and the vocal fold. The piriform recess is a pear-shaped fossa in the wall of the laryngopharynx lateral to the arytenoid cartilage.

53. A 3-year-old boy is admitted to the children's hospital with complaints of restlessness, abdominal pain, and fever. An MRI examination reveals that he has a double ureter. Which of the following embryonic structures is most likely failed to develop normally? (A) Mesonephric (Wolffian) duct (B) Paramesonephric (Müllerian) duct (C) Ureteric bud (D) Metanephros (E) Pronephros

53. Answer is C. The ureteric bud is an outgrowth of the mesonephric duct and develops into the ureter, renal pelvis, calyces, and collecting tubules. However, a bifurcated ureteric bud results in a partial duplication (bifid) of the ureter, whereas two separate ureteric buds result in a complete duplication. Mesonephric duct forms efferent ductules, epididymal duct, ductus deferens, ejaculatory duct, and seminal vesicles. Paramesonephric duct regress and its vestigial remnants form the appendix testis. Metanephros develops into the adult kidney. Pronephros degenerates and never forms functional nephrons.

53. A 59-year-old stroke patient is unable to swallow because of a nerve injury. Which of the following nerves is unaffected? (A) Hypoglossal nerve (B) Spinal accessory nerve (C) Vagus nerve (D) Facial nerve (E) Trigeminal nerve

53. The answer is B. The spinal accessory nerve supplies the sternocleidomastoid and trapezius muscles, which are not involved in the act of swallowing. Swallowing involves movements of the tongue to push the food into the oropharynx, elevation of the soft palate to close the entrance of the nasopharynx, elevation of the hyoid bone and the larynx to close the opening into the larynx, and contraction of the pharyngeal constrictors to move the food through the pharynx. The hypoglossal nerve supplies all of the tongue muscles except the palatoglossus, which is innervated by the vagus nerve. The vagus nerve innervates the muscles of the palate, larynx, and pharynx. The mandibular division of the trigeminal nerve supplies the suprahyoid muscles (e.g., the anterior belly of the digastric and the mylohyoid muscles).

53. During a partial hysterectomy leaving the ovaries intact, the surgeon detaches the ovary from the uterus by transecting the ovarian ligament. This ligament developed from which embryonic structure? (A) Mesonephric duct (B) Urogenital folds (C) Gubernaculum (D) Processus vaginalis (E) Paramesonephric ducts

53. The answer is C. The ovarian ligament and the round ligament of the uterus are formed by the gubernaculum. The mesonephric duct gives rise only to the vestigial epoophoron in the female. The urogenital folds form the labia minora. The processus vaginalis forms no adult female structures, while the paramesonephric ducts form the uterine tubes, uterus, cervix, and upper vagina.

Questions 51 to 55: A 10-year-old boy falls off his bike, has difficulty in moving his shoulder, and is brought to the emergency department. His radiograph and angiograph reveal fracture of the surgical neck of his humerus and bleeding from the point of the fracture. 53. The damaged nerve causes numbness of the lateral side of the arm. Cell bodies of the injured nerve fibers involved in sensory loss are located in which of the following structures? (A) Anterior horn of the spinal cord (B) Posterior horn of the spinal cord (C) Lateral horn of the spinal cord (D) Dorsal root ganglia (E) Sympathetic chain ganglia

53. The answer is D. Axillary nerve contains GSE, GSA, GVA, and sympathetic postganglionic GVE fibers. Cell bodies of GSA and GVA fibers are located in the dorsal root ganglia. Cell bodies of GSE fibers are located in the anterior horn of the spinal cord. Cell bodies of sympathetic postganglionic GVE fibers are located in the sympathetic chain ganglia, but cell bodies of sympathetic preganglionic GVE fibers lie in the lateral horn of the spinal cord.

54. A neonate has a small reducible protrusion through a defined ring at the umbilicus. His pediatrician indicates to the parents that this will likely close spontaneously. Which of the following congenital malformations is present? (A) Umbilical hernia (B) Symptomatic patent urachus (C) Patent omphalomesenteric duct (D) Omphalocele (E) Gastroschisis

54. The answer is A. In most case, an umbilical hernia closes spontaneously by age 4 and requires no surgery unless there is incarceration. A symptomatic patent urachus (drainage of urine at the umbilicus) is typically surgically excised. A patent omphalomesenteric duct (Meckel diverticulum) is promptly repaired to minimize the potential for intestinal obstruction or prolapse. Omphalocele and gastroschisis are defects that require surgical repair.

54. A 2-year-boy presents with midfacial and mandibular hypoplasia, cleft palate, deformed external ear, and defect in hearing. Which of the following embryonic structures is most likely developed abnormally? (A) First pharyngeal arch (B) Second pharyngeal arch (C) Third pharyngeal arch (D) Fourth pharyngeal arch (E) Sixth pharyngeal arch

54. The answer is A. The patient's abnormal appearance results from abnormal development of the first pharyngeal arch because the first pharyngeal arch develops into muscles of mastication, mylohyoid, digastric anterior belly, tensor veli palatini, tensor tympani, maxilla, mandible, malleus, incus, zygomatic bone, temporal bone, palatine bone, vomer, and sphenomandibular ligament.

Questions 51 to 55: A 10-year-old boy falls off his bike, has difficulty in moving his shoulder, and is brought to the emergency department. His radiograph and angiograph reveal fracture of the surgical neck of his humerus and bleeding from the point of the fracture. 54. This accident most likely leads to the damage of which of the following arteries? (A) Axillary (B) Deep brachial (C) Posterior humeral circumflex (D) Superior ulnar collateral (E) Scapular circumflex

54. The answer is C. The posterior humeral circumflex artery accompanies the axillary nerve that passes around the surgical neck of the humerus. None of the other arteries are involved.

Questions 51 to 55: A 10-year-old boy falls off his bike, has difficulty in moving his shoulder, and is brought to the emergency department. His radiograph and angiograph reveal fracture of the surgical neck of his humerus and bleeding from the point of the fracture. 55. Following this accident, the boy has weakness in rotating his arm laterally. Which of the following muscles are paralyzed? (A) Teres major and teres minor (B) Teres minor and deltoid (C) Infraspinatus and deltoid (D) Supraspinatus and subscapularis (E) Teres minor and infraspinatus

55. The answer is B. The lateral rotators of the arm include the teres minor, deltoid, and infraspinatus muscles, but the infraspinatus muscle is innervated by the suprascapular nerve.

55. A 64-year-old woman is unable to open her mouth or jaw because of tetanus resulting from a penetrating wound from a rusty nail. Which of the following muscles would most likely be paralyzed? (A) Masseter muscle (B) Medial pterygoid muscle (C) Lateral pterygoid muscle (D) Buccinator muscle (E) Temporalis muscle

55. The answer is C. The lateral pterygoid muscle opens the mouth by depressing the jaw. The masseter, medial pterygoid, and temporalis muscles close the jaw. The buccinator muscle is a muscle of facial expression.

Questions 56 to 57: A 64-year-old man with a history of liver cirrhosis has been examined for hepatitis A, B, and C viruses. In an attempt to obtain a blood sample from the patient's median cubital vein, a registered nurse inadvertently procures arterial blood. 56. The blood most likely comes from which of the following arteries? (A) Brachial (B) Radial (C) Ulnar (D) Common interosseous (E) Superior ulnar collateral

56. The answer is A. The median cubital vein lies superficial to the bicipital aponeurosis and thus separates it from the brachial artery, which can be punctured during intravenous injections and blood transfusions.

56. A 60-year-old man is unable to open his eye because of a rare neuromuscular disease. Which of the following muscles would most likely be paralyzed? (A) Orbicularis oculi (B) Orbicularis oris (C) Frontalis (D) Levator palpebrae superioris (E) Superior rectus

56. The answer is D. The levator palpebrae superioris muscle opens the eye by elevating the upper eyelid. The orbicularis oculi closes the eye, the orbicularis oris closes the lips, the frontalis elevates the eyebrow, and the superior rectus elevates the eyeball.

57. A 46-year-old man visits the speech therapist complaining of dryness of the mouth. The therapist performs a swallowing study and, on examination, finds that the man has a lack of salivary secretion from the submandibular gland. This indicates a lesion of which of the following nervous structures? (A) Lingual nerve at its origin (B) Chorda tympani in the middle ear cavity (C) Superior cervical ganglion (D) Lesser petrosal nerve (E) Auriculotemporal nerve

57. The answer is B. The chorda tympani nerve contains preganglionic parasympathetic fibers responsible for secretion of the submandibular gland. The lingual nerve at its origin is not yet joined by the chorda tympani. The superior cervical ganglion provides sympathetic fibers, which supply blood vessels in the submandibular gland. The lesser petrosal nerve contains preganglionic parasympathetic fibers that synapse in the otic ganglion. The auriculotemporal nerve contains postganglionic parasympathetic fibers, which are responsible for secretion of the parotid gland.

Questions 56 to 57: A 64-year-old man with a history of liver cirrhosis has been examined for hepatitis A, B, and C viruses. In an attempt to obtain a blood sample from the patient's median cubital vein, a registered nurse inadvertently procures arterial blood. 57. During the procedure, the needle hits a nerve medial to the artery. Which of the following nerves is most likely damaged? (A) Radial (B) Median (C) Ulnar (D) Lateral antebrachial (E) Medial antebrachial

57. The answer is B. The median nerve is damaged because it lies medial to the brachial artery. The bicipital aponeurosis lies on the brachial artery and the median nerve. The V-shaped cubital fossa contains (from medial to lateral) the median nerve, brachial artery, biceps tendon, and radial nerve. The ulnar nerve runs behind the medial epicondyle; the lateral and medial antebrachial cutaneous nerves are not closely related to the brachial artery.

Questions 58 to 62: A 17-year-old boy is injured in an automobile accident. He has a fracture of the shaft of the humerus 58. Which of the following nerves is most likely damaged? (A) Axillary nerve (B) Radial nerve (C) Musculocutaneous nerve (D) Median nerve (E) Ulnar nerve

58. The answer is B. The radial nerve runs in the radial groove on the back of the shaft of the humerus with the profunda brachii artery. The axillary nerve passes around the surgical neck of the humerus. The ulnar nerve passes the back of the medial epicondyle. The musculocutaneous and median nerves are not in contact with the bone, but the median nerve can be damaged by supracondylar fracture.

58. A 51-year-old woman traveling through British Columbia can see the beautiful blue sky with white clouds but is unable to focus on her face in the mirror. Her lack of accommodation results from paralysis of which of the following muscles? (A) Tarsal muscle (B) Sphincter pupillae (C) Dilator pupillae (D) Ciliary muscles (E) Orbitalis muscles

58. The answer is D. Near focus (accomodation) occurs with contraction of the ciliary muscles and is mediated by parasympathetic fibers running within the oculomotor nerve. The levator palpebrae superioris inserts on the tarsal smooth muscle plate in the upper eyelid and skin of the upper eyelid and opens the eye by elevating the upper eyelid. The sphincter pupillae and dilator pupillae constrict and dilate the pupil, respectively. The orbitalis muscle is a smooth muscle that bridges the inferior orbital fissure and protrudes the eye.

59. A 3-year-old girl is admitted to the hospital with pain and hearing defect. An MRI examination reveals that she has developmental defects in the auditory tube and middle ear cavity. Which of the following pharyngeal pouches is most likely developed abnormally? (A) First pouch (B) Second pouch (C) Third pouch (D) Fourth pouch (E) Second and fourth pouches

59. The answer is A. The first pharyngeal pouch gives rise to the auditory tube and middle ear cavity. The second pouch forms the palatine tonsils. The third pouch gives rise to the inferior parathyroid gland and thymus. The fourth pouch develops into the superior parathyroid gland and ultimobranchial body of the thyroid.

Questions 58 to 62: A 17-year-old boy is injured in an automobile accident. He has a fracture of the shaft of the humerus 59. As a result of this fracture, the patient shows lack of sweating on the back of the arm and forearm. Cell bodies of the damaged nerve fibers involved in sweating are located in which of the following structures? (A) Anterior horn of the spinal cord (B) Posterior horn of the spinal cord (C) Lateral horn of the spinal cord (D) Sympathetic chain ganglion (E) Dorsal root ganglion

59. The answer is D. The (damaged) radial nerve contains sympathetic postganglionic nerve fibers whose cell bodies are located in the sympathetic chain ganglion. Sympathetic postganglionic fibers supply sweat glands, blood vessels, and hair follicles (arrector pili muscles). The radial nerve also contains GSE fibers whose cell bodies are located in the anterior horn of the spinal cord, and GSA and GVA fibers whose cell bodies are located in the dorsal root ganglion. The lateral horn of the spinal cord between T1 and L2 contains cell bodies of sympathetic preganglionic nerve fibers.

6. After a 26-year-old man's car was broadsided by a large truck, he is brought to the emergency department with multiple fractures of the transverse processes of the cervical and upper thoracic vertebrae. Which of the following muscles might be affected? (A) Trapezius (B) Levator scapulae (C) Rhomboid major (D) Serratus posterior superior (E) Rectus capitis posterior major

6. The Answer is B. The levator scapulae arise from the transverse processes of the upper cervical vertebrae and inserts on the medial border of the scapula. The other muscles are attached to the spinous processes of the vertebrae.

6. A 28-year-old woman comes to her family physician and complains of difficulty in swallowing. Further examination reveals that she has no taste sensation of the posterior one-third of her tongue and a lack of secretion of the parotid gland. Which of the following would most likely cause this condition? (A) Fracture of the mandibular canal (B) Section of the zygomatic nerve (C) Glossopharyngeal nerve injury (D) Tumor in the pituitary gland (E) Lesion of the hypoglossal nerve

6. The Answer is C. Injury to the glossopharyngeal nerve causes paralysis of the stylopharoyngeus muscle, which is involved in swallowing, no general and taste sensation of the posterior onethird of the tongue, a lack of salivary secretion from the parotid gland due to parasympathetic nerve injury, and no visceral sensation from the carotid sinus and body. Lesion of the inferior alveolar nerve in the mandibular canal results in a lack of sensation to the canine and incisor teeth and the skin over the chin. Section of the zygomatic nerve causes a lack of lacrimal secretion because it carries postganglionic parasympathetic fibers from the pterygopalatine ganglion for lacrimal secretion. Tumor in the pituitary gland may damage the optic chiasma, resulting in the bitemporal hemianopia. Lesion of the hypoglossal nerve causes deviation of the tongue toward the injured side on protrusion.

6. A 22-year-old patient is unable to "unlock" the knee joint to permit flexion of the leg. Which of the following muscles is most likely damaged? (A) Rectus femoris (B) Semimembranosus (C) Popliteus (D) Gastrocnemius (E) Biceps femoris

6. The Answer is C. The popliteus muscle rotates the femur laterally ("unlocks" the knee) or rotates the tibia medially, depending on which bone is fixed. This action results in unlocking of the knee joint to initiate flexion of the leg at the joint. The rectus femoris flexes the thigh and extends the knee. The gastrocnemius flexes the knee and plantar flexes the foot. The semimembranosus extends the thigh and flexes and rotates the leg medially. The biceps femoris extends the thigh and flexes and rotates the leg laterally.

6. A 45-year-old woman is suffering from numbness over the tip of her nose. Which of the following nerves is most likely to be damaged? (A) Ophthalmic division of the trigeminal nerve (B) Maxillary division of the trigeminal nerve (C) Mandibular division of the trigeminal nerve (D) Facial nerve (E) Auriculotemporal nerve

6. The answer is A. The skin over the tip of the nose is innervated by the external nasal branch of the nasociliary branch of the ophthalmic division of the trigeminal nerve. The maxillary division of the trigeminal nerve innervates the skin of the face above the upper lip but below the lower eyelid. The mandibular division of the trigeminal nerve supplies the lower part of the face below the lower lip. The facial nerve provides no cutaneous sensation on the face but innervates muscles of facial expression. The auriculotemporal nerve is a branch of the mandibular division of the trigeminal nerve and innervates the skin of the auricle and the scalp.

6. A 19-year-old college student came to his doctor's office for a neurologic examination. His physician told him that normally synapses are absent in or on which of the following structures? (A) Anterior horn of the spinal cord (B) Dorsal root ganglia (C) Sympathetic chain ganglia (D) Dendrites (E) Cell bodies

6. The answer is B. Dorsal root ganglia consist of cell bodies of the unipolar or pseudounipolar neurons and have no synapses. Axosomatic and axodendritic synapses are the most common, but axoaxonal and dendrodendritic contacts are also found in many nerve tissues.

6. A 19-year-old man came to the emergency department, and his angiogram exhibited that he was bleeding from the vein that is accompanied by the posterior interventricular artery. Which of the following veins is most likely to be ruptured? (A) Great cardiac vein (B) Middle cardiac vein (C) Anterior cardiac vein (D) Small cardiac vein (E) Oblique veins of the left atrium

6. The answer is B. The middle cardiac vein ascends in the posterior interventricular groove, accompanied by the posterior interventricular branch of the right coronary artery. The great cardiac vein is accompanied by the anterior interventricular artery, the anterior cardiac vein drains directly into the right atrium, and the small cardiac vein is accompanied by the marginal artery

6. A 35-year-old woman comes to a local hospital with abdominal tenderness and acute pain. On examination, her physician observes that an abdominal infection has spread retroperitoneally. Which of the following structures is most likely affected? (A) Stomach (B) Transverse colon (C) Jejunum (D) Descending colon (E) Spleen

6. The answer is D. The descending colon is a retroperitoneal organ. The rest of the organs are surrounded by peritoneum.

6. A 22-year-old patient received a stab wound in the chest that injured the intercostobrachial nerve. Which of the following conditions results from the described lesion of the nerve? (A) Inability to move the ribs (B) Loss of tactile sensation on the lateral aspect of the arm (C) Absence of sweating on the posterior aspect of the arm (D) Loss of sensory fibers from the second intercostal nerve (E) Damage to the sympathetic preganglionic fibers

6. The answer is D. The intercostobrachial nerve arises from the lateral cutaneous branch of the second intercostal nerve and pierces the intercostal and serratus anterior muscles. It may communicate with the medial brachial cutaneous nerve, and it supplies skin on the medial side of the arm. It contains no skeletal motor fibers but does contain sympathetic postganglionic fibers, which supply sweat glands.

6. A 59-year-old woman comes to a local hospital for uterine cancer surgery. As the uterine artery passes from the internal iliac artery to the uterus, it crosses superior to which of the following structures that is sometimes mistakenly ligated during such surgery? (A) Ovarian artery (B) Ovarian ligament (C) Uterine tube (D) Ureter (E) Round ligament of the uterus

6. The answer is D. The ureter runs under the uterine artery near the cervix; thus, the ureter is sometimes mistakenly ligated during pelvic surgery. The other structures mentioned are not closely related to the uterine artery near the uterine cervix.

60. A 42-year-old patient has an aneurysm at the junction of the posterior cerebral and posterior communicating arteries that has compressed a nerve. On the affected side, the patient is most likely to exhibit: (A) Bitemporal hemianopsia (B) A dilated pupil (C) A medially deviated eye (D) A constricted pupil (E) Anosmia

60. The answer is B. An aneurysm at the junction of the posterior communicating and posterior cerebral arteries compresses parasympathetic nerve fibers of the oculomotor nerve, causing a dilated pupil. Bitemporal hemianopsia is caused by the pituitary tumor that compresses the optic chiasma. A medially deviated eye (internal strabismus) is caused by a paralysis of the lateral rectus due to an injury to the abducens nerve. A constricted pupil is caused by a damage of the cervical sympathetic nerve fibers to the dilator muscle of the pupil. An anosmia is caused by an injury to the olfactory nerves due to a fracture of the cribriform plate.

Questions 58 to 62: A 17-year-old boy is injured in an automobile accident. He has a fracture of the shaft of the humerus 60. Following this accident, the patient has no cutaneous sensation in which of the following areas? (A) Medial aspect of the arm (B) Lateral aspect of the forearm (C) Palmar aspect of the second and third digits (D) Area of the anatomic snuffbox (E) Medial one and one-half fingers

60. The answer is D. The superficial branch of the radial nerve runs distally to the dorsum of the hand to innervate the radial side of the hand, including the area of the anatomic snuffbox and the radial two and one-half digits over the proximal phalanx. The medial aspect of the arm is innervated by the medial brachial cutaneous nerve; the lateral aspect of the forearm is innervated by the lateral antebrachial cutaneous nerve of the musculocutaneous nerve; the palmar aspect of the second and third digits is innervated by the median nerve; and the medial one and one-half fingers are innervated by the ulnar nerve

Questions 58 to 62: A 17-year-old boy is injured in an automobile accident. He has a fracture of the shaft of the humerus 61. Which of the following arteries may be damaged? (A) Brachial artery (B) Posterior humeral circumflex artery (C) Profunda brachii artery (D) Radial artery (E) Radial recurrent artery

61. The answer is C. The radial nerve accompanies the profunda brachii artery in the radial groove on the posterior aspect of the shaft of the humerus. The posterior humeral circumflex artery accompanies the axillary nerve around the surgical neck of the humerus. Other arteries are not associated with the radial groove of the humerus.

Questions 58 to 62: A 17-year-old boy is injured in an automobile accident. He has a fracture of the shaft of the humerus. 62. After this accident, supination is still possible through contraction of which of the following muscles? (A) Supinator (B) Pronator teres (C) Brachioradialis (D) Biceps brachii (E) Supraspinatus

62. The answer is D. A lesion of the radial nerve causes paralysis of the supinator and brachioradialis. The biceps brachii muscle is a flexor of the elbow and also a strong supinator; thus, supination is still possible through action of the biceps brachii muscle. Other muscles cannot supinate the forearm.

63. A cyclist is thrown over his handle bars and breaks his clavicle as he hits the ground on his shoulder. Which of the following is correct regarding the development of the clavicle? (A) It develops through intramembranous ossification (B) It is the last upper limb bone to begin ossification (C) The clavicle forms from somitic mesoderm (D) It is the first limb bone to complete ossification (E) Associated muscles form from somatic lateral plate mesoderm

63. The answer is A. The clavicle is the only upper limb bone to develop through intramembranous ossification. It is the first upper limb bone to begin ossification and is the last bone to complete ossification. The clavicle forms from somatic lateral plate mesoderm and the associated muscles develop from somitic mesoderm (from somites).

64. A patient presents with pain in the neck, numbness and tingling in the fingers, and a week grip. This presentation suggests thoracic outlet syndrome. Which of the following causes has an embryological etiology? (A) Traumatic injury (B) Cardiovascular disease (C) Cervical rib (D) Pancoast tumor (E) Scalene muscle inflammation

64. The answer is C. A cervical rib is a congenital abnormality that can cause thoracic outlet syndrome because the space that transmits the neurovasculature to the arm is reduced. Other causes listed are not a result of embryological development.

7. A 29-year-old woman is admitted to a hospital because the birth of her child is several days overdue. Tearing of the pelvic diaphragm during childbirth leads to paralysis of which of the following muscles? (A) Piriformis (B) Sphincter urethrae (C) Obturator internus (D) Levator ani (E) Sphincter ani externus

7. The answer is D. The pelvic diaphragm is formed by the levator ani and coccygeus, whereas the urogenital diaphragm consists of the sphincter urethrae and deep transverse perinei muscles. The piriformis passes through the greater sciatic notch and inserts on the greater trochanter of the femur. The obturator internus forms the lateral wall of the ischiorectal fossa. The sphincter ani externus is composed of three layers, including the subcutaneous (corrugator cutis ani), superficial, and deep portions, and maintains a voluntary tonic contracture.

7. A patient presents with sensory loss on adjacent sides of the great and second toes and impaired dorsiflexion of the foot. These signs probably indicate damage to which of the following nerves? (A) Superficial peroneal (B) Lateral plantar (C) Deep peroneal (D) Sural (E) Tibial

7. The Answer is C. The deep peroneal nerve supplies the anterior muscles of the leg, including the tibialis anterior, extensor hallucis longus, extensor digitorum longus, and peroneus tertius muscles, which dorsiflex the foot. The medial branch of the deep peroneal nerve supplies the skin of adjacent sides of the great and second toes, whereas the lateral branch supplies the extensor digitorum brevis and extensor hallucis brevis. The superficial peroneal nerve innervates the peroneus longus and brevis, which plantar flexes the foot, and supplies the skin on the side of the lower leg and the dorsum of the ankle and foot. The tibial nerve innervates the muscles of the posterior compartment that plantar flexes and supplies the skin on the heel and plantar aspect of the foot. The lateral plantar nerve innervates muscles and skin of the lateral plantar aspect of the foot. The sural nerve supplies the skin on the posterolateral aspect of the leg and the lateral aspect of the foot and the little toe.

7. A 27-year-old mountain climber falls from a steep rock wall and is brought to the emergency department. His physical examination and computed tomography (CT) scan reveal dislocation fracture of the upper thoracic vertebrae. The fractured body of the T4 vertebra articulates with which of the following parts of the ribs? (A) Head of the third rib (B) Neck of the fourth rib (C) Tubercle of the fourth rib (D) Head of the fifth rib (E) Tubercle of the fifth rib

7. The Answer is D. The body of vertebra T4 articulates with the heads of the fourth and fifth ribs. The body of the T3 vertebra articulates with the head of the third and fourth ribs. The neck of a rib does not articulate with any part of the vertebra. The transverse process of the vertebra articulates with the tubercle of the corresponding rib. Therefore, the transverse process of vertebra T4 articulates with the tubercle of the fourth rib.

7. A 16-year-old boy fell from a motorcycle, and his radial nerve was severely damaged because of a fracture of the midshaft of the humerus. Which of the following conditions would most likely result from this accident? (A) Loss of wrist extension leading to wrist drop (B) Weakness in pronating the forearm (C) Sensory loss over the ventral aspect of the base of the thumb (D) Inability to oppose the thumb (E) Inability to abduct the fingers

7. The answer is A. Injury to the radial nerve results in loss of wrist extension, leading to wrist drop. The median nerve innervates the pronator teres, pronator quadratus, and opponens pollicis muscles and the skin over the ventral aspect of the thumb. The ulnar nerve innervates the dorsal interosseous muscles, which act to abduct the fingers.

7. A 26-year-old singer visits her physician—an ear, nose, and throat (ENT) surgeon—and complains of changes in her voice. A laryngoscopic examination demonstrates a lesion of the superior laryngeal nerve, causing weakness of which of the following muscles? (A) Inferior pharyngeal constrictor (B) Middle pharyngeal constrictor (C) Superior pharyngeal constrictor (D) Thyroarytenoid (E) Thyrohyoid

7. The answer is A. The external laryngeal branch of the superior laryngeal nerve supplies the cricothyroid and inferior pharyngeal constrictor muscles. The superior, middle, and inferior pharyngeal constrictors are innervated by the vagus nerve through the pharyngeal plexus. The recurrent (or inferior) laryngeal nerve supplies the thyroarytenoid muscle and the C1 via the hypoglossal nerve supplies the thyrohyoid muscle.

7. During an annual health examination of a 46-year-old woman, a physician finds hypersecretion of norepinephrine from her suprarenal medulla. Which of the following types of nerve fibers are most likely overstimulated? (A) Preganglionic sympathetic fibers (B) Postganglionic sympathetic fibers (C) Somatic motor fibers (D) Postganglionic parasympathetic fibers (E) Preganglionic parasympathetic fibers

7. The answer is A. The suprarenal medulla is the only organ that receives preganglionic sympathetic fibers. No other nerve fibers are involved in secretion of norepinephrine from the suprarenal medulla.

7. A 27-year-old woman involved in a car accident is brought into the emergency department. Her magnetic resonance imaging reveals that she has a laceration of the spinal cord at the L4 spinal cord level. Which of the following structures would you expect to be intact? (A) Dorsal horn (B) Lateral horn (C) Ventral horn (D) Gray matter (E) White matter

7. The answer is B. The lateral horns are found in the gray matter of the spinal cord between T1 and L2 and also between S2 and S4. Therefore, the lateral horns are absent at the L4 spinal cord level.

7. A 37-year-old patient with palpitation was examined by her physician, and one of the diagnostic records included a posterior-anterior chest radiograph. Which of the following comprises the largest portion of the sternocostal surface of the heart seen on the radiograph? (A) Left atrium (B) Right atrium (C) Left ventricle (D) Right ventricle (E) Base of the heart

7. The answer is D. The right ventricle forms a large part of the sternocostal surface of the heart. The left atrium occupies almost the entire posterior surface of the right atrium. The right atrium occupies the right aspect of the heart. The left ventricle lies at the back of the heart and bulges roundly to the left. The base of the heart is formed by the atria, which lie mainly behind the ventricles.

7. A 34-year-old man in a bar fight suffers a knife wound that severs the abducens nerve proximal to its entrance into the orbit. Which of the following conditions results from this injury? (A) Ptosis of the upper eyelid (B) Loss of the ability to dilate the pupil (C) External strabismus (lateral deviation) (D) Loss of visual accommodation (E) Internal strabismus (medial deviation)

7. The answer is E. The abducens nerve (CN VI) innervates the lateral rectus muscle, which abducts the eyeball. A lesion of the abducens nerve results in internal strabismus (medial deviation) and diplopia (double vision). Ptosis of the upper eyelid is caused by lesions of the oculomotor nerve or sympathetic nerve to the levator palpebrae superioris. Inability to dilate the pupil is caused by a lesion of the sympathetic nerve to the dilator pupillae. External strabismus (lateral deviation) is caused by paralysis of the medial rectus muscle, which is innervated by the oculomotor nerve. Loss of visual accommodation is due to a lesion of parasympathetic nerve fibers to the ciliary muscle.

8. A motorcyclist falls from his bike in an accident and gets a deep gash that severs the superficial peroneal nerve near its origin. Which of the following muscles is paralyzed? (A) Peroneus longus (B) Extensor hallucis longus (C) Extensor digitorum longus (D) Peroneus tertius (E) Extensor digitorum brevis

8. The Answer is A. The superficial peroneal nerve supplies the peroneus longus and brevis muscles. Other muscles are innervated by the deep peroneal nerve. .

8. A young toddler presents to her pediatrician with rather new onset of bowel and bladder dysfunction and loss of the lower limb function. Her mother had not taken enough folic acid (to the point of a deficiency) during her pregnancy. On examination, the child has protrusion of the spinal cord and meninges and is diagnosed with which of the following conditions? (A) Spina bifida occulta (B) Meningocele (C) Meningomyelocele (D) Myeloschisis (E) Syringomyelocele

8. The Answer is C. Meningomyelocele is protrusion of the meninges and spinal cord through the unfused arch of the vertebra. A sufficient amount of folic acid during pregnancy is shown to prevent these kinds of neural tube defects. Spina bifida occulta is failure of the vertebral arch to fuse (only bony defect). Meningocele is protrusion of the meninges through the defective vertebral arch. Syringomyelocele is protrusion of the meninges and a pathologic tubular cavity in the spinal cord or brain.

8. A 44-year-old man with "crocodile tears syndrome" has spontaneous lacrimation during eating because of misdirection of regenerating autonomic nerve fibers. Which of the following nerves has been injured? (A) Facial nerve proximal to the geniculate ganglion (B) Auriculotemporal nerve (C) Chorda tympani in the infratemporal fossa (D) Facial nerve at the stylomastoid foramen (E) Lacrimal nerve

8. The answer is A. "Crocodile tears syndrome" (lacrimation during eating) is caused by a lesion of the facial nerve proximal to the geniculate ganglion resulting from misdirection of regenerating parasympathetic fibers, which formerly innervated the salivary glands, to the lacrimal glands. An injury to the auriculotemporal nerve may result in Frey syndrome (sweating while eating), which results from misdirection of regenerating parasympathetic and sympathetic fibers. The chorda tympani carries preganglionic parasympathetic fibers to the submandibular ganglion and taste fibers to the anterior two-thirds of the tongue. The facial nerve innervates the muscles of facial expression. The terminal part of the lacrimal nerve contains postganglionic parasympathetic fibers for lacrimation.

8. Following radical resection of a primary tongue tumor, a 72-year-old patient has lost general sensation on the anterior two-thirds of the tongue. This is probably due to injury to branches of which of the following nerves? (A) Trigeminal nerve (B) Facial nerve (C) Glossopharyngeal nerve (D) Vagus nerve (E) Hypoglossal nerve

8. The answer is A. The anterior two-thirds of the tongue is innervated by the lingual nerve, a branch of the mandibular division of the trigeminal nerve (CN V). The posterior one-third of the tongue is innervated by the glossopharyngeal nerve (CN IX) for general and taste sensations. The facial nerve supplies taste fibers to the tongue through the chorda tympani but does not supply general sensation. The vagus nerve supplies general sensation and taste sensation to the epiglottis by way of the internal laryngeal branch. The hypoglossal nerve innervates the tongue muscles

8. A 33-year-old male patient complains of feeling severe pain when he tries to turn his neck. The physician realizes that the problem is in his pivot (trochoid) joint. Which of the following joints would most likely be examined? (A) Atlantooccipital joint (B) Atlantoaxial joint (C) Carpometacarpal joint (D) Proximal tibiofibular joint (E) Intervertebral disks

8. The answer is B. The atlantoaxial joint is the pivot or trochoid joint. The atlantooccipital joints are the condyloid (ellipsoidal) joints, the carpometacarpal joint of the thumb is the saddle (sellar) joint, and the proximal tibiofibular joint is the plane (gliding) joint. The intervertebral disk is the secondary cartilaginous (symphysis) joint.

8. A 37-year-old small business manager receives a gunshot wound in the pelvic cavity, resulting in a lesion of the sacral splanchnic nerves. Which of the following nerve fibers would primarily be damaged? (A) Postganglionic parasympathetic fibers (B) Postganglionic sympathetic fibers (C) Preganglionic sympathetic fibers (D) Preganglionic parasympathetic fibers (E) Postganglionic sympathetic and parasympathetic fibers

8. The answer is C. The sacral splanchnic nerves consist primarily of preganglionic sympathetic neurons and also contain GVA fibers. None of the other fibers listed are contained in these nerves.

8. A 6-year-old girl comes to her pediatrician with constipation, abdominal distention, and vomiting. After thorough examination, she is diagnosed as having Hirschsprung disease (aganglionic megacolon), which is a congenital disease and leads to dilation of the colon. This condition is caused by an absence of which of the following kinds of neural cell bodies? (A) Sympathetic preganglionic neuron cell bodies (B) Sympathetic postganglionic neuron cell bodies (C) Parasympathetic preganglionic neuron cell bodies (D) Parasympathetic postganglionic neuron cell bodies (E) Sensory neuron cell bodies

8. The answer is D. Aganglionic megacolon (Hirschsprung disease) is caused by the absence of enteric ganglia (parasympathetic postganglionic neuron cell bodies) in the lower part of the colon, which leads to dilatation of the colon proximal to the inactive segment, resulting in an inability to evacuate the bowels. The other neuron cell bodies listed are not involved in this condition.

8. A 5-year-old girl is brought to the emergency department because of difficulty breathing (dyspnea), palpitations, and shortness of breath. Doppler study of the heart reveals an atrial septal defect (ASD). This malformation usually results from incomplete closure of which of the following embryonic structures? (A) Ductus arteriosus (B) Ductus venosus (C) Sinus venarum (D) Foramen ovale (E) Truncus arteriosus

8. The answer is D. An atrial septal defect (ASD) is a congenital defect in the interatrial septum. During partitioning of the two atria, the opening in the foramen secundum (the foramen ovale) usually closes at birth. If this foramen ovale is not closed completely, this would result in an ASD, shunting blood from the left atrium to the right atrium.

8. A patient comes in complaining that she cannot flex her proximal interphalangeal joints. Which of the following muscles appear(s) to be paralyzed on further examination of her finger? (A) Palmar interossei (B) Dorsal interossei (C) Flexor digitorum profundus (D) Flexor digitorum superficialis (E) Lumbricals

8. The answer is D. The flexor digitorum superficialis muscle flexes the proximal interphalangeal joints. The flexor digitorum profundus muscle flexes the DIP joints. The palmar and dorsal interossei and lumbricals can flex metacarpophalangeal joints and extend the interphalangeal joints. The palmar interossei adduct the fingers, and the dorsal interossei abduct the fingers.

9. A 34-year-old woman crashes into a tree during a skiing lesson and is brought to a hospital with multiple injuries that impinge the dorsal primary rami of several spinal nerves. Such lesions could affect which of the following muscles? (A) Rhomboid major (B) Levator scapulae (C) Serratus posterior superior (D) Iliocostalis (E) Latissimus dorsi

9. The Answer is D. The dorsal primary rami of the spinal nerves innervate the deep muscles of the back, including the iliocostalis. The other muscles are the superficial muscles of the back, which are innervated by the ventral primary rami of the spinal nerves

9. A 67-year-old patient has been given a course of antibiotics by gluteal intramuscular injections after a major abdominal surgery. To avoid damaging the sciatic nerve during an injection, the needle should be inserted into which of the following areas? (A) Over the sacrospinous ligament (B) Midway between the ischial tuberosity and the lesser trochanter (C) Midpoint of the gemelli muscles (D) Upper lateral quadrant of the gluteal region (E) Lower medial quadrant of the gluteal region

9. The Answer is D. To avoid damaging the sciatic nerve during an intramuscular injection, the clinician should insert the needle in the upper lateral quadrant of the gluteal region The inserted needle in the lower medial quadrant may damage the pudendal and sciatic nerves. The inserted needle midway between the ischial tuberosity and the lesser trochanter may damage the sciatic and posterior femoral cutaneous nerves on the quadratus femoris. The inserted needle over the sacrospinous ligament may damage the pudendal nerve and vessels.

9. A 53-year-old woman is diagnosed as having a pituitary tumor. If the tumor is large enough, she could exhibit which of the following disorders? (A) Blindness (B) Bitemporal (heteronymous) hemianopia (C) Right nasal hemianopia (D) Left homonymous hemianopia (E) Binasal hemianopia

9. The answer is B. Lesion of the optic chiasma by a pituitary tumor results in bitemporal hemianopia resulting from loss in the nasal field of vision of both eyes. Lesion of the optic nerve causes blindness. A right perichiasmal lesion by an aneurysm of the internal carotid artery leads to right nasal hemianopia because of loss of vision in the nasal field of the right eye. Lesion of the right optic tract or optic radiation causes left homonymous hemianopia resulting from loss of the left half of the visual fields of both eyes. Aneurysms of both internal carotid arteries cause right and left perichiasmal lesions, leading to binasal hemianopia (loss of vision in the nasal fields of both eyes).

9. A young girl complains of dryness of the nose and the palate. This would indicate a lesion of which of the following ganglia? (A) Nodose ganglion (B) Otic ganglion (C) Pterygopalatine ganglion (D) Submandibular ganglion (E) Ciliary ganglion

9. The answer is C. Postganglionic parasympathetic fibers originating in the pterygopalatine ganglion innervate glands in the palate and nasal mucosa. The postganglionic parasympathetic fibers from the otic ganglion supply the parotid gland, those from the submandibular ganglion supply the submandibular and sublingual glands, and those from the ciliary ganglion supply the ciliary muscle and sphincter pupillae. The nodose (inferior) ganglion of the vagus nerve is a sensory ganglion.

9. A patient presents with a loss of sensation to the skin over the shoulder. Injury to which of the following nerve cells would most likely affect the conduction of sensory information to the central nervous system? (A) Multipolar neurons (B) Bipolar neurons (C) Unipolar or pseudounipolar neurons (D) Neurons in the ventral horn (E) Neurons in sympathetic chain ganglia

9. The answer is C. Sensation from the skin is carried by GSA fibers, and their cells are unipolar or pseudounipolar types located in the dorsal root ganglia. Multipolar neurons and neurons in the ventral horn and in sympathetic chain ganglia are motor neurons. Bipolar neurons are sensory neurons, but they are not somatic sensory neurons.

9. A 21-year-old woman walks in with a shoulder and arm injury after falling during horseback riding. Examination indicates that she cannot adduct her arm because of paralysis of which of the following muscles? (A) Teres minor (B) Supraspinatus (C) Latissimus dorsi (D) Infraspinatus (E) Serratus anterior

9. The answer is C. The latissimus dorsi adducts the arm, and the supraspinatus muscle abducts the arm. The infraspinatus and the teres minor rotate the arm laterally. The serratus anterior rotates the glenoid cavity of the scapula upward, abducts the arm, and elevates it above a horizontal position.

9. A pediatric surgeon is resecting a possible malignant mass from the liver of a neonate with cerebral palsy. The surgeon divides the round ligament of the liver during surgery. A fibrous remnant of which of the following fetal vessels is severed? (A) Ductus venosus (B) Ductus arteriosus (C) Left umbilical vein (D) Right umbilical vein (E) Umbilical artery

9. The answer is C. The left umbilical vein becomes the round ligament of the liver after birth. The right umbilical vein did not leave a fibrous remnant because it was degenerated during the early embryonic period. The ductus venosus forms the ligamentum venosum; the ductus arteriosus forms the ligamentum arteriosum; the umbilical artery forms the medial umbilical ligament.

9. A 54-year-old patient is implanted with an artificial cardiac pacemaker. Which of the following conductive tissues of the heart had a defective function that required the pacemaker? (A) Atrioventricular (AV) bundle (B) AV node (C) Sinoatrial (SA) node (D) Purkinje fiber (E) Moderator band

9. The answer is C. The sinoatrial (SA) node initiates the impulse of contraction and is known as the pacemaker of the heart. Impulses from the SA node travel through the atrial myocardium to the AV node and then race through the AV bundle (bundle of His), which divides into the right and left bundle branches. The bundle breaks up into terminal conducting fibers (Purkinje fibers) to spread out into the ventricular walls. The moderate band carries the right limb of the AV bundle from the septum to the sternocostal wall of the ventricle.

9. A young couple is having difficulty conceiving a child. Their physician at a reproduction and fertility clinic explains to them that (A) The ovary lies within the broad ligament (B) The glans clitoris is formed from the corpus spongiosum (C) Erection of the penis is a sympathetic response (D) Ejaculation follows parasympathetic stimulation (E) Fertilization occurs in the infundibulum or ampulla of the uterine tube

9. The answer is E. Fertilization takes place in the infundibulum or ampulla of the uterine tube. The glans clitoris is derived from the corpora cavernosa, whereas the glans penis is the expanded terminal part of the corpus spongiosum. Erection of the penis is caused by parasympathetic stimulation, whereas ejaculation is mediated via the sympathetic nerve. The ovaries are not enclosed in the broad ligament, but their anterior surface is attached to the posterior surface of the broad ligament.


Kaugnay na mga set ng pag-aaral

Exam 2 Video Questions Intro To Supply Chain

View Set

Abeka 7th Grade Science, Section 13.3 Review

View Set

PLTW Comp Sci P: Unit 1 Chapter 1.1-1-4

View Set

Exam #3 (CH 62 - Mgmnt of Pts With Burn Injury)

View Set